You are on page 1of 61

C2017

Local Government Law


Midterms Reviewer
C2017 / LAW 154 / Local Government Law / Midterm Reviewer

Table of Contents
I. HISTORICAL BACKGROUND OF LOCAL GOVERNMENTS IN THE PHILIPPINES .......................................................................................... 3
HISTORY OF PHILIPPINE OF LOCAL GOVERNMENT AND ADMINISTRATION (TAPALES) ................................................................................ 3
II. NATURE AND STATUS ...................................................................................................................................................................... 4
A. MUNICIPAL CORPORATION, LOCAL GOVERNMENT, DEFINED, LOCAL GOVERNMENT INTERCHANGEABLE WITH MUNICIPAL
CORPORATIONS............................................................................................................................................................................................... 4
B. NATURE AND STATUS OF MUNICIPAL CORPORATIONS ............................................................................................................................. 4
C. KINDS OF MUNICIPAL CORPORATIONS....................................................................................................................................................... 4
D. DUAL NATURE ............................................................................................................................................................................................. 5
1. DUAL NATURE AND FUNCTIONS OF MUNICIPAL CORPORATIONS ......................................................................................................... 5
2. PURPOSE .................................................................................................................................................................................................. 5
3. GENERAL POWERS AND ATTRIBUTES ..................................................................................................................................................... 6
III. GENERAL PRINCIPLES AND POLICIES...............................................................................................................................................14
A. LOCAL AUTONOMY ..................................................................................................................................................................................... 15
B. DECENTRALIZATION ................................................................................................................................................................................... 17
IV. CREATION OF MUNICIPAL CORPORATIONS ......................................................................................................................................18
A. NATURE AND POWES TO CREATE MUNICIPAL CORPORATION ................................................................................................................. 18
B. CREATION OF MUNICIPAL COPORATIONS ................................................................................................................................................. 18
1. CONSTITUTIONAL PROVISIONS ............................................................................................................................................................... 18
3. REVISED ADMIN CODE, SEC 68. ............................................................................................................................................................. 20
4. MUNICIPAL CORPOATION BY PRESCRIPTION ........................................................................................................................................ 21
5. DE FACTO MUNICIPAL CORPORATIONS ................................................................................................................................................. 21
6. ATTACK AGAINST VALIDITY OF MUNICIPAL CORPORATIONS ................................................................................................................ 21
7. BEGINNING OF CORPORATE EXISTENCE OF MUNICIPAL CORPORATIONS............................................................................................ 21
V. ALTERATION AND DISSOLUTION OF MUNICIPAL CORPORATIONS ...................................................................................................... 22
A. NATURE OF POWER ................................................................................................................................................................................... 22
1. NECESSITY FOR DEFINING TERRITORIAL BOUNDARIES ........................................................................................................................ 22
B. MANNER OR MODE .................................................................................................................................................................................... 22
C. EFFECTS ..................................................................................................................................................................................................... 23
1. EFFECTS OF ANNEXATION OR CONSOLIDATION OF MUNICIPAL CORPORATIONS............................................................................... 23
2. EFFECTS OF DIVISION OF MUNICIPAL CORPORATIONS ........................................................................................................................ 23
D. WHEN THERE IS NO DISSOLUTION............................................................................................................................................................ 23
1. NON-USER OR SURRENDER OF CHARTER ............................................................................................................................................. 23
2. FAILURE TO ELECT MUNICIPAL OFFICERS ............................................................................................................................................. 23
3. CHANGE OF SOVEREIGNTY .................................................................................................................................................................... 23
VI. PLEBISCITE REQUIREMENTS .......................................................................................................................................................... 25
VII. GENERAL POWERS OF LOCAL GOVERNMENTS ............................................................................................................................... 26
A. POLICE POWER .......................................................................................................................................................................................... 26
GENERAL WELFARE ................................................................................................................................................................................... 26
CLOSURE AND OPENING OF ROADS ......................................................................................................................................................... 29

Bautista, Gastanes, Lopez, Sulit Page | 1


C2017 / LAW 154 / Local Government Law / Midterm Reviewer
B. EMINENT DOMAIN....................................................................................................................................................................................... 31
RULE 67 EXPROPRIATION ........................................................................................................................................................................... 31
C. TAXATION AND FISCAL ADMINISTRATION ................................................................................................................................................ 35
1. LOCAL TAXATION .................................................................................................................................................................................... 35
2. REAL PROPERTY TAXATION ................................................................................................................................................................... 39
3. SHARES OF LGUS IN THE PROCEEDS OF NATIONAL TAXES ................................................................................................................... 41
4. CREDIT FINANCING, LOANS, CREDITS AND OTHER FORMS OF INDEBTEDNESS OF LGUS .................................................................. 42
5. LOCAL FISCAL ADMINSTRATION............................................................................................................................................................ 42
VIII. LOCAL GOVERNMENT UNITS ........................................................................................................................................................ 44
A. THE BARANGAY ......................................................................................................................................................................................... 46
1. KATARUNGANG PAMBARANGAY............................................................................................................................................................ 47
2. SANGGUNIANG KABATAAN.................................................................................................................................................................... 50
B. THE MUNICIPALITY..................................................................................................................................................................................... 53
C. THE CITY ..................................................................................................................................................................................................... 54
C. THE PROVINCE ........................................................................................................................................................................................... 54
E. APPLICATION OF THE CODE TO LGU IN AUTONOMOUS REGION ............................................................................................................. 54
F. THE AUTONOMOUS REGION IN MUSLIM MINDANAO ............................................................................................................................... 54
G. CORDILLERA ADMINISTRATIVE REGION ................................................................................................................................................... 56
H. THE METROPOLITAN MANILA DEVELOPMENT AUTHORITY ......................................................................................................................57
IX. MUNICIPAL OFFICERS AND EMPLOYEES ......................................................................................................................................... 58
A. ELECTIVE OFFICIALS .................................................................................................................................................................................. 58
1. QUALIFICATIONS ..................................................................................................................................................................................... 58

Bautista, Gastanes, Lopez, Sulit Page | 2


C2017 / LAW 154 / Local Government Law / Midterm Reviewer

I. HISTORICAL BACKGROUND OF LOCAL GOVERNMENTS


IN THE PHILIPPINES
In the ’71 Constitutional Convention in the draft Constitution a separate
History of Philippine of Local Government and
article on local government was provided giving emphasis on the process of
Administration (Tapales) local governance. The article also mandated that congress should enact a
Local Government Code which would serve as guide in the organization,
The Malolos Constitution provided for the creation of the municipal powers and functions of local authorities.
assemblies to administer the affairs of the towns and provincial assemblies
for the provinces. It provided for autonomous local units but recognized a The Constitution was ratified in early ’73 by citizen assemblies. The President
certain degree of supervision or intervention from the central government. named the barrios into barangays and designated 15 year-old and older
barangay residents as members of citizen assemblies
Local Government in the Post-War Era
’35 Constitution and Act Nos. 82 and 83 which were incorporated the Here, there was also a creation of a region-wide government for Metropolitan
Administrative Code became the legal basis for the system of the local Manila it was the result of studies and proposals made by urban experts who
governance in the country even after political independence from America in prepared different organization models.
1946, and until the Constitution was revised and Martial Law decreed in 1972.
Taken together with PD 826 (alters membership of local development
The ’35 Constitution mentioned local government only in connection with the councils which expended the membership of local councils and made many
President’s power of general supervision in Art 7 Sec 10. This provision was a of Metro Manila councils too big and the strong personality of Metro Manila
result of a compromise. It provides that Presidential supervision over local Governor, PD 824 brought about a weakening of the cities and municipalities
government “as may be provided by law”. of Metro Manila. Moreover, the MMC derived its funds for the integrated
Thus, while the President has the power of general supervision, Congress services from their contributions in the amount of 20% each of their annual
may determine the parameters of the supervision through legislation. income, thus crippling the delivery of their local basic services.

Exercising its authority, Congress passed several measures affecting local PD 826 renamed local councils into Sanggunians including the sectoral
government: representatives, barangay captains, kabataang barangay.
 RA 2259 (Omnibus Law on Cities) - Provides for some uniformity in the
governance of cities In ’83 the Local Gov’t Code BP 337 was enacted. Noteworthy of all the
 RA 2370 as amended by RA 3590 (The Barrio Charter) - Provided for the presidential decrees were: PD 144 which pertained to the sharing of internal
election of barrio officials revenue allotments, along with PD 231 or the Local Tax Code which
enumerated te revenue-raising powers and fund sources of each level of local
 RA 2264 (Local Autonomy Act) – Pertained to greater taxing powers to
authority and PD 464 the decree on local fiscal taxation. These and other
cities and municipalities
laws on local government were codified in BP 337.
 RA 5185 (The Decentralization Act) – Authorized local governments to
supplement the national government’s efforts in agricultural extension
People Power at EDSA
and rural health work.
Pimentel became senator in ’87 he started work on the new LGC. The Code
 Municipalities were governed by Act No.82 and the provinces, by Act No.
was not enacted until congress reached almost the end of its first term. It was
83. The cities were governed by their own charters which were enacted by
signed into law on 10 Oct 1991 and it was implemented in 1992.
Congress
 ’35 Constitution and Act No.82 and 83: Legal Basis of Local Gov’t ’46-’72. Its most important feature is the decentralization of 5 basic services- health,
agriculture, social welfare, public works, and environmental and natural
Supervision was delegated to the Department of the Interior. However, The resources. The decentralization mandated in’91 Code goes beyond the
department was abolished after the notorious Presidential elections of 1949. devolution of local services. It increased participation in the process of
During this time the LGUs had the following main sources of revenue to tap: governance and local development through 1) sectoral representation in the
(1) Taxation, including IRA from national government; real property tax, local legislative councils; 2) membership of non-governmental
municipal or city taxes and other taxes organizations’ representatives in local boards and committees; 3)
(2) Leasing public utilities such as markets, waterworks, electricity, and participation in the political exercises like plebiscite, referendum, and recall;
transportation; and 4) involvement in the planning and implementation of development
(3) Intergovernmental revenue sources, including direct national aid and programs
other types of assistance; and
(4) Miscellaneous sources, like tuition fees and sale of bonds

Local Government Under Martial Law


The government embarked upon an Integrated Reorganization Plan. It had
the following features which would affect local governments:
(1) A regional delineation of 11 regions, where a regional center was
located for each, to house the deconcentrated agencies
(2) Department of Local Government and Community Development which
would put together under one roof of agencies having direct
relationships with local government

Bautista, Gastanes, Lopez, Sulit Page | 3


C2017 / LAW 154 / Local Government Law / Midterm Reviewer

II. NATURE AND STATUS

A. MUNICIPAL CORPORATION, LOCAL C. KINDS OF MUNICIPAL CORPORATIONS


GOVERNMENT, DEFINED, LOCAL GOVERNMENT
INTERCHANGEABLE WITH MUNICIPAL Provisions
CORPORATIONS
New Civil Code
Alvarez v Guingona
Facts: RA 7720 was passed converting the municipality of Santiago Article 44. The following are juridical persons:
(1) The State and its political subdivisions;
into an ICC. The petitioners are assailing the constitutionality of the
law on the ground that the income of Santiago, after deducting the (2) Other corporations, institutions and entities for public interest
or purpose, created by law; their personality begins as soon as
IRA, does not meet the income requirement for conversion. Court
ruled that IRA is included in the computation for the income they have been constituted according to law;
requirement (3) Corporations, partnerships and associations for private interest
or purpose to which the law grants a juridical personality,
Held: the IRA regularly and automatically accrues to the local separate and distinct from that of each shareholder, partner or
member.
treasury without need of any further action on the part of the local
government unit. It is the local government’s source of income so it
Article 45. Juridical persons mentioned in Nos. 1 and 2 of the
is part of the income requirement.
preceding article are governed by the laws creating or recognizing
A Local Government Unit is a political subdivision of the State
which is constituted by law and possessed of substantial control them.
Private corporations are regulated by laws of general
over its own affairs. Remaining to be an intra-sovereign subdivision
of one sovereign nation, but not intended, however, to be an application on the subject.
imperium in imperio, the local government unit is autonomous in the Partnerships and associations for private interest or purpose
are governed by the provisions of this Code concerning partnerships.
sense that it is given more powers, authority, responsibilities and
resources.
Article 46. Juridical persons may acquire and possess property of all
kinds, as well as incur obligations and bring civil or criminal actions,
Basco v PAGCOR in conformity with the laws and regulations of their organization.
Facts: Petitioners filed a petition seeking to annul the PAGCOR (38a)
charter for being contrary to morals, public policy and order. Also
that it is a waiver of a right prejudicial to Manila City government’s Article 47. Upon the dissolution of corporations, institutions and
right to impose taxes and license fees, which is recognized by law 2) other entities for public interest or purpose mentioned in No. 2 of
this is in contravention of constitutionally enshrined principle of article 44, their property and other assets shall be disposed of in
local autonomy 3) it violates the equal protection clause. Court ruled pursuance of law or the charter creating them. If nothing has been
for PAGCOR. specified on this point, the property and other assets shall be
applied to similar purposes for the benefit of the region, province,
Held: SC upheld the validity of the charter. It does not contravene city or municipality which during the existence of the institution
the principle of local autonomy because municipal corporations are derived the principal benefits from the same.
mere creatures of Congress, which the power to create and abolish
municipal corporations due to its general legislative powers.
Notes
Congress thus may control Local Governments. If Congress can
grant the City the power to tax, then it may provide exemptions or
(1) Municipal Corporation Proper
even take back the power. The principle of local autonomy does not
make local governments an imperium in imperio. It is a measure for  Generally term refers to incorporated cities, towns, or villages
decentralization of the functions of the government. invested with the power of local legislation
 The power of local government is the distinctive purpose and
distinguishing feature of a municipal corporation
B. NATURE AND STATUS OF MUNICIPAL
CORPORATIONS (2) Quasi-Municipal Corporation
 This is another terms for quasi- corporation
 Quasi-Corporation are public corporations created as agencies of
the state for a narrow and limited purpose
o Unlike other public corporations they are not possessed of the
powers and liabilities of self-governing corporations

Bautista, Gastanes, Lopez, Sulit Page | 4


C2017 / LAW 154 / Local Government Law / Midterm Reviewer
o Their power generally related to matters of State, as political subdivision of the national government and as a corporate
distinguished from municipal concern entity representing the inhabitants of its territory.
o They possess a limited number of corporate powers and have
a low grade of corporate existence Breakdown of Sec 15, LGC:
o The main purpose of their creation is to aid the state in, or to Every LGU created or recognized under the LGC is a body politic and
take charge of, some public, or state work other than corporate
community government, for the general welfare A. Endowed with powers to be exercised in conformity with
o EXAMPLES: NPC, NAWASA, DBP, Boy Scouts of the PH law
B. It shall exercise powers as:
(3) Distinguish Municipal Corporation Proper from Quasi-Municipal o Political Subdivision of the National Government
Corporation o Corporate Entity representing the inhabitants of
Municipal Corporation Proper Quasi- Municipal Corporation/ its territory
Quasi- Corporation
A public corporation Also a public Corporation 2. Purpose
Has the power of local Agents of the state for limited
government and narrow, purposes
It is created for a 2-Fold purpose:
Has the powers and liabilities Invested with a few of
of self-government characteristics of corporates Governmental Private
existence for purpose of civil It is a representative of the Its act in a similar category as a
administration State for the government of business corporation,
Called into existence either at Sometimes involuntary the territory and the performing functions not
the direct solicitation of by the corporations, created by the inhabitants within the strictly governmental or
free consent of the persons sovereign legislative power of municipal limits political
composing them its own sovereign will without It exercises by delegation a It stands for the community in
solicitation, consent or part of the sovereignty of the the administration of local
concurrent action of the State affairs which is wholly beyond
people who inherit them the sphere of the public
purposes for which the
(4) Distinguish Municipal Corporation from Public Corporation governmental powers are
conferred
 Public Corporations are those formed or organized for the
government of a portion of the State
Villas v City of Manila
 Those organized for political purposes, with political powers to be
Facts: Manila City, under Spanish rule, owed debts to Villas, et al.
exercised for purposes connected with the public good in the
Villas now claims the debts against Manila, which is now under
administration of civil government; an instrument of the
American rule. Manila asserts that it is a separate entity from the
government, subject to the control of the legislature and its
Spanish Manila and should not be made liable for the latter’s
members, officers of the government, for the administration or
liabilities. US SC rules in favor of Villas.
discharge of public duties as in the case of cities and towns.
(Regents of University v Williams)
Held: Municipal governments have a dual character – governmental
 All municipal corporations are public corporations but not all
and private/business. In view of this dual character, there is no
public corporations are municipal corporations. Municipal
reason for presuming their dissolution of municipal corps. as a mere
Corporations are a subset of public Corporations.
consequence of territorial cession. With respect to laws affecting
 A non-municipal public corporations is one created by the state (among others) possession, use, and transfer of property, which are
as its own agency or instrumentality to carry-out a certain public
strictly of a municipal character, the general rule is that a change of
purpose other than the local government carried on in a government leaves them in force until, by direct action of the new
designated portion of its territory by municipal corporations
government, they are altered or repealed

D. DUAL NATURE Lidasan v COMELEC


Facts: RA 4790, an act creating the Municipality of DIanaton in the
1. Dual Nature and Functions of Municipal Province of Lanao Del Sur, was being challenged as unconstitutional
Corporations for incorporation 12 barrios of the Province of Cotabato. The
Supreme Court declared the RA null and void for having a
misleading title since the title did not reflect the transfer of a
Provisions sizeable portion of one province to another.

SECTION 15. Political and Corporate Nature of Local Government Units. Held: With regard to the argument that the statute could be
Every local government unit created or recognized under this Code salvaged by removing the 12 provinces outside Lanao Del Sur, the
is a body politic and corporate endowed with powers to be exercised SC declared that it could not since it was the intent of the legislature.
by it in conformity with law. As such, it shall exercise powers as a It further stated that Municipal corporations perform twin functions.

Bautista, Gastanes, Lopez, Sulit Page | 5


C2017 / LAW 154 / Local Government Law / Midterm Reviewer
Firstly, they serve as an instrumentality of the State in carrying out (a) Income. It must be sufficient, based on acceptable standards,
the functions of government. Secondly, they act as an agency of the to provide for all essential government facilities and services
community in the administration of local affairs. It is in the 2nd and special functions commensurate with the size of its
character that they are a separate entity acting for their own purposes population, as expected of the local government unit
and not a subdivision of the State. The Court considered income and concerned;
population of the remaining 9 states and doubted if they could (b) Population. It shall be determined as the total number of
maintain an independent municipality. inhabitants within the territorial jurisdiction of the local
government unit concerned; and
Republic of the PH v City of Davao (c) Land Area. It must be contiguous, unless it comprises two (2) or
Facts: The City of Davao filed an application for a Certificate of Non- more islands or is separated by a local government unit
Coverage for its proposed project, the Davao City Artica Sports independent of the others; properly identified by metes and
Dome. The Environmental Management Bureau denied the bounds with technical descriptions; and sufficient to provide for
application, finding that the 7 proposed project was within an such basic services and facilities to meet the requirements of its
environmentally critical area. Davao City filed a petition for populace. Compliance with the foregoing indicators shall be
mandamus and injunction with RTC Davao for the issuance of the attested to by the Department of Finance (DOF), the National
CNC, which was granted by the court. Statistics Office (NSO), and the Lands Management Bureau
(LMB) of the Department of Environment and Natural
Held: The SC held that the City of Davao was entitled to a CNC. The Resources (DENR).
Court found that Davao City cannot claim exemption from the
coverage of P.D. 1586 (Environmental Impact Statement System). Breakdown of Sec 7, LGC:
As a body politic and corporate endowed with powers to be exercised GEN RULE: Creation of LGU or its conversion from one level to
by it in conformity with law, it performs dual functions: another shall be based on verifiable indicators of viability and
governmental and proprietary. As an LGU it has the duty to promote projected capacity to provide services:
the people’s right to a balanced ecology. However, it is still entitled
to the CNC as the trial court found that the Artica Sports Dome is (a) Income
not within an environmentally critical area, nor is it an  It must be sufficient to provide for all essential government
environmentally critical project. facilities and special functions commensurate with population
 Based on acceptable standards
3. General Powers and Attributes (b) Population- Total number must be within territorial jurisdiction
(c) Land Area
 Must be contiguous UNLESS:
Provisions o Comprises of 2 or more islands, or
o Separated by LGU independent of the others
Local Government Code (PD 7160)  Must be properly Identified by metes and bounds with technical
descriptions
SECTION 6. Authority to Create Local Government Units.  Must be sufficient to provide for basic services and facilities
A local government unit may be created, divided, merged, (d) Compliance to these indicators shall be attested to by:
abolished, or its boundaries substantially altered either by law  DOF
enacted by Congress in the case of a province, city, municipality, or
 NSO
any other political subdivision, or by ordinance passed by the
 LMB
sangguniang panlalawigan or sangguniang panlungsod concerned
 DENR
in the case of a barangay located within its territorial jurisdiction,
subject to such limitations and requirements prescribed in this Code.
SECTION 8. Division and Merger. Division and merger of existing local
government units shall comply with the same requirements herein
Breakdown of Sec 6, LGC:
prescribed for their creation: Provided, however, That such division
An LGU may be created, divided, merged, abolished, or its
shall not reduce the income, population, or land area of the local
boundaries substantially altered either by:
government unit or units concerned to less than the minimum
(1) Law enacted by congress- for province, city, municipality, or any
requirements prescribed in this Code: Provided, further, That the
other political subdivision, or
income classification of the original local government unit or units
(2) Ordinance by Sangguiang Panlalawigan or Sangguniang
shall not fall below its current income classification prior to such
Panglungsod- for barangay located within its territorial
division.
jurisdiction
The income classification of local government units shall be
Subject to limitations and requirements prescribed by LGC
updated within six (6) months from the effectivity of this Code to
reflect the changes in their financial position resulting from the
SECTION 7. Creation and Conversion. As a general rule, the creation
increased revenues as provided herein.
of a local government unit or its conversion from one level to another
level shall be based on verifiable indicators of viability and projected
capacity to provide services, to wit:

Bautista, Gastanes, Lopez, Sulit Page | 6


C2017 / LAW 154 / Local Government Law / Midterm Reviewer
Breakdown of Sec 8, LGC:  Within 120 days from its effectively
Division and Merger shall comply with the same requirements as  On the date fixed by law or ordinance effecting such action
creation, Provided: In Tan v COMELEC the LGUs directly affected should participate in the
(1) Division shall not reduce income, population, or land area plebescite which means the newly created LGU and the parent LGU.
(2) Income classification of original LGU shall NOT fall below its
current income classification prior to such division SECTION 11. Selection and Transfer of Local Government Site, Offices
Income classification of LGU shall be updated within 6mos from LGC and Facilities.
effectivity (a) The law or ordinance creating or merging local government
units shall specify the seat of government from where
SECTION 9. Abolition of Local Government Units. A local government governmental and corporate services shall be delivered. In
unit may be abolished when its income, population, or land area has selecting said site, factors relating to geographical centrality,
been irreversibly reduced to less than the minimum standards accessibility, availability of transportation and communication
prescribed for its creation under Book III of this Code, as certified by facilities, drainage and sanitation, development and economic
the national agencies mentioned in Section 7 hereof to Congress or progress, and other relevant considerations shall be taken into
to the sanggunian concerned, as the case may be. account.
The law or ordinance abolishing a local government unit shall (b) When conditions and developments in the local government
specify the province, city, municipality, or barangay with which the unit concerned have significantly changed subsequent to the
local government unit sought to be abolished will be incorporated establishment of the seat of government, its sanggunian may,
or merged. after public hearing and by a vote of two-thirds (2/3) of all its
members, transfer the same to a site better suited to its needs.
Breakdown of Sec 9, LGC: Provided, however, That no such transfer shall be made outside
An LGU may be abolished when its income, population, or land area the territorial boundaries of the local government unit
has been irreversibly reduced to less than the minimum standards concerned.
prescribed for its creation The old site, together with the improvements thereon, may
 Certified by national agencies in Sec.7 to Congress or to the be disposed of by sale or lease or converted to such other use
Sangguian concerned, as the case may be as the sanggunian concerned may deem beneficial to the local
Law or ordinance abolishing the LGU shall specify the province, city, government unit concerned and its inhabitants.
municipality; barangay with which the LGU sought to be abolished (c) Local government offices and facilities shall not be transferred,
will be incorporated or merged. relocated, or converted to other uses unless public hearings are
first conducted for the purpose and the concurrence of the
It does result in an automatic cessation of the LGU; Congress or the majority of all the members of the sanggunian concerned is
Sanggunian concerned must pass a law or ordinance for the abolition, obtained.
and such must be subjected to a plebiscite (As was said in Sultan
Usman Sarangani v COMELEC) Breakdown of Sec 11, LGC:
(1) Law or ordinance creating or merging the LGUs shall specify the
IMPT: Dissolution does not occur due to: seat of government from where governmental and corporate
(1) Non-User or surrender of charter services shall be delivered; factors to take into account:
(2) Failure to elect municipal Officers  Geographical Centrality
(3) Change of Sovereignty  Accessibility
(4) Change of Name  Availability of transportation and communication
 Drainage and Sanitation
SECTION 10. Plebiscite Requirement. No creation, division, merger,
 Other relevant considerations
abolition, or substantial alteration of boundaries of local
(2) When conditions and developments in the LGU have
government units shall take effect unless approved by a majority of
significantly changed subsequent to establishment of seat of
the votes cast in a plebiscite called for the purpose in the political
government, LGU sanggunian may transfer the same to a site
unit or units directly affected. Said plebiscite shall be conducted by
better suited to its needs
the Commission on Elections (Comelec) within one hundred twenty
 How?
(120) days from the date of effectivity of the law or ordinance
o Public Hearing
effecting such action, unless said law or ordinance fixes another
o Vote of 2/3 of its members
date.
 No such transfer shall be made outside its territorial
Breakdown of Sec 10, LGC: boundaries
No creation, dicion, merger, abolition, orsubtantial alteration of  Old site and its improvements may be disposed of by sale or
boundaries of LGUs shall take effect UNLESS approved by a majority lease or converted to such other uses as he Sanggunian may
of the votes cost in a PLEBISCITE: deem beneficial
(1) Plebiscite called for that purpose (3) Local Government offices and facilities shall NOT be transferred,
(2) In the Political Unit/s directly affected relocated, or converted to other uses UNLESS
(3) Conducted by COMELEC  Public hearings are first conducted AND

Bautista, Gastanes, Lopez, Sulit Page | 7


C2017 / LAW 154 / Local Government Law / Midterm Reviewer
 Concurrence of MAJORITY of Sanggunian members change the name of the following within its territorial
jurisdiction:
This provision concerns itself with the physical location or situs of the (1) City and municipal barangays, upon recommendation of
seats or capital of the LGU the sangguniang barangay concerned;
(2) City, municipal and barangay roads, avenues, boulevards,
SECTION 12. Government Centers. Provinces, cities, and thoroughfares, and bridges;
municipalities shall endeavour to establish a government center (3) City and municipal public elementary, secondary and
where offices, agencies, or branches of the National Government, vocational or technical schools, post-secondary and other
local government units, or government-owned or controlled tertiary schools;
corporations may, as far as practicable, be located. In designating (4) City and municipal hospitals, health centers and other
such a center, the local government unit concerned shall take into health facilities; and
account the existing facilities of national and local agencies and (5) Any other public place or building owned by the municipal
offices which may serve as the government center as contemplated government.
under this Section. The National Government, local government unit (d) None of the foregoing local government units, institutions,
or government-owned or controlled corporation concerned shall places, or buildings shall be named after a living person, nor
bear the expenses for the construction of its buildings and facilities may a change of name be made unless for a justifiable reason
in the government center. and, in any case, not oftener than once every ten (10) years. The
name of a local government unit or a public place, street or
This section, as was said in the case of SAMSON v AGUIRRE, speaks structure with historical, cultural, or ethnic significance shall
of the site of government centres, such site can very well also be the not be changed, unless by a unanimous vote of the sanggunian
seat of government, “from where governmental and corporate service concerned and in consultation with the PHC.
shall be delivered.” (e) A change of name of a public school shall be made only upon
the recommendation of the local school board concerned.
SECTION 13. Naming of Local Government Units and Public Places, (f) A change of name of public hospitals, health centers, and other
Streets and Structures. health facilities shall be made only upon the recommendation
(a) The sangguniang panlalawigan may, in consultation with the of the local health board concerned.
Philippine Historical Commission (PHC), change the name of (g) The change of name of any local government unit shall be
the following within its territorial jurisdiction: effective only upon ratification in a plebiscite conducted for the
(1) Component cities and municipalities, upon the purpose in the political unit directly affected.
recommendation of the sanggunian concerned; (h) In any change of name, the Office of the President, the
(2) Provincial roads, avenues, boulevards, thoroughfares, and representative of the legislative district concerned, and the
bridges; Bureau of Posts shall be notified.
(3) Public vocational or technical schools and other
postsecondary and tertiary schools;  This provision provides that the Spanlalawigan may change the
(4) Provincial hospitals, health centers, and other health name of a component city if done upon the recommendation of the
facilities; and Sanggunian concerned.
(5) Any other public place or building owned by the provincial  Spanlungsod changing the name of barangays can only do so upon
government. recommendation of the Sbrgy concerned.
(b) The sanggunians of highly urbanized cities and of component  Cannot be names after a LIVING PERSON, nor may change of name
cities whose charters prohibit their voters from voting for be made UNLESS for a justifiable reason, IN ANY CASE, not oftener
provincial elective officials, hereinafter referred to in this Code than ONCE every 10 years.
as independent component cities, may, in consultation with the
Philippine Historical Commission, change the name of the SECTION 14. Beginning of Corporate Existence. When a new local
following within its territorial jurisdiction: government unit is created, its corporate existence shall commence
(1) City barangays, upon the recommendation of the upon the election and qualification of its chief executive and a
sangguniang barangay concerned; majority of the members of its sanggunian, unless some other time
(2) City roads, avenues, boulevards, thoroughfares, and is fixed therefor by the law or ordinance creating it.
bridges;
(3) Public elementary, secondary and vocational or technical Breakdown of Sec 14, LGC:
schools, community colleges and non-chartered colleges; When a new LGU is created, corporate existence shall commence:
(4) City hospitals, health centers and other health facilities; (1) Upon ELECTION and QUALIFICATION of
and (a) Its CHIEF EXECUTIVE, and
(5) Any other public place or building owned by the city (b) Majority of the members of its sanggunian
government. (2) UNLESS another time is fixed by the LAW or ORDINANCE
(c) The sanggunians of component cities and municipalities may, creating it
in consultation with the Philippine Historical Commission,

Bautista, Gastanes, Lopez, Sulit Page | 8


C2017 / LAW 154 / Local Government Law / Midterm Reviewer
SECTION 15. Political and Corporate Nature of Local Government Units. distribution channels, preferably through cooperatives;
Already discussed earlier. inter-barangay irrigation systems; water and soil resource
utilization and conservation projects; and enforcement of
SECTION 16. General Welfare. Every local government unit shall fishery laws in municipal waters including the conservation
exercise the powers expressly granted, those necessarily implied of mangroves;
therefrom, as well as powers necessary, appropriate, or incidental (ii) Pursuant to national policies and subject to supervision,
for its efficient and effective governance, and those which are control and review of the DENR, implementation of
essential to the promotion of the general welfare. Within their Community-based forestry projects which include integrated
respective territorial jurisdictions, local government units shall
social forestry programs and similar projects; management
ensure and support, among other things, the preservation and
and control of communal forests with an area not exceeding
enrichment of culture, promote health and safety, enhance the right
fifty (50) square kilometers; establishment of tree parks,
of the people to a balanced ecology, encourage and support the
greenbelts, and similar forest development projects;
development of appropriate and self-reliant scientific and
(iii) Subject to the provisions of Title Five, Book I of this Code,
technological capabilities, improve public morals, enhance
health services which include the implementation of
economic prosperity and social justice, promote full employment
programs and projects on primary health care, maternal and
among their residents, maintain peace and order, and preserve the
child care, and communicable and non-communicable
comfort and convenience of their inhabitants.
disease control services; access to secondary and tertiary
SECTION 17. Basic Services and Facilities. health services; purchase of medicines, medical supplies,
(a) Local government units shall endeavor to be self-reliant and and equipment needed to carry out the services herein
shall continue exercising the powers and discharging the duties enumerated;
and functions currently vested upon them. They shall also (iv) Social welfare services which include programs and projects
discharge the functions and responsibilities of national on child and youth welfare, family and community welfare,
agencies and offices devolved to them pursuant to this Code. women’s welfare, welfare of the elderly and disabled
Local government units shall likewise exercise such other persons; community-based rehabilitation programs for
powers and discharge such other functions and responsibilities vagrants, beggars, street children, scavengers, juvenile
as are necessary, appropriate, or incidental to efficient and delinquents, and victims of drug abuse; livelihood and other
effective provision of the basic services and facilities Pro-poor projects; nutrition services; and family planning
enumerated herein. services;
(b) Such basic services and facilities include, but are not limited to, (v) Information services which include investments and job
the following: placement information systems, tax and marketing
information systems, and maintenance of a public library;
(1) For a Barangay: (vi) Solid waste disposal system or environmental management
(i) Agricultural support services which include planting system and services or facilities related to general hygiene
materials distribution system and operation of farm produce and sanitation;
collection and buying stations; (vii) Municipal buildings, cultural centers, public parks including
(ii) Health and social welfare services which include freedom parks, playgrounds, and other sports facilities and
maintenance of barangay health center and daycare center; equipment, and other similar facilities;
(iii) Services and facilities related to general hygiene and (viii) Infrastructure facilities intended primarily to service the
sanitation, beautification, and solid waste collection; needs of the residents of the municipality and which are
(iv) Maintenance of katarungang pambarangay; funded out of municipal funds including, but not limited to,
(v) Maintenance of barangay roads and bridges and water municipal roads and bridges; school buildings and other
supply systems; facilities for public elementary and secondary schools;
(vi) Infrastructure facilities such as multipurpose hall, multi- clinics, health centers and other health facilities necessary to
purpose pavement, plaza, sports center, and other similar carry out health services; communal irrigation, small water
facilities; impounding projects and other similar projects; fish ports;
(vii) Information and reading center; and artesian wells, spring development, rainwater collectors and
(viii) Satellite or public market, where viable; water supply systems; seawalls, dikes, drainage and
sewerage, and flood control; traffic signals and road signs;
(2) For a Municipality: and similar facilities;
(i) Extension and onsite research services and facilities related (ix) Public markets, slaughterhouses and other municipal
to agriculture and fishery activities which include dispersal of enterprises;
livestock and poultry, fingerlings, and other seeding
(x) Public cemetery;
materials for aquaculture; palay, corn, and vegetable seed
(xi) Tourism facilities and other tourist attractions, including the
farms; medicinal plant gardens; fruit tree, coconut, and other
acquisition of equipment, regulation and supervision of
kinds of seedling nurseries; demonstration farms; quality business concessions, and security services for such facilities;
control of copra and improvement and development of local and

Bautista, Gastanes, Lopez, Sulit Page | 9


C2017 / LAW 154 / Local Government Law / Midterm Reviewer
(xii) Sites for police and fire stations and substations and (c) Notwithstanding the provisions of subsection (b) hereof, public
municipal jail; works and infrastructure projects and other facilities, programs
and services funded by the National Government under the
(3) For a Province: annual General Appropriations Act, other special laws,
(i) Agricultural extension and onsite research services and pertinent executive orders, and those wholly or partially funded
facilities which include the prevention and control of plant from foreign sources, are not covered under this section, except
and animal pests and diseases; dairy farms, livestock in those cases where the local government unit concerned is
markets, animal breeding stations, and artificial duly designated as the implementing agency for such projects,
insemination centers; and assistance in the organization of facilities, programs, and services.
farmers’ and fishermen’s cooperatives and other collective (d) The designs, plans, specifications, testing of materials, and the
organizations, as well as the transfer of appropriate procurement of equipment and materials from both foreign
technology; and local sources necessary for the provision of the foregoing
(ii) (ii) Industrial research and development services, as well as services and facilities shall be undertaken by the local
government unit concerned, based on national policies,
the transfer of appropriate technology;
standards and guidelines.
(iii) Pursuant to national policies and subject to supervision,
(e) National agencies or offices concerned shall devolve to local
control and review of the DENR, enforcement of forestry laws
government units the responsibility for the provision of basic
limited to community-based forestry projects, pollution
services and facilities enumerated in this section within six (6)
control law, small-scale mining law, and other laws on the
months after the effectivity of this Code.
protection of the environment; and minihydroelectric
As used in this Code, the term "devolution" refers to the act
projects for local purposes;
by which the National Government confers power and authority
(iv) Subject to the provisions of Title Five, Book I of this Code,
upon the various local government units to perform specific
health services which include hospitals and other tertiary
functions and responsibilities.
health services; (f) The National Government or the next higher level of local
(v) Social welfare services which include programs and projects government unit may provide or augment the basic services
on rebel returnees and evacuees; relief operations; and and facilities assigned to a lower level of local government unit
population development services; when such services or facilities are not made available or, if
(vi) Provincial buildings, provincial jails, freedom parks and other made available, are inadequate to meet the requirements of its
public assembly areas, and similar facilities; inhabitants.
(vii) Infrastructure facilities intended to service the needs of the (g) The basic services and facilities hereinabove enumerated shall
residents of the province and which are funded out of be funded from the share of local government units in the
provincial funds including, but not limited to, provincial proceeds of national taxes and other local revenues and
roads and bridges; inter-municipal waterworks, drainage funding support from the National Government, its
and sewerage, flood control, and irrigation systems; instrumentalities and government-owned or controlled
reclamation projects; and similar facilities; corporations which are tasked by law to establish and maintain
(viii) Programs and projects for low-cost housing and other mass such services or facilities. Any fund or resource available for the
dwellings, except those funded by the Social Security System use of local government units shall be first allocated for the
(SSS), Government Service Insurance System (GSIS), and the provision of basic services or facilities enumerated in subsection
Home Development Mutual Fund (HDMF); Provided, That (b) hereof before applying the same for other purposes, unless
national funds for these programs and projects shall be otherwise provided in this Code.
equitably allocated among the regions in proportion to the (h) Regional offices of national agencies or offices whose functions
ratio of the homeless to the population; are devolved to local government units as provided herein shall
(ix) Investment support services, including access to credit be phased out within one (1) year from the approval of this
Code. Said national agencies and offices may establish such
financing;
field units as may be necessary for monitoring purposes and
(x) Upgrading and modernization of tax information and
providing technical assistance to local government units. The
collection services through the use of computer hardware
properties, equipment, and other assets of these regional
and software and other means;
offices shall be distributed to the local government units in the
(xi) Inter-municipal telecommunications services, subject to
region in accordance with the rules and regulations issued by
national policy guidelines; and
the Oversight Committee created under this Code.
(xii) Tourism development and promotion programs; (i) The devolution contemplated in this Code shall include the
transfer to local government units of the records, equipment, and
(4) For a City: All the services and facilities of the municipality and other assets and personnel of national agencies and offices
province, and in addition thereto, the following: corresponding to the devolved powers, functions, and
(i) Adequate communication and transportation facilities; responsibilities.
(ii) (ii) Support for education, police and fire services and Personnel of said national agencies or offices shall be
facilities; absorbed by the local government units to which they belong
or in whose areas they are assigned to the extent that it is

Bautista, Gastanes, Lopez, Sulit Page | 10


C2017 / LAW 154 / Local Government Law / Midterm Reviewer
administratively viable as determined by the said oversight (d) Intra-municipal telecom services
committee: Provided, That the rights accorded to such
personnel pursuant to civil service law, rules and regulations SECTION 18. Power to Generate and Apply Resources.
shall not be impaired: Provided, further, That regional directors Local government units shall have the power and authority to:
who are career executive service officers and other officers of (a) establish an organization that shall be responsible for the
similar rank in the said regional offices who cannot be absorbed efficient and effective implementation of their development
by the local government unit shall be retained by the National plans, program objectives and priorities;
Government, without any diminution of rank, salary or tenure. (b) to create their own sources of revenues and to levy taxes, fees,
(j) To ensure the active participation of the private sector in local and charges which shall accrue exclusively for their use and
governance, local government units may, by ordinance, sell, disposition and which shall be retained by them;
lease, encumber, or otherwise dispose of public economic (c) to have a just share in national taxes which shall be
enterprises owned by them in their proprietary capacity. automatically and directly released to them without need of
Costs may also be charged for the delivery of basic services any further action;
or facilities enumerated in this section. (d) to have an equitable share in the proceeds from the utilization
and development of the national wealth and resources within
Notes their respective territorial jurisdictions including sharing the
 Sec 17, LGC is the core of DECENTRALIZATION and same with the inhabitants by way of direct benefits;
DEVOLUTION (e) to acquire, develop, lease, encumber, alienate, or otherwise
 Thrusts of LGUs: dispose of real or personal property held by them in their
(a) Discharge functions and responsibilities of central agencies proprietary capacity and to apply their resources and assets for
and offices that are now devolved to them productive, developmental, or welfare purposes, in the exercise
(b) Strive for self-reliance or furtherance of their governmental or proprietary powers and
(c) Continue exercising the powers and discharging the duties functions and thereby ensure their development into self-
that are currently vested upon them reliant communities and active participants in the attainment
(d) Exercise such other powers and discharge such other of national goals.
functions and responsibilities as are necessary, appropriate,
or incidental to efficient and effective provision of the basic SECTION 19. Eminent Domain. A local government unit may, through
services and facilities enumerated its chief executive and acting pursuant to an ordinance, exercise the
(e) This list is NOT exclusive power of eminent domain for public use, or purpose or welfare for
the benefit of the poor and the landless, upon payment of just
THE LGU’S POWERS IN GENERAL compensation, pursuant to the provisions of the Constitution and
pertinent laws: Provided, however, That the power of eminent
Barangay domain may not be exercised unless a valid and definite offer has
 One power which exclusively belongs to this LGU and it is the been previously made to the owner, and such offer was not
power to ADMINISTER THE KATARUNGANG PAMBARANAGAY accepted: Provided, further, That the local government unit may
immediately take possession of the property upon the filing of the
Municipality expropriation proceedings and upon making a deposit with the
 Involves the delivery of 4 major services: proper court of at least fifteen percent (15%) of the fair market value
o Agriculture and Aquaculture of the property based on the current tax declaration of the property
o Health Services to be expropriated: Provided, finally, That, the amount to be paid for
o Social Services the expropriated property shall be determined by the proper court,
o Other services including information dissemination on based on the fair market value at the time of the taking of the
investment needs and job placements property.
City
Refer to the portion of Eminent Domain
 Of all the LGUs, the city has been granted the MOST powers
 LGC devolved to it ALL the services and facilities which the brgy,
SECTION 20. Reclassification of Lands.
muni, prov, had been empowered to deliver
(a) A city or municipality may, through an ordinance passed by the
 EXCEPT power to administer the katarungang pambragy
sanggunian
 On top of these the city is expressly empowered to provide: - AFTER conducting public hearings for the purpose, authorize the
(a) Adequate communication and transportation facilities reclassification of agricultural lands and provide for the manner of
(b) Suport for education, police, and fire services and facilities their utilization or disposition in the following cases:
(1) when the land ceases to be economically feasible and sound for
Province agricultural purposes as determined by the Department of
 Power to deliver varied services such as: Agriculture or
(a) Industrial development services (2) where the land shall have substantially greater economic value
(b) Health care and tertiary healthcare for residential, commercial, or industrial purposes, as
(c) Computerizing tax information and collection services determined by the sanggunian concerned:

Bautista, Gastanes, Lopez, Sulit Page | 11


C2017 / LAW 154 / Local Government Law / Midterm Reviewer
(3) Provided, That such reclassification shall be limited to the (c) Any national or local road, alley, park, or square may be
following percentage of the total agricultural land area at the temporarily closed during an actual emergency, or fiesta
time of the passage of the ordinance: celebrations, public rallies, agricultural or industrial fairs, or an
(1) For highly urbanized and independent component cities, undertaking of public works and highways,
fifteen percent (15%); telecommunications, and waterworks projects, the duration of
(2) For component cities and first to the third class which shall be specified by the local chief executive concerned
municipalities, ten percent (10%); and in a written order: Provided, however, That no national or local
(3) For fourth to sixth class municipalities, five percent (5%): road, alley, park, or square shall be temporarily closed for
(4) Provided, further, That agricultural lands distributed to athletic, cultural, or civic activities not officially sponsored,
agrarian reform beneficiaries pursuant to R.A. No.6657 recognized, or approved by the local government unit
aka "The Comprehensive Agrarian Reform Law", shall not concerned.
be affected by the said reclassification and the conversion (d) Any city, municipality, or barangay may, by a duly enacted
of such lands into other purposes shall be governed by ordinance, temporarily close and regulate the use of any local
Section 65 of said Act. street, road, thoroughfare, or any other public place where
(b) The President may authorize a city or municipality to re-classify shopping malls, Sunday, flea or night markets, or shopping
lands in excess of the limits set in the next preceding paragraph: areas may be established and where goods, merchandise,
(1) when public interest so requires and foodstuffs, commodities, or articles of commerce may be sold
(2) upon recommendation of the National Economic and and dispensed to the general public.
Development Authority,
(c) The local government units shall, in conformity with existing Please refer to the portion of Closure and Opening of Roads
laws, continue to prepare their respective comprehensive land
use plans enacted through zoning ordinances which shall be SECTION 22. Corporate Powers.
the (a) Every local government unit, as a corporation, shall have the
(1) primary and dominant bases for the future use of land following powers:
resources: (1) To have continuous succession in its corporate name;
(2) Provided, That the requirements for food production, (2) To sue and be sued;
human settlements, and industrial expansion shall be (3) To have and use a corporate seal;
taken into consideration in the preparation of such plans. (4) To acquire and convey real or personal property;
(d) Where approval by a national agency is required for (5) To enter into contracts; and
reclassification such approval shall: (6) To exercise such other powers as are granted to
(1) not be unreasonably withheld corporations, subject to the limitations provided in this
(2) Failure to act on a proper and complete application for Code and other laws.
reclassification within three (3) months from receipt of the (b) Local government units may continue using, modify, or change
same shall be deemed as approval thereof. their existing corporate seals Provided, That:
(e) Nothing in this Section shall be construed as repealing, (1) newly established local government units or those without
amending, or modifying in any manner the provisions of R.A. corporate seals may create their own corporate seals
No. 6657. which shall be registered with the Department of the
Interior and Local Government:
This provision is NOT to convert land for any purpose contrary to CARL (2) any change of corporate seal shall also be registered as
but merely to reclassify land provided hereon.
(c) No contract may be entered into by the local chief executive in
SECTION 21. Closure and Opening of Roads. behalf of the local government unit without prior authorization
(a) A local government unit may, pursuant to an ordinance, by the sanggunian concerned UNLESS:
permanently or temporarily close or open any local road, alley, (1) provided otherwise in the LGC
park, or square falling within its jurisdiction: Provided, however, (2) A legible copy of such contract shall be posted at a
That in case of permanent closure, such ordinance must be conspicuous place in the provincial capital or the city,
approved by at least two-thirds (2/3) of all the members of the municipal or barangay hall.
sanggunian, and when necessary, an adequate substitute for (d) Local government units shall enjoy full autonomy in the
the public facility that is subject to closure is provided. exercise of their proprietary functions and in the management
(b) No such way or place or any part thereof shall be permanently of their economic enterprises, subject to the limitations
closed without making provisions for the maintenance of public provided in this Code and other applicable laws.
safety therein. A property thus permanently withdrawn from
public use may be used or conveyed for any purpose for which Notes
other real property belonging to the local government unit  There is no hard and fast rule for the purposes of determining the
concerned may be lawfully used or conveyed: Provided, true nature of an undertaking or function of an LGU; the
however, That no freedom park shall be closed permanently surrounding circumstances are to be considered:
without provision for its transfer or relocation to a new site.

Bautista, Gastanes, Lopez, Sulit Page | 12


C2017 / LAW 154 / Local Government Law / Midterm Reviewer
 The basic element, however, beneficial to the public the o When such national agency fails to act on the request for
undertaking maybe, is that it is governmental in essence otherwise approval within thirty (30) days from receipt thereof, the
it is proprietary like in City of MNL v CA. same shall be deemed approved.
 The local chief executive shall, within thirty (30) days upon
SECTION 23. Authority to Negotiate and Secure Grants. Local chief signing of such grant agreement or deed of donation, report the
executives may negotiate and secure financial grants or donations nature, amount, and terms of such assistance to both Houses of
in kind: Congress and the President.
 FOR: support of the basic services or facilities enumerated under
Section 17 hereof, SECTION 24. Liability for Damages. Local government units and their
 FROM: from local and foreign assistance agencies without officials are not exempt from liability for death or injury to persons
necessity of securing clearance or approval therefor from any or damage to property.
department, agency, or office of the National Government or from
any higher local government unit (upon authority of the Notes
Sanggunian)  Properties NOT subject to levy and execution of necessary for
 Provided, That projects financed by such grants or assistance with Public Use cannot be attached and sold at an execution sale to
national security implications shall be approved by the national satisfy money claims
agency concerned

Bautista, Gastanes, Lopez, Sulit Page | 13


C2017 / LAW 154 / Local Government Law / Midterm Reviewer

III. GENERAL PRINCIPLES AND POLICIES

Provisions Creation of Special Metropolitan Political Subdivision


 Note: Do not mistake the “special metropolitan political
subdivisions” in Sec 11 as another territorial political subdivision
1987 Constitution
in Sec 1
 Jurisdiction of the metropolitan authority is LIMITED to basic
Art. X, Sec. 1
services requiring coordination (as in BAVA v MMDA and MMDA
The territorial and political subdivisions of the Republic of the
v GARIN)
Philippines are the provinces, cities, municipalities, and barangays.
There shall be autonomous regions in Muslim Mindanao and the
Art. X, Sec. 12
Cordilleras as hereinafter provided.
Cities that are highly urbanized, as determined by law, and
component cities whose charters prohibit their voters from voting for
The LGUs
provincial elective officials, shall be independent of the province.
 The republic is divided into territorial and political subdivisions
The voters of component cities within a province, whole charters
called the LGUs:
contain no such prohibition, shall not be deprived of their right to
(1) Provinces
vote for elective provincial officials.
(2) Cities
(3) Municipalities
Classification of Cities
(4) Barangays
 LGC classifies cities into:
(1) Highly Urbanized
The Autonomous Regions
(2) Independent Component
 In addition to the 4 types of subdivisions, the Constitution
(3) Component Cities
mandates the creation of 2 autonomous regions:
 They are classified based on their regular income
(1) One for Muslim Mindanao (won in plebiscite)
(2) One Cordilleras  It is indicative of its capability to develop as relatively
independent social, economic, and political unit
Nature of LGUs
 It is a political subdivision of the State which is constituted by law Local Government Code (RA 7160)
and possessed over substantial control over its own affairs.
 An intra-sovereign subdivision of one sovereign nation Sec. 1. Title. This Act shall be known and cited as the "Local
 Not intended to be an imperium in imperio Government Code of 1991".
 Autonomous in a sense that it is given more powers, authority,
Sec. 2. Declaration of Policy.
responsibilities and resources
(a) It is hereby declared the policy of the State that the territorial
and political subdivisions of the State shall enjoy genuine and
Art. X, Sec. 3
meaningful local autonomy to enable them to attain their
The Congress shall enact a local government code which shall
fullest development as self-reliant communities and make
provide for a more responsive and accountable local government
them more effective partners in the attainment of national
structure instituted through a system of decentralization with
goals. Toward this end, the State shall provide for a more
effective mechanisms of recall, initiative, and referendum, allocate
responsive and accountable local government structure
among the different local government units their powers,
instituted through a system of decentralization whereby local
responsibilities, and resources, and provide for the qualifications,
government units shall be given more powers, authority,
election, appointment and removal, term, salaries, powers and
responsibilities, and resources. The process of decentralization
functions and duties of local officials and all other matters relating
shall proceed from the National Government to the local
to the organization and operation of local units.
government units.
(b) It is also the policy of the State to ensure the accountability of
Art. X, Sec. 11
local government units through the institution of effective
The Congress may, by law, create special metropolitan political
mechanisms of recall, initiative and referendum.
subdivisions, subject to a plebiscite as set forth in Sec 29 hereof. The
(c) It is likewise the policy of the State to require all national
component cities and municipalities shall retain their basic
agencies and offices to conduct periodic consultations with
autonomy and shall be entitled to their own local executives and
appropriate local government units, non-governmental and
legislative assemblies. The jurisdiction of the metropolitan authority
people’s organizations, and other concerned sectors of the
that will hereby be created shall be limited to basic services
community before any project or program is implemented in
requiring coordination.
their respective jurisdictions.

Bautista, Gastanes, Lopez, Sulit Page | 14


C2017 / LAW 154 / Local Government Law / Midterm Reviewer
Sec. 3. Operative Principles of Decentralization. The formulation and (m) The national government shall ensure that decentralization
implementation of policies and measures on local autonomy shall contributes to the continuing improvement of the performance
be guided by the following operative principles: of local government units and the quality of community life.
(a) There shall be an effective allocation among the different local
government units of their respective powers, functions, Sec. 4. Scope of Application. This Code shall apply to all provinces,
responsibilities, and resources; cities, municipalities, barangays, and other political subdivisions as
(b) There shall be established in every local government unit an may be created by law, and, to the extent herein provided, to
accountable, efficient, and dynamic organizational structure officials, offices, or agencies of the national government.
and operating mechanism that will meet the priority needs and
service requirements of its communities; Sec. 5. Rules of Interpretation. In the interpretation of the provisions
(c) Subject to civil service law, rules and regulations, local officials of this Code, the following rules shall apply:
and employees paid wholly or mainly from local funds shall be (a) Any provision on a power of a local government unit shall be
appointed or removed, according to merit and fitness, by the liberally interpreted in its favor, and in case of doubt, any
appropriate appointing authority; question thereon shall be resolved in favor of devolution of
(d) The vesting of duty, responsibility, and accountability in local powers and of the lower local government unit. Any fair and
government units shall be accompanied with provision for reasonable doubt as to the existence of the power shall be
reasonably adequate resources to discharge their powers and interpreted in favor of the local government unit concerned;
effectively carry out their functions; hence, they shall have the (b) In case of doubt, any tax ordinance or revenue measure shall be
power to create and broaden their own sources of revenue and construed strictly against the local government unit enacting it,
the right to a just share in national taxes and an equitable share and liberally in favor of the taxpayer. Any tax exemption,
in the proceeds of the utilization and development of the incentive or relief granted by any local government unit
national wealth within their respective areas; pursuant to the provisions of this Code shall be construed
(e) Provinces with respect to component cities and municipalities, strictly against the person claiming it.
and cities and municipalities with respect to component (c) The general welfare provisions in this Code shall be liberally
barangays, shall ensure that the acts of their component units interpreted to give more powers to local government units in
accelerating economic development and upgrading the quality
are within the scope of their prescribed powers and functions;
(f) Local government units may group themselves, consolidate or of life for the people in the community;
coordinate their efforts, services, and resources for purposes (d) Rights and obligations existing on the date of effectivity of this
Code and arising out of contracts or any other source of
commonly beneficial to them; (g) The capabilities of local
prestation involving a local government unit shall be governed
government units, especially the municipalities and barangays,
by the original terms and conditions of said contracts or the law
shall be enhanced by providing them with opportunities to
participate actively in the implementation of national programs in force at the time such rights were vested; and
(e) In the resolution of controversies arising under this Code where
and projects;
no legal provision or jurisprudence applies, resort may be had
(g) (missing?)
to the customs and traditions in the place where the
(h) There shall be a continuing mechanism to enhance local
controversies take place.
autonomy not only by legislative enabling acts but also by
administrative and organizational reforms;
(i) Local government units shall share with the national A. Local Autonomy
government the responsibility in the management and
maintenance of ecological balance within their territorial Provisions
jurisdiction, subject to the provisions of this Code and national
policies;
1987 Constitution
(j) Effective mechanisms for ensuring the accountability of local
government units to their respective constituents shall be
Art. II, Sec. 25
strengthened in order to upgrade continually the quality of
The State shall ensure the autonomy of local governments.
local leadership;
(k) The realization of local autonomy shall be facilitated through
Art. X, Sec. 2
improved coordination of national government policies and The territorial and political subdivisions shall enjoy local autonomy,
programs and extension of adequate technical and material
assistance to less developed and deserving local government
units; Art. X, Sec. 4
(l) The participation of the private sector in local governance, The president of the Philippine shall exercise general supervision
particularly in the delivery of basic services, shall be over local governments. Provinces with respect to component cities
encouraged to ensure the viability of local autonomy as an and municipalities, and cities and municipalities with respect to
alternative strategy for sustainable development; and component barangays shall ensure that the acts of their component
units are within the scope of their prescribed powers and functions.

Bautista, Gastanes, Lopez, Sulit Page | 15


C2017 / LAW 154 / Local Government Law / Midterm Reviewer
Notes: Held: The Supreme Court upheld Sec. 1 and reasoned that it was
Presidential Power of Supervision NOT control merely advisory. As for Sec. 4, it was struck down because it was in
 This means he exercises general supervision but only to ensure contravention of the automatic release of the IRA to LGUs. Local
that local affairs are administered according to law. autonomy signified a more responsive and accountable local
 Supervision means overseeing or the authority of an officer to see government structure instituted through a system of
that the subordinate officers perform their duties. Control means to decentralization. Decentralization simply means the devolution of
the power to amend, correct, or substitute subordinate officer’s national administration, not power, to local governments. Local
judgment. officials remain accountable to the central government as the law
may provide. Under the Philippine concept of local autonomy, the
Art. X, Sec. 11 national government has not completely relinquished all its powers
The Congress may, by law, create special metropolitan political over local governments, including autonomous regions. Only
subdivisions, subject to a plebiscite as set forth in Sec 29 hereof. The administrative powers over local affairs are delegated to political
component cities and municipalities shall retain their basic subdivisions. The purpose of the delegation is to make governance
autonomy and shall be entitled to their own local executives and more directly responsive and effective at the local levels.
legislative assemblies. The jurisdiction of the metropolitan authority
that will hereby be created shall be limited to basic services Ganzon v. CA
requiring coordination. Facts: 3 preventive suspension orders were issued by the Secretary
of Interior Local Government against Mayor Ganzon, in relation to
(see earlier explanation) various administrative complaints filed against him. Ganzon
questioned the power of the SILG, as the President's alter-ego, to
Art. X, Sec. 12 suspend officials, arguing that under the 1987 Constitution, the
Cities that are highly urbanized, as determined by law, and President does not have the power to suspend or remove local
component cities whose charters prohibit their voters from voting for officials.
provincial elective officials, shall be independent of the province. Held: Court held that the President possesses such power. The
The voters of component cities within a province, whole charters change in constitutional language (with respect to the supervision
contain no such prohibition, shall not be deprived of their right to clause) was meant but to deny legislative control over local
vote for elective provincial officials. governments; it did not exempt the latter from legislative
regulations provided regulation is consistent with the fundamental
(see earlier explanation) premise of autonomy. Since local governments remain accountable
to the national authority, the latter may, by law, and in the manner
San Juan v. CSC set forth therein, impose disciplinary action against local officials.
Facts: The position of PBO of Rizal province became vacant. Gov.
San Juan, pursuant to EO 112, recommended Santos to the DBM as CAB v. CA
new PBO (but there were also other recommendees/ nominees Facts: Constitutionality of EO No. 220 is being questioned here,
submitted by him). DBM however, appointed Almajose who was not which created the CAR. This EO’s main function is to coordinate the
among those nominated averring that the governor’s nominees are planning and implementation of programs and services in the
only directory and not mandatory; and cited DBM Local Budget Circ. region.
31 which reserves them the right to appoint one not nominated by
the governor. Gov. San Juan then protested with DBM and CSC but Held: EO No. 220 does not create the autonomous region
was denied. contemplated in the Constitution, but merely provides for transitory
measures. EO No. 220 only created a region, covering a specified
Held: The SC ruled that EO 112 must be interpreted in favor of local area, for administrative purposes with the objective of coordinating
autonomy. Thus, DBM may only appoint from the governor’s the planning and implementation of programs and services. The
nominees. The appointment of Almajose was void, and the Local constitutional guarantee of local autonomy refers to the
Budget Circ 31 was struck down for being ultra vires. Where a law is administrative autonomy of LGUs or the decentralization of
capable of two interpretations, one in favor of centralized power and government authority. The creation of autonomous regions
the other beneficial to local autonomy, the scales must be weighed contemplates the grant of political autonomy and not just
in favor of autonomy. administrative autonomy to these regions. The CAR is a mere
transitory coordinating agency that would prepare the stage for
Pimentel v. Aguirre political autonomy for the Cordilleras.
Facts: AO 372, later amended by AO 43, was issued which reduced
total expenditures for the year by at least 25% of authorized regular SEMA v. COMELEC
appropriations for non-personal services items for government Facts: Four days before the May 2007 elections, COMELEC
departments and agencies. (Sec. 1) and the amount equivalent to promulgated Resolution No. 7902, whereby it resolved to maintain
10% of the internal revenue allotment to local government units the composition of what had been the First District of Maguindanao,
shall be withheld (Sec. 4). composed of Cotabato City (a chartered city) and other
municipalities, even though these municipalities formerly belonging

Bautista, Gastanes, Lopez, Sulit Page | 16


C2017 / LAW 154 / Local Government Law / Midterm Reviewer
to Maguindanao have been constituted as part of the province of Limbona v Mangellin
Shariff Kabunsuan (SK). The Province of SK was created by the Facts: Petitioner was ousted from his position as Speaker of the
Regional Assembly by virtue of Muslim Mindanao Autonomy Act No. Regional Legislative ASSEMBLY of Central Mindanao, with the
201 in Aug 2006. Sema, a candidate, sought to nullify Resolution other members voting behind his back as well as expelling him from
7902 since it usurped Congress’ power to create and reapportion membership in said Assembly. In this case, the SC discusses the
legislative districts. propriety of its intervention in this issue and the nature of the
autonomous governments of Mindanao, and whether they are
Held: SC upheld the validity of Resolution 7902 since the creation of subject to the jurisdiction of the national courts.
the province of SK (MMA Act 201) is void. Only an act of Congress
can create a province, according to the LGC. The delegation of Held: PD1619 creating the autonomous government of Mindanao
Congress to ARMM to create provinces and cities is unconstitutional persuades us that they were never meant to exercise autonomy in
because it is a non-delegable power of Congress. the second sense. "[t]he President shall have the power of general
supervision and control over Autonomous Regions." The
Dissenting/ Concurring Opinion of J. Tinga: Nothing in the Sangguniang Pampook is made to discharge chiefly administrative
Constitution bars the Congress from delegating the power to create services.
provinces and cities. But it is subject to the criteria set forth by
Congress. However, Congress cannot delegate the power to increase
the composition of the HoR since the Constitution specifically
Notes:
designates it to Congress.
DECENTRALIZATION:
 Decentralization of administration: the central government
B. Decentralization delegates administrative powers to political subdivisions in order
to broaden the base of government power and in the process to
Provisions make local governments "more responsive and accountable",
and "ensure their fullest development as self-reliant
1987 Constitution communities and make them more effective partners in the
pursuit of national development and social progress." At the
Art. X, Sec. 3 same time, it relieves the central government of the burden of
The Congress shall enact a local government code which shall managing local affairs and enables it to concentrate on national
provide for a more responsive and accountable local government concerns. The President exercises "general supervision" over
them, but only to "ensure that local affairs are administered
structure instituted through a system of decentralization with
effective mechanisms of recall, initiative, and referendum, allocate according to law." He has no control over their acts in the sense
that he can substitute their judgments with his own.
among the different local government units their powers,
responsibilities, and resources, and provide for the qualifications,  Decentralization of power: abdication of political power in favor
election, appointment and removal, term, salaries, powers and of local government units declared to be autonomous. In that
functions and duties of local officials and all other matters relating case, the autonomous government is free to chart its own destiny
to the organization and operation of local units. and shape its future with minimum intervention from central
authorities. According to a constitutional author,
Art. X, Sec. 14 decentralization of power amounts to "self-immolation", since in
The President shall provide for regional development councils or that event, the autonomous government becomes accountable
other similar bodies composed of local government officials, not to the central authorities but to its constituency.
regional heads of departments and other government offices, and
representatives from non-governmental organizations within the AUTONOMY
region for purposes of administrative decentralization to strengthen  Autonomy of LGU’s in TWO senses: An autonomous government
the autonomy of the units therein and to accelerate the economic that enjoys autonomy of the latter category is subject alone to the
and social growth and development of the units in the region. decree of the organic act creating it and accepted principles on
the effects and limits of "autonomy". On the other hand, an
Notes: autonomous government of the former class is under the
LGUs Relationship with Regional Development Councils supervision of the national government acting through the
 LGUs relate to the Regional Development Council in matters that President.
concern their own development.

Bautista, Gastanes, Lopez, Sulit Page | 17


C2017 / LAW 154 / Local Government Law / Midterm Reviewer

IV. CREATION OF MUNICIPAL CORPORATIONS

A. NATURE AND POWES TO CREATE MUNICIPAL  These subdivisions may be created, divided, merged, abolished,
or its boundary substantially altered:
CORPORATION
o in accordance with the criteria established in the Local
Government Code
PELAEZ v. AUDITOR GENERAL o subject to approval by a majority of the votes cast in a
Facts: President Diosdado Macapagal issued several Executive plebiscite in the political units directly affected. (Sec. 10).
Orders creating 33 municipalities in total. He purportedly relied  Congress may create special metropolitan political subdivisions
upon Sec. 68 of the Revised Administrative Code, as basis of by law, subject to a plebiscite.
authority to create municipal corporations. VP Pelaez sought to o Component cities/municipalities shall retain their basic
restrain Auditor General from passing in audit/disbursing money in autonomy and shall be entitled to their own local executives
implementation of the EO’s. and legislative assemblies.
o The jurisdiction of the metropolitan authority shall be limited
Held: Court ruled in favor of VP and enjoined the Auditor General to basic services requiring coordination (Sec. 11).
from doing the acts mentioned. Under RA 2370 (Barrio Charter Act),  Autonomous regions in Muslim Mindanao and in the Cordilleras
barrios may not be created or their boundaries altered, or names shall consist of provinces, cities, municipalities, and geographical
changed except by an act of Congress, or the corresponding areas sharing common and distinctive historical and cultural
provincial board upon petition of a majority of voters and heritage, economic and social structures, and other relevant
recommendation by municipal council. If barrios may not be created characteristics within the framework of this Constitution and the
by President, then there is more reason to deny him of the bigger national sovereignty, as well as territorial integrity of the
power to create municipalities, which are groups of several barrios. Republic (Sec. 11).
The constitution merely confers upon the President the power of o Congress shall enact an organic act for each autonomous
supervision over local government units, as distinguished from the region with the assistance and participation of the regional
presidential grant of power of control over executive departments, consultative commission composed of representatives
bureaus and offices. If Sec 68 were valid, he could in effect remove appointed by the President from a list of nominees from
any of its officials by creating a new municipality and including multisectoral bodies. The organic act shall:
therein the barrio where the official resides, for his office would  define the basic structure of government for the region
become vacant. consisting of the executive department and legislative
assembly, both of which shall be elective and
Municipality of Kapalong v. Moya representative of the constituent political units, and
Facts: There was a boundary dispute between the Municipality of  shall provide for special courts with personal, family, and
Kapalong and the Municipality of Sto. Tomas (the latter a creation property law jurisdiction consistent with the provisions of
of President CP Garcia using portions of Kapalong). Sto. Tomas filed this Constitution and national laws.
an action with the CFI for the settlement of the dispute. Kapalong o Creation of the autonomous regions shall be effective when
filed an MTD, alleging that Sto. Tomas lacked legal personality. approved by majority of the votes cast by the constituent units
in a plebiscite called for the purpose. However, those
Held: CFI case should have been dismissed. It was held in Pelaez v. geographic areas voting favorably in such plebiscite shall be
Auditor General that the President has no power to create included in the autonomous region (Sec. 18).
municipalities. Thus, the Municipality of Sto. Tomas, as a creation of  The first Congress elected under the 1987 Constitution shall,
the President, had no legal personality to file a suit. Motion to within 18 months from the time of organization of both Houses,
dismiss granted. pass the organic acts for the autonomous regions provided herein
(Sec 19).
B. CREATION OF MUNICIPAL COPORATIONS
NAVARRO v. ERMITA
Facts: The Province of Dinagat Islands was created by RA 9355.
1. Constitutional Provisions Petitioners sought to have RA 9355 declared unconstitutional for its
failure to meet either the land or the population requirement in the
SUMMARY OF PERTINENT CONSTITUTIONAL PROVISIONS LGC as required by Art X of the 1987 Constitution.
(ARTICLE X)
 The territorial and political subdivisions of the Philippines are the Held (APR 2010): SC ruled that the law is unconstitutional because
provinces, cities, municipalities, and barangays. There shall be it did not meet the standards set by the LGC as to land area (only
autonomous regions in Muslim Mindanao and the Cordilleras. 802.12 sq. km out of the required 2,000 sq km) or population (only
(Sec. 1). 106 951 inhabitants of the required 250 000). If a proposed province

Bautista, Gastanes, Lopez, Sulit Page | 18


C2017 / LAW 154 / Local Government Law / Midterm Reviewer
is composed of 2 or more islands, the “territory under the LGC 2. Statutory Provisions
includes only the land mass above the water.
Held (MAY 2010): SC upheld the above ruling. this time adding that SUMMARY OF PERTINENT LGC PROVISIONS
the operative fact doctrine does not apply. The Court, while  An LGU may be created, divided, merged, abolished, or its
respecting the doctrine of separation of powers, cannot renege on boundaries substantially altered:
its duty to determine whether the other branches of the government o Province, city, municipality, or any other political subdivision:
have kept themselves within the limits of the Constitution, and by law enacted by Congress
determine whether illegality attached to the creation of the province o Barangay: by law enacted by Congress or by ordinance
in question. To abandon this duty only because the Province of passed by the Sangguniang Panlalawigan or Sangguniang
Dinagat Islands has began its existence is to consent to the passage Panlungsod located within its territorial jurisdiction (Sec 6).
of a law that is violative of the provisions of the Constitution and the o IN ALL CASES, subject to approval by a majority of the votes
Local Government Code, rendering the law and the province created cast in a plebiscite to be conducted by the COMELEC in the
null and void. LGUs directly affected.
 Verifiable indicators of viability and projected capacity to provide
Held (2011): SC reverses itself. RA 9355 is constitutional. In the services (Sec. 7):
creation of municipalities, component cities, and provinces, the 3 o Income (attested to by DOF): sufficient to provide for all
indicators of viability and projected capacity to provide services, i.e., essential government facilities and services and special
income, population, and land area, are provided for. However, when functions commensurate with the size of its population, as
the LGU to be created consists of one or more islands, it is expressly expected of the LGU concerned;
exempt from the land area requirement as expressly provided in o Population (attested to by NSO):: the total number of
Secs. 442 and 450 of the LGC if the LGU to be created is a inhabitants within the territorial jurisdiction of the LGU
municipality or a component city, respectively. This exemption is concerned; and
absent in the enumeration of the requisites for the creation of a o Land Area (attested to by DENR-Lands Management
province under Sec. 461, although it is expressly stated under Art. Bureau): contiguous (unless it comprises two or more islands
9(2) of the IRR. Thus, the LGC provides an exemption from territorial or is separated by an independent LGU), properly identified
contiguity when the intended province consists of two or more by metes and bounds with technical descriptions, and
islands. This includes the exemption from the application of the sufficient to provide for basic services and facilities to meet
minimum land area requirement, as embodied in the LGC’s IRR. the requirements of its populace.

REQUISITES FOR CREATION OF LGUS


BARANGAY MUNICIPALITY CITY PROVINCE
Not less than 2,000 Not less than 25,000 Not less than 150,000 Not less than 250,000
inhabitants inhabitants inhabitants inhabitants
(5,000 for cities and
municipalities within (OR) (OR)
POPULATION
MM and other
metropolitan
subdivisions or highly
urbanized cities)
LAND AREA At least 50 sq km At least 100 sq km At least 2000 sq km
ANNUAL INCOME At least P2,500,000 At least P100,000,000 At least P20,000,000

NOTES: LEAGUE OF CITIES v. COMELEC


 In the case of the creation of barangays by the Sangguniang Facts: The 11th Congreds enacted into law 33 bills converting 33
Panlalawigan, the recommendation of the Sangguniang Bayan municipalities into cities. However, Congress did not act on bills
concerned shall be necessary. converting 24 other municipalities into cities. The 12th Congress
 Creation of new LGUs shall not reduce the population of the then enacted into law RA 9009, which amended the LGC by
original LGUs to less than the minimum requirement prescribed increasing the annual income requirement for conversion of a
in the LGC. municipality into a city from P20 million to P100 million, in order to
 The territorial jurisdiction of a newly-created LGU shall be prevent “the mad rush” of municipalities to convert into cities solely
properly identified by metes and bounds. to secure a larger share in the IRA despite the fact that they are
 The average annual income shall include the income accruing to incapable of fiscal independence.
the general fund of the LGU concerned, exclusive of special funds, Plaintiffs now assailed the validity of 16 cityhood laws which
transfers and non-recurring income. exempted certain municipalities from the new P100k income
requirement imposed by RA 9009.

Bautista, Gastanes, Lopez, Sulit Page | 19


C2017 / LAW 154 / Local Government Law / Midterm Reviewer
Held (2008): The Cityhood bills are unconstitutional. First, applying SAMSON v. AGUIRRE
the P100 million income requirement in RA 9009 to the present case Facts: Samson challenges the constitutionality of RA 8535, creating
is a prospective, not a retroactive application, because RA 9009 took the City of Novaliches. He alleges that no certifications as required
effect in 2001 while the cityhood bills became law more than five by the LGC were submitted.
years later. Second, the Constitution requires that Congress shall
prescribe all the criteria for the creation of a city in the Local Held: RA8535 is constitutional. It met all the requirements. The
Government Code and not in any other law, including the Cityhood Court found that present during the public hearings held by the
Laws. Third, the Cityhood Laws violate Section 6, Article X of the Senate Committee on Local Government were resource persons
Constitution because they prevent a fair and just distribution of the from the different government offices like National Statistics Office,
national taxes to local government units. Fourth, the criteria Bureau of Local Government Finance, Land Management Bureau,
prescribed in Section 450 of the Local Government Code, as and Department of Budget and Management, aside from officials of
amended by RA 9009, for converting a municipality into a city are Quezon City itself. Although only oral manifestations were made the
clear, plain and unambiguous, needing no resort to any statutory representatives were present along with other officers armed with
construction. Fifth, the intent of members of the 11th Congress to official statistics and reference materials. In their official capacity,
exempt certain municipalities from the coverage of RA 9009 they spoke and shed light on population, land area and income of
remained an intent and was never written into Section 450 of the the proposed city. Thus, their official statements could serve the
Local Government Code. Sixth, the deliberations of the 11th or 12th same purpose contemplated by law requiring certificates. In this
Congress on unapproved bills or resolutions are not extrinsic aids in case, the Court also stated that under the Local Government Code,
interpreting a law passed in the 13th Congress. Seventh, even if the the proposed city must comply with requirements as regards income
exemption in the Cityhood Laws were written in Section 450 of the and population or land area. Compliance with either requirement, in
Local Government Code, the exemption would still be addition to income, is sufficient.
unconstitutional for violation of the equal protection clause.
Held (2009): The laws are not unconstitutional. Based on the Drilon-
Pimentel exchange during deliberations, Congress did not intend to
3. REVISED ADMIN CODE, SEC 68.
give RA 9009 retroactive effect. Moreover, the doctrine of operative
fact applies. The existence of the cities consequent to the approval General authority of (Governor-General) President of the Philippines to
of the creating, but challenged, cityhood laws in the plebiscites held fix boundaries and make new subdivisions. – The (Governor-General)
in the affected LGUs is now an operative fact. New cities appear to President of the Philippines may by executive order define the
have been organized and are functioning accordingly, with new sets boundary, or boundaries, of any province, subprovince,
of officials and employees. municipality,[township] municipal district, or other political
Held (2010): The laws are unconstitutional. The Constitution subdivision, and increase or diminish the territory comprised therein,
mandates that LGUs shall be created by Congress in accordance may divide any province into one or more subprovinces, separate any
with the criteria provided in the LGC. The exemptions granted to the political division other than a province, into such portions as may be
16 municipalities are contained in separate laws and not in the LGC. required, merge any of such subdivisions or portions with another,
Meanwhile, RA 9009 which increased the income threshold to name any new subdivision so created, and may change the seat of
P100M amended the LGC, which makes it part of LGC. Hence, from government within any subdivision to such place therein as the
the time RA 9009 took effect, all municipalities had to follow this public welfare may require: Provided, That the authorization of the
requirement to be able to convert into a component city. (Philippine Legislature) National Assembly of the Philippines shall
Held (Feb 2011): The laws are not unconstitutional. The Congress first be obtained whenever the boundary of any province or
intended to exempt these cities from the LGC. Since the Cityhood subprovince is to be defined or any province is to be divided into one
Bills are legislative acts, they may be basis to amend the LGC – or more subprovinces. When any action by the (Governor-General)
another legislative act. The Congress saw the wisdom of exempting President of the Philippines in accordance herewith makes
the 16 municipalities from complying with the higher income necessary a change of the territory under the jurisdiction of any
requirement imposed by RA 9009. Further, the Bills do not violate administrative officer or any judicial officer, the (Governor-General)
the equal protection clause since there was valid classification as to President of the Philippines, with the recommendation and advice
the municipalities with pending cityhood bills against those without. of the head of the Department having executive control of such
Since the P100M amount was arbitrarily made, there is no reason to officer, shall redistrict the territory of the several officers affected
prevent these cities pending bills from being enacted into law, and assign such officers to the new districts so formed.
seeing as the purpose of preventing “the mad rush” was not yet Upon the changing of the limits of political divisions in
enacted into law when the Bills were introduced in Congress. They pursuance of the foregoing authority, an equitable distribution of
also used the parable of the laborers in this case. the funds and obligations of the divisions thereby affected shall be
Held (Apr 2011): The laws are not unconstitutional. The newly made in such manner as may be recommended by the (Insular
created cities were intended by the Congress to be exempted from Auditor) Auditor General and approved by the (Governor-General)
the requirement of P100 million income as stated in RA 9009, which President of the Philippines.
amended the LGC.

Bautista, Gastanes, Lopez, Sulit Page | 20


C2017 / LAW 154 / Local Government Law / Midterm Reviewer
MUNICIPALITY OF CANDIJAY (BOHOL) v. CA  As such, it may still be regarded as an existing corporation de
Facts: The Municipalty of Candijay sued the Municipality of Alicia facto. A corporation may be considered at least de facto if the
asserting that Brgy. Pagahat belongs to the former’s territorial following requisites are complied with:
jurisdiction. As one of its grounds, Candijay assailed the juridical o a valid law authorizing incorporation;
personality of Alicia. It contended that E.O. No. 265 issued by o an attempt in good faith to organize under it;
President Quirino in 1949 creating Alicia is null and void ab initio, o a colorable compliance with the law; and
inasmuch as Sec. 68 of the Revised Administrative Code, on which o an assumption of corporate powers.
said E.O. was based, constituted an undue delegation of legislative  Note: According to Ma’am, considering the provision in Section
powers to the President, and was therefore declared 442(d) of the LGC, no de facto municipal corporations or
unconstitutional. municipal corporations by prescription continue to exist.

Held: Citing San Andres v. Mendez, Sr., the SC held that Alicia is a 6. ATTACK AGAINST VALIDITY OF MUNICIPAL
de jure municipality given the governmental acts indicating the
CORPORATIONS
State’s recognition thereof throughout its 16 years of existence prior
to Pelaez and even after A.O. No. 33, Ordinance appended to the  When a corporation is de jure, it is impregnable to any attack,
1987 Constituiton, and Sec. 442(d) of Local Government Code. A direct or otherwise.
municipality created under an Executive Order issued pursuant to  The validity of incorporation and corporate existence of a
RAC Sec 68, though declared void by Pelaez v. Auditor General, may municipal corporation may not be attacked collaterally.
be considered as de jure in view of governmental acts throughout  It may be challenged only by the State in a direct proceeding such
the years indicating the State's recognition and acknowledgment of as quo warranto.
the existence thereof, and also of the curative effect of Sec. 442(d)  However, where the corporation is an absolute nullity, it is subject
of the Local Government Code. to collateral attack.

This parts were taken from the E2016 Reviewer. Thank you, E2016! 7. BEGINNING OF CORPORATE EXISTENCE OF
MUNICIPAL CORPORATIONS
4. MUNICIPAL CORPOATION BY
PRESCRIPTION LGC Sec 14. Beginning of Corporate Existence. − When a new local
government unit is created, its corporate existence shall commence
A municipal corporation may exist by prescription. upon the election and qualification of its chief executive and a
 Its existence will be presumed where it is shown that the majority of the members of its Sanggunian, unless some other time
community has claimed and exercised corporate functions is fixed therefor by the law or ordinance creating it.
without interruption or objection for a period long enough to  The legal existence of a municipal corporation is to be
afford title by prescription. determined by the law creating it.
 Where the law is silent as to be the beginning of its corporate
5. DE FACTO MUNICIPAL CORPORATIONS existence, such shall commence upon the election and
qualification of its chief executive and a majority of the members
of the sanggunian.
 A corporation may exist in fact although not in point of law
because of certain defects in some essential features of its
organization.

Bautista, Gastanes, Lopez, Sulit Page | 21


C2017 / LAW 154 / Local Government Law / Midterm Reviewer

V. Alteration and Dissolution of Municipal Corporations


*Notes adopted from E2016 Reviewer. Thank you, E2016!
A. Nature of Power sangguniang panlalawigans of the provinces involved. Neither can
it exercise its appellate jurisdiction as no petition had been filed and
decided by the said sangguniang panlalawigans.
1. Necessity for Defining Territorial Boundaries
Barangay Sangalang v. Barangay Maguihan (2009)
 A municipal corporation cannot, without legal authorization,
Summary: Case involves a barangay jurisdiction dispute between
exercise its powers beyond its own corporate limits.
Barangay Sangalang and Barangay Maguihan. Case was lodged
 Ergo, it is necessary that it have its boundaries fixed, definite, and before the Sangguniang Bayan (SB), which ruled in favor of
certain, in order that they may be identified and that all may know Sangalang. In the exercise of its appellate jurisdiction, the RTC
the exact scope or section of territory or geographical division reversed the SB Resolution and ruled that the subject lots belonged
embraced within the corporate limits and over which the local to Maguihan. Sangalang filed a Notice of Appeal with the CA, which
corporation has jurisdiction. dismissed the same for being the wrong remedy. Court agreed that
 An incorporation is void where the boundaries of the municipal Sangalang should have filed a petition for review under Rule 42, but
corporation are not described with certainty. held that the CA, notwithstanding the wrong mode of appeal,
should not have easily dismissed the petition, considering that the
 The legislature may apportion the common property or assets parties involved are local government units and that what is involved
and the common borders between or among the corporations in is the determination of their respective territorial jurisdictions. Court
a reasonable and equitable manner. This may be done in the law ruled in favor of Maguihan after taking into consideration the
dividing the corporations, or it may be left to the executive and documents presented by both parties and finding greater weight in
administrative authorities for determination. the cadastral map presented by Maguihan than the assessors’
 Basic Procedure for boundary disputes: certifications presented by Sangalang.
o Disputes between municipal governments over areas or
barangays – heard and decided by the Sangguniang Doctrine: Decision of the sanggunian may be appealed to the RTC
Panlalawigan of the province where the municipalities are having jurisdiction over the area in dispute. When RTC takes
situated. If the municipalities are in different provinces, the cognizance of a case in the exercise of its appellate jurisdiction any
Sangguniang Panlalawigan of both provinces shall jointly further appeal must be through a petition for review under Rule 42.
hear and decide the dispute. If no settlement is reached
within 60 days from referral to the Sangguniang
Panlalawigan, the dispute shall be brought to the RTC, which
B. Manner or Mode
shall decide the case within one year from the start of
proceedings. LGC
o Disputes within barangays – heard and decided by the Sec. 7. Creation and Conversion. - As a general rule, the creation of
Sangguniang Panlungsod or Sangguniang Bayan concerned. alocal government unit or its conversion from one level to another
In case no settlement is reached within 60 days, the case shall level shall be based on verifiable indicators of viability and projected
be heard and decided by the RTC within one year from the capacity to provide services, to wit:
start of court proceedings (a) Income. - It must be sufficient, based on acceptable standards,
to provide for all essential government facilities and services
and special functions commensu- rate with the size of its
Calanza v. PICOP (2009)
population, as expected of the local government unit
Summary: Petitioners filed applications for small-scale mining
concerned;
permits for the purpose of extracting gold, and these were approved
(b) Population. - It shall be determined as the total number of
by the Governor of Davao Oriental. However, the mining areas
inhabitants within the territorial jurisdiction of the local
applied for were within the logging concession area under the
government unit concerned; and
Timber License Agreements with PICOP, and the latter refused the
(c) Land Area. - It must be contiguous, unless it comprises two or
petitioners’ entry. One of the arguments of PICOP was that RTC of
more islands or is separated by a local government unit
Davao Oriental had no jurisdiction over the complaint for injunction
independent of the others; properly identified by metes and
filed by petitioners since the disputed area was in Surigao del Sur,
bounds with technical descriptions; and sufficient to provide for
not Davao Oriental. SC held that RTC cannot exercise either original
such basic services and facilities to meet the requirements of its
or appellate jurisdiction in this case. Court also held that the small-
populace. Compliance with the foregoing indicators shall be
scale mining permits were legally questionable as the governor had
attested to by the Department of Finance (DOF), the
no legal authority to issue said permits
NationalStatistics Office (NSO), and the Lands Management
Bureau(LMB) of the Department of Environment and Natural
Doctrine: The RTC cannot assume original jurisdiction over
Resources(DENR).
boundary disputes since the LGC allocates such power to the

Bautista, Gastanes, Lopez, Sulit Page | 22


C2017 / LAW 154 / Local Government Law / Midterm Reviewer
Sec. 8. Division and Merger. - Division and merger of existinglocal annexed territory shall terminate their official relation with their
government units shall comply with the same requirements herein offices
prescribed for their creation: Provided, however, That such division  On the title to the property of the annexed territory: the annexing
shall not reduce the income, population, or land area of the local territory shall acquire title to the property of the annexed territory
government unit or units concerned to less than the minimum at the time of the annexation, unless the annexing statute
requirements prescribed in this Code: Provided, further, That the provides otherwise.
income classification of the original local government unit or units o When the annexed territory forms part of a municipality from
shall not fall below its current income classification prior to such which it is taken, the legislature may provide for the payment
division. The income classification of local government units shall be of compensation for the indebtedness incurred on account of
updated within six (6) months from the effectivity of this Code to the property taken.
reflect the changes in their financial position resulting from the o Public buildings and improvements in the annexed territory
increased revenues as provided herein. are not required to be paid for by the annexing territory, as
they have already been paid for by the annexed territory.
Sec. 9. Abolition of Local Government Units. - A local government However, if any indebtedness on these exists, the annexing
unit may be abolished when its income, population, or land area has territory must be required to share in the payment of said
been irreversibly reduced to less than the minimum standards indebtedness.
prescribed for its creation under Book III of this Code, as certified by  On the debts and obligations of the annexed territory: those
the national agencies mentioned in Section 17 hereof to Congress or contracted prior to the annexation shall be assumed by the
to the sanggunian concerned, as the case may be. annexing territory in the absence of any provision to the contrary
The law or ordinance abolishing a local government unit shall
specify the province, city, municipality, or barangay with which 2. Effects of Division of Municipal Corporations
thelocal government unitsought to be abolished will be
incorporated or merged.
 On the legal existence of the original corporation: the corporate
Sec. 10. Plebiscite Requirement. - No creation, division, merger, existence of the original municipality is extinguished
abolition, or substantial alteration of boundaries of local  On the property, powers and rights of the original corporation:
government units shall take effect unless approved by a majority of unless the law provides otherwise, when a municipal corporation
the votes cast in a plebiscite called for the purpose in the political is divided into two or more municipalities, each mun icipality
unit or units directly affected. Said plebiscite shall be conducted by acquires title to all the property, powers, rights and obligations
the Commission on Elections (Comelec) within one hundred twenty falling within its territorial limits.
(120) days from the date of effectivity of the law or ordinance
effecting such action, unless said law or ordinance fixes another D. When there is no Dissolution
date.
1. Non-User or Surrender of Charter
1987 Constitution, Art. X
Sec. 10. No province, city, municipality, or barangay may be created,
divided, merged, abolished, or its boundary substantially altered,  Municipal corporations cannot bring about their own dissolution
except in accordance with the criteria established in the Local by a mere surrender of their charter.
Government Code and subject to approval by a majority of the votes  A municipal corporation is not ipso facto dissolved or destroyed
cast in a plebiscite in the political units directly affected. by non-user of its powers in whole or in part, or for its failure for a
number of years to exercise the functions of a municipality.
o In such cases, the municipal corporation would be suspended
C. Effects
but not civilly dead, since its dormant functions could be
revived without action on account of the sovereignty from
1. Effects of Annexation or Consolidation of which, in theory of law, corporate life originally came.
Municipal Corporations
2. Failure to Elect Municipal Officers
 On the legal existence of the territory annexed: unless otherwise
provided, the annexation of one municipal corporation to another  In the absence of a statute, a municipal corporation is not
will dissolve the annexed territory dissolved by the mere failure to elect or appoint its officers and
 On the laws and ordinances of the annexed corporations: in the agents to conduct its government. The officers do not constitute
absence of a provision to the contrary, the annexed territory shall a corporation. Rather, the inhabitants of the designated locality
become subject to the laws and ordinances by which the are the incorporators.
annexing corporation is governed
 On the right of officers or employees of the annexed corporation 3. Change of Sovereignty
to continue to hold their offices: subject to what the legislature
provides upon annexation, the officers and employees of the
Vilas v. Manila

Bautista, Gastanes, Lopez, Sulit Page | 23


C2017 / LAW 154 / Local Government Law / Midterm Reviewer
supra exerted efforts to investigate the facts and verified that there were
no public or private buildings in the said place, hence its conclusion
that there were no inhabitants. It is not impossible for a certain
Sultan Usman Sarangani v. COMELEC (2000)
barangay not to actually have inhabitants considering that people
Summary: A petition for annulment of several precincts and
migrate. A barangay may officially exist on record and the fact that
annulment of book of voters in Madalum, Lanao Del Sur, was filed
nobody resides in the place does not result in its automatic cessation
with the COMELEC. Among the precincts sought to be annulled was
as a unit of local government.
Padian Torogan. COMELEC conducted an ocular inspection on the
alleged ghost precincts and found, among others: that there are
Doctrine: The determination of W/N a certain election precinct
only two structures in the supposed Brgy. Padian Torogan (concrete
actually exists and whether the voters registered in said precinct are
house with no roof, and a wooden structure without walls and roof),
real voters is a factual matter. Under the LGC, the abolition of an
that Padian Torogan means cemetery (not a residential place), and
LGU may be done by Congress in the case of a province, city,
that Padian Torogan is uninhabited. The Madalum Municipal Chief
municipality, or any other political subdivision. In the case of a
of Police also confronted and threatened the COMELEC
barangay, except in Metropolitan Manila area and in cultural
investigating team during the inspection. Based on said inspection,
communities, it may be done by the Sangguniang Panlalawigan or
COMELEC issued the assailed Order finding Padian Torogan to be a
Sangguniang Panglungsod concerned subject to the mandatory
ghost precinct and excluding it from the special election to be
requirement of a plebiscite conducted for the purpose in the political
conducted in Madalum. SC upholds COMELEC’s Order. COMELEC
units affected.

Bautista, Gastanes, Lopez, Sulit Page | 24


C2017 / LAW 154 / Local Government Law / Midterm Reviewer

VI. Plebiscite Requirements

1987 Constitution, Art. X


Sec. 10. No province, city, municipality, or barangay may be created, Held: the Constitution and LGC require a plebiscite. The rule covers
divided, merged, abolished, or its boundary substantially altered, ALL conversions, whether upward or downward, so long as they
except in accordance with the criteria established in the Local result in a material change in the local government unit directly
Government Code and subject to approval by a majority of the votes affected, especially a change in political and economic rights of the
cast in a plebiscite in the political units directly affected. people. RA 8528 is UNCONSTITUTIONAL.

LGC, Sec. 10. Plebiscite Requirement. - No creation, division, merger, Tobias v. Abalos (1994)
abolition, or substantial alteration of boundaries of local Facts: Petitioners assail the constitutionality of RA 7675, which
government units shall take effect unless approved by a majority of converted the Municipality off Mandaluyong into a Highly Urbanized
the votes cast in a plebiscite called for the purpose in the political City (Note: this entailed the separation of San Juan and
unit or units directly affected. Said plebiscite shall be conducted by Mandaluyong, which used to be under one legislative district, into
the Commission on Elections (Comelec) within one hundred twenty separate legislative districts). Peitioners argue that the people of
(120) days from the date of effectivity of the law or ordinance San Juan should have been made to participate in the plebiscite on
effecting such action, unless said law or ordinance fixes another RA 7675, as the same involved a change in their legislative district.
date.
Held: Contention is bereft of merit. The principal subject involved in
Padilla v. COMELEC (1992) the plebiscite was the conversion of Mandaluyong into a highly
Facts: In the plebiscite held for the creation of the Municipality of urbanized city. The matter of separate district was only ancillary
Tulay-Na-Lupa, only 2890 favored its creation while 3439 voted thereto. Thus, the inhabitants of San Juan were properly excluded
against it. Padilla argues that the plebiscite was invalid since the from the plebiscite since they had nothing to do with the change of
plebiscite should have included only the political units affected (i.e. status of Mandaluyong.
the 12 barangays) and excluded the mother Municipality of Labo.
Municipality of Nueva Era, Ilocos v. Marcos (2008)
Held: The plebiscite was valid because “in the political units directly
Facts: Marcos, an adjacent municipality of Nueva Era, claims that
affected,” means that residents of the political entity who would be
the former’s charter provides that is bounded on the east by the
economically dislocated by the separation of a portion thereof have
Ilocos Norte-Mt. Province Boundary. However, Marcos between the
a right to vote. Logically, this includes the mother Municipality.
Ilocos Norte-Mt. Province boundary and the Marcos municipality lies
the Nueva Era municipality. As such, Marcos claims certain portions
Tan v. COMELEC (1996) of Nueva Era as its territory. Nueva Era claimed as one of its defenses
Facts: BP 885 created the Province of Negros del Norte. The that Marcos was created without the plebiscite requirement of the
plebiscite was held only in the areas within the new province, and Constitution so its claim of territory is untenable. The SC agreed with
the rest of Negros Occidental were excluded. Nueva Era’s contention that Marcos’ claim over parts of its territory
is not tenable. However, the reason is not the lack of the required
Held: Boundaries of the existing province of Negros Occidental plebiscite under the 1987 and 1973 constitutions and the Local
would necessarily be substantially altered by the division of its Government Code of 1991 but because Nueva Era was excluded in
existing boundaries in order that there can be created the proposed the enumeration of Marcos’ territory.
new province of Negros del Norte. Therefore, both the new province
and the Mother province come within the meaning of “political units Held: The plebiscite is one instance where the people in their
affected”. sovereign capacity decide on a matter that affects them – direct
democracy of the people as opposed to democracy thru people’s
Miranda v. Aguirre (1999) representatives. This plebiscite requirement is also in accord with
Facts: The Municipality of Santiago Isabela was converted into an the philosophy of the Constitution granting more autonomy to local
INDEPENDENT COMPONENT CITY in 1994. This conversion was government units. Moreover, it is prospective in character and
ratified in a plebiscite. Now RA 8528 was enacted, this time cannot apply to municipalities already created.
DOWNGRADING/CONVERTING it to a component city. Petitioners
assailed RA 8528 for not providing for a plebiscite.

Bautista, Gastanes, Lopez, Sulit Page | 25


C2017 / LAW 154 / Local Government Law / Midterm Reviewer

VII. General Powers of Local Governments

A. Police Power respective territorial jurisdictions, local government units shall


ensure and support, among other things, the preservation and
enrichment of culture, promote health and safety, enhance the right
General Welfare of the people to a balanced ecology, encourage and support the
development of appropriate and self-reliant scientific and
LGC Sec.. 16. General Welfare. - Every local government unit shall technological capabilities, improve public morals, enhance
exercise the powers expressly granted, those necessarily implied economic prosperity and social justice, promote full employment
therefrom, as well as powers necessary, appropriate, or incidental among their residents, maintain peace and order, and preserve the
for its efficient and effective governance, and those which are comfort and convenience of their inhabitants.
essential to the promotion of the general welfare. Within their

Legislative Sangguniang Barangay Sangguniang Bayan Sangguniang Panlungsod Sangguniang


Body (Sec. 391) (Sec. 447) (Sec. 458) Panlalawigan
(Sec. 468)
Powers, (a) Enact ordinances (a) Enact ordinances, (a) Approve ordinances and (a) Approve ordinances
Duties, and (b) Enact tax and revenue approve resolutions and pass resolutions and pass resolutions
Functions ordinances appropriate funds necessary for an (b) Generate and maximize
(c) Enact annual and (b) Generate and maximize efficient and effective the use of resources
supplemental budgets the use of resources and city government and revenues
(d) Provide for the construction revenues (b) Generate and maximize (c) Adopt measures to
and maintenance of facilities (c) Grant franchises, enact the use of resources and enhance the full
and projects chargeable to ordinances authorizing revenues implementation of the
the general fund of the the issuance of permits (c) Enact ordinances national agrarian
Barangay or licenses, or enact granting franchises and reform program in
(e) Submit to the Sangguniang ordinances levying authorizing the issuance coordination with DAR;
Panlungsod or Sangguniang taxes, fees and charges of permits or licenses (d) Grant franchises,
(f) Bayan suggestions or upon such conditions (d) Regulate activities approve the issuance of
recommendations (d) Regulate activities relative to the use of permits or licenses, or
(g) Assist in the establishment, relative to the use of land, buildings and enact ordinances
of cooperative enterprises land, buildings and structures within the city levying taxes, fees and
that will improve the structures within the (e) Approve ordinances charges
economic condition and municipality which shall ensure the (e) Approve ordinances
well-being of the residents; (e) Approve ordinances to efficient and effective which shall ensure the
(h) Regulate the use of facilities ensure the efficient and delivery of the basic efficient and effective
constructed with effective delivery of the services and facilities delivery of the basic
government funds and basic services and (f) Exercise such other services and facilities
charge reasonable fees h) facilities powers and perform
Solicit or accept monies, (f) Exercise such other such other duties and
materials and voluntary powers and perform functions as may be
labor such other duties and prescribed by law or
(i) Solicit or accept cooperation functions as may be ordinance.
by national, provincial, city, prescribed by law or
or municipal agencies to ordinance.
render financial, technical,
and advisory assistance to
Barangays and to Barangay
residents

Bautista, Gastanes, Lopez, Sulit Page | 26


C2017 / LAW 154 / Local Government Law / Midterm Reviewer
White Light Corp v. City of Manila (2009) Technology Developers Inc. v. CA (1991)
Summary: Owners and operators of hotels filed a case to invalidate Summary: Petitioner, Technology Developers Inc., is a domestic
an ordinance sanctioning any person or corporation who will allow private corporation engaged in the manufacture and export of
the admission and charging of room rates for less than 12 hours. The charcoal briquette. The acting mayor ordered the full cessation of
SC held that such ordinance is an invalid exercise of police power for the operation of the petitioner's plant because of the air pollution.
not meeting the requisites for valid exercise of police power. Petitioner instituted an action for certiorari, prohibition, mandamus
with preliminary injunction which granted but later on dissolved due
Doctrine: Police power, while incapable of an exact definition, has to adduced evidence. The SC held that the lower courts did not err
been purposely veiled in general terms to underscore its in denying the motions of the petitioner. Factors were considered,
comprehensiveness to meet all exigencies and provide enough room such as lack of mayor’s permit, to justify the dissolution of the writ
for an efficient and flexible response as the conditions warrant. It is of preliminary injunction.
based upon the concept of necessity of the State and its
corresponding right to protect itself and its people. In order to test Doctrine: It must be recognized that the mayor of a town has as
the validity of a police power measure, the ff. requisites must concur: much responsibility to protect its inhabitants from pollution, and by
(1) It must appear that the interests of the public generally, as virtue of his police power, he may deny the application for a permit
distinguished from those of a particular class, require an to operate a business or otherwise close the same unless
interference with private rights. appropriate measures are taken to control and/or avoid injury to the
(2) It must also be evident that no other alternative for the health of the residents of the community from the emissions in the
accomplishment of the purpose less intrusive of private rights operation of the business.
can work.
(3) A reasonable relation must exist between the purposes of the Chua Huat v. CA (1991)
measure and the means employed for its accomplishment Summary: Petitioners Chua Hua et al. were residents of buildings
that were condemned and ordered to be demolished. The orders
Ermita Malate v. City of Manila (1967) were based on inspection reports, which showed that the buildings
Summary: Ermita-Malate Hotel and Motel Operators Association, suffer from structural deterioration by more than 50% and as much
and one of its members Hotel del Mar Inc. petitioned for the as 80%. Petitioners formally protested the condemnation orders
prohibition of Ordinance 4670, claiming Sec. 1 of the ordinance was over 3 months after receiving notice. SC held that there was no grave
unconstitutional and void for being unreasonable and violative of abuse of discretion because the orders were issued based on
due process because it would impose higher license fees for motels; inspections. Moreover, petitioners failed to exhaust administrative
because it required that guests would fill up a form specifying their remedies (appeal to the Mayor within 7 days, or request for re-
personal information. There was also a provision that the premises inspection within 15 days).
and facilities of such hotels, motels and lodging houses would be
open for inspection from city authorites. They claimed this to be Doctrine: Power to condemn and remove buildings and structures is
violative of due process for being vague. The ordinance also caused an exercise of the police power granted to the City to promote public
the automatic cancellation of the license of the hotels that violated safety
the ordinance. Lower court: ordinance unconstitutional. SC held
Ordinance constitutional as valid exercise of police power to Binay v. Domingo (1991)
safeguard common good, curtail illegal activities and protect public Summary: Resolution No. 60 was enacted to give relief to grieving
morals. families whose gross income does not exceed 2k/month for 500php.
This was approved by the Metro Manila Commission and was
Doctrine: To satisfy the due process requirement, official action, referred to respondent COA for its audit. It disallowed the
must not outrun the bounds of reason and result in sheer disbursement of funds for the project. Makati’s Municipal Council
oppression. Due process is freedom from arbitrariness. passed another Resolution No. 243 re-affirming the Burial
Assistance Fund. Since this project has been stayed by the COA
Dela Cruz v. Paras (1983) Binay filed a special civil action of certiorari with the SC to set aside
Summary: Ordinance No. 84 was enacted by the Municipal Council the COA decision. SC grants the petition.
of Bocaue, Bulacan. This Ordinance prohibits the operation of
nightclubs, cabaret joints, etc. in the vicinity of Bocaue. Petitioners, Doctrine: It held that it is a valid exercise of Police Power under the
owners of such establishments, filed 2 cases for Prohibition with the general welfare clause (in BP 337) taking particular notice in the shift
CFI. CFI finds the ordinance not unconstitutional because of Police towards social legislation in the constitution.
Power granted to Municipalities. SC however struck down the
ordinance because BP 337 gives the Sangguniang Bayan no power Tatel v. Municipality of Virac
to prohibit the establishments involved only to regulate. Summary: Tatel is a businessman engaged in the import and export
of abaca and other products. His warehouse was subject of a
Doctrine: Police Power may be exercised by a Municipality however Municipal Resolution wherein it was declared to be a public
it must be read in connection with the constitution and pertinent nuisance and he was ordered to transfer the same within 2 months.
laws. Municipalities have no power to prohibit, only regulate. Tatel filed a prohibition case before the CFI and lost. On appeal, he

Bautista, Gastanes, Lopez, Sulit Page | 27


C2017 / LAW 154 / Local Government Law / Midterm Reviewer
asserted that Ordinance 13 (the law being implemented by the Parayno v. Jovellanos (2006)
Municipal Resolution) was unconstitutional, and that the CFI added Summary: Parayno’s gas station in Calasiao was complained of
a new meaning to the Ordinance by holding that the law penalized before the SB. They then issued Resolution No. 50 recommending
the storing of inflammable objects when the law only penalized the to the Mayor the closure or transfer of the station on the ground that
construction of the warehouses. SC ruled for the validity of the it was in violation of the Zoning Code, and that it endangered the
Ordinance and Resolution. It held that while the syntax of the lives and safety of the people, among others. The SC held that the
Ordinance is problematic (which is common on legislation of local Municipality invalidly exercised its police powers by failing to accord
bodies whose members lack education and training), the law’s Parayno due process (Req #2*) in issuing the Resolution. There was
purpose is to avoid loss of life and property in case of accidental fire no showing that there was actual measurement conducted to verify
– and the law really intends to penalize the construction of that the station indeed violated a distance requirement. Moreover,
warehouses wherein inflammable objects were to be stored. the distance requirement cited did not apply to Parayno’s filling
station but only to service stations as clearly distinguished under the
Doctrine: Municipal corporations are agencies of the State for the Zoning Code. In addition, an earlier HLURB decision has ruled that
promotion and maintenance of local self-government and are she has complied with all legal requirements, and that the station
endowed with police powers in order to accomplish and carry out the was not a threat to safety.
declared objects of their creation.
Doctrine: Requisites to consider that a local government has
Judge Tamin v. CA (1992) properly exercised its police powers: (1) the interests of the public
Summary: The Municipality of Dumingag filed an ejectment suit with generally, as distinguished from those of a particular class, require
prayer for a writ of preliminary injunction and writ of possession over the interference of the State, and (2) *the means employed are
a parcel of land rented by private respondents. Judge Tanim applied reasonably necessary for the attainment of the object sought to be
the rule on eminent domain granted the writ of possession and accomplished and not unduly oppressive.
issued a writ of demolition for the buildings of private respondents.
CA declared that the issued writs of the RTC were null and void. The Greater Balanga Devt Corp v. Municipality of Balanga
SC held that under the then Local Government Code (BP 337), the (1994)
Sangguniang Bayan has to first pass an ordinance before the Summary: Petitioner applied for a business permit to engage in
municipality may summarily abate a public nuisance. A public plaza business as a public market operator and real estate dealer. The
(even if still to be constructed) is outside the commerce of man and permit was initially granted but the Sangguniang Bayan of Balanga
constructions thereon can be abated summarily by the municipality. passed a resolution annulling the said permit. Pursuant to that
However, the writs were still unjustified because the status of the resolution, Mayor Banzon issued an executive order revoking the
ownership of the parcel was still in question in the cadastral permit. According to the SC, the permit was not rightfully revoked.
proceedings, and its outcome would become a prejudicial question The different grounds raised by respondents (false statements in
that would need to be addressed application form, application for 2 businesses in one permit, bad
faith, land was subject of adverse claims) were not sufficient to
Doctrine: LGC allows the municipality, through legislative body, to justify the revocation.
pass an ordinance in order to summarily abate a public nuisance.
Doctrine: The Sangguniang Bayan has the duty in the exercise of its
Patalinhug v. CA (1994) police powers to regulate any business subject to municipal license
Summary: Sangguniang Panlungsod enacted an ordinance on fees and prescribe the conditions under which a municipal license
zoning of the city. A building permit was issued to the petitioner to already issued may be revoked. In this case, the claim that the EO
construct a funeral parlor at Cabaguio Avenue. Respondents and resolution were measures “designed to promote peace and
complained that the said funeral parlor was within 50 m radius of order and protect the general welfare of the people” was considered
INC Chapel and resident structures. The Court held that petitioner by the Court to be too amorphous and convenient an excuse to justify
did not violate the ordinance. The Court found that the nearest respondents’ acts.
structure is a commercial building. Even though the Tepoot building
was declared as residential for tax purposes, it was reclassified as Tano v. Socrates (1997)
commercial by the local government and that determination for Summary: Petitioners are assailing the constitutionality and validity
zoning purposes must prevail. of Ordinances (banning shipment of lobsters et. al.), which were
enacted by the Sangguniang Panlungsod of Puerto Princesa and the
Doctrine: Declaration of the area as a commercial zone thru a Sangguniang Panlalawigan of Palawan. SC held that in light of the
municipal ordinance is an exercise of police power to promote the principles of decentralization and devolution enshrined in the LGC
good order and general welfare of the people in the locality. The and the powers granted to LGUs under Sec. 16 (General Welfare
state, in order to promote the general welfare, may interfere with Clause), and under Secs. 149, 447 (a) (1) (vi), 458 (a) (1) (vi) and 468
personal liberty, with property, and with business and occupations. (a) (1) (vi), which unquestionably involve the exercise of police power,
the validity of the questioned Ordinances cannot be doubted.

Bautista, Gastanes, Lopez, Sulit Page | 28


C2017 / LAW 154 / Local Government Law / Midterm Reviewer
Doctrine: Indispensable to the system of decentralization is power through their respective legislative bodies. Through the
devolution. One of the devolved powers enumerated in the section general welfare clause, LGUs may prescribe regulations to protect
of the LGC on devolution is the enforcement of fishery laws in the lives, health, and property of their constituents and maintain
municipal waters including the conservation of mangroves. This peace and order within their respective territorial jurisdictions.
necessarily includes enactment of ordinances to effectively carry out
such fishery laws within the municipal waters. Closure and Opening of Roads

City of Manila v. Laguio (2005) LGC Sec. 21. Closure and Opening of Roads. –
Summary: Malate Tourist Dev’t Corp. (owner of Victoria Court in (a) A local government unit may, pursuant to an ordinance,
Malate) filed a petition for declaratory relief challenging the permanently or temporarily close or open any local road, alley,
constitutionality of Ordinance No. 7783, enacted by the City Council park, or square falling within its jurisdiction: Provided, however,
of Manila. The Ordinance prohibited the establishment or operation That in case of permanent closure, such ordinance must be
of businesses providing certain forms of amusement, entertainment, approved by at least two-thirds (2/3) of all the members of the
services and facilities in the Ermita-Malate area. MTDC prayed that sanggunian, and when necessary, an adequate substitute for
the Ordinance, insofar as it includes motels and inns among its the public facility that is subject to closure is provided.
prohibited establishments, be declared invalid and (b) No such way or place or any part thereof shall be permanently
unconstitutional. Judge Laguio rendered the assailed decision, closed without making provisions for the maintenance of public
enjoining petitioners from implementing the Ordinance, and safety therein. A property thus permanently withdrawn from
declaring it null and void. SC upholds Judge Laguio/RTC. public use may be used or conveyed for any purpose for which
The Ordinance invades fundamental personal and property rights other real property belonging to the local government unit
and impairs personal privileges. It is constitutionally infirm. It concerned may be lawfully used or conveyed: Provided,
contravenes statutes; it is discriminatory and unreasonable in its however, That no freedom park shall be closed permanently
operation; it is not sufficiently detailed and explicit that abuses may without provision for its transfer or relocation to a new site.
attend the enforcement of its sanctions. The City Council under the (c) Any national or local road, alley, park, or square may be
LGC had no power to enact the Ordinance and is therefore ultra temporarily closed during an actual emergency, or fiesta
vires, null and void. In spite of its virtuous aims, the enactment of the celebrations, public rallies, agricultural or industrial fairs, or an
Ordinance has no statutory or constitutional authority to stand on. undertaking of public works and highways,
telecommunications, and waterworks projects, the duration of
Doctrine: Local legislative bodies cannot prohibit the operation of which shall be specified by the local chief executive concerned
the enumerated establishments under Section 1 or order their in a written order: Provided, however, That no national or local
transfer or conversion without infringing the constitutional road, alley, park, or square shall be temporarily closed for
guarantees of due process and equal protection of laws, not even athletic, cultural, or civic activities not officially sponsored,
under the guise of police power. LGUs, as agencies of the State, are recognized, or approved by the local government unit
endowed with police power in order to effectively accomplish and concerned.
carry out the declared objects of their creation. The police power of (d) Any city, municipality, or barangay may, by a duly enacted
the City Council, however broad and far-reaching, is subordinate to ordinance, temporarily close and regulate the use of any local
the constitutional limitations thereon; and subject to the limitation street, road, thoroughfare, or any other public place where
that its exercise must be reasonable and for the public good. The shopping malls, Sunday, flea or night markets, or shopping
police power granted to LGUs must always be exercised with utmost areas may be established and where goods, merchandise,
observance of the rights of the people to due process and equal foodstuffs, commodities, or articles of commerce may be sold
protection of the law. and dispensed to the general public.

Roble v. Villaflor (2006) Sangalang v. IAC (1989)


Summary: Roble’s application for a permit was denied by the Summary: The main issue is the validity of the act of the Mayor of
Municipal Mayor pursuant to a Resolution – disallowing a company opening Jupiter Streets and Orbit Streets to the public. BAVA
that owns a shipping line to perform arrastre services. Roble filed a opposed and alleged that the streets have always been kept open
petition for mandamus to compel the Mayor to issue the permit voluntarily by the Association and that Rufino R. Santos, BAVA
alleging that it is her ministerial duty and it is the PPA which has the president, has never agreed on the opening of Jupiter and Orbit
discretion to issue or refuse permits. SC held that mandamus cannot streets. SC held that opening of Jupiter Street was warranted by the
lie to compel the Mayor to perform a discretionary duty. demands of the common good, in terms of traffic decongestion and
public convenience. Opening of Orbit was also upheld on the same
Doctrine: The Mayor’s power to issue licenses is pursuant to Sec. 16 rationale. The very Deed of Donation executed by the Ayala
of LGC (Gen. Welfare Clause), which encapsulates the delegated Corporation covering Jupiter and Orbit Streets effectively required
police power to local governments. Municipal mayor has the power both passageways open to the general public. There is no merit in
to issue licenses and permits and suspend or revoke the same for BAVA's claims that the demolition of the gates at Orbit and Jupiter
any violation of the conditions upon which said licenses or permits Streets amounts to deprivation of property without due process of
had been issued, pursuant to law or ordinance. LGUs exercise police law or expropriation without just compensation. There is no taking

Bautista, Gastanes, Lopez, Sulit Page | 29


C2017 / LAW 154 / Local Government Law / Midterm Reviewer
of property involved here. The act of the Mayor now challenged is in were intended to serve i.e. as arteries of travel for vehicles and
the concept of police power. BAVA has failed to show that the pedestrians.
opening up of Orbit Street was unjustified, or that the Mayor acted
unreasonably Macasiano v. Diokno (1992)
Summary: The Municipality of Parañaque passed Ordinance No. 86
Doctrine: Unlike the power of eminent domain, police power is
Series of 1990 authorizing the closure of five streets (J. Gabriel, G.G.
exercised without provision for just compensation. However, it may
Cruz, Bayanihan, Lt. Garcia Extension and Opena) for the
not be done arbitrarily or unreasonably. But the burden of showing establishment of a flea market. The municipal council approved this;
that it is unjustified lies on the aggrieved party.
the MMA also approved subject to four conditions. Thereafter, the
municipality of Parañaque and Palanyag entered into an agreement
Cabrera v. CA (1991) for the management of the vending areas, pursuant to a resolution
Summary: An old road in the town of Virac was CLOSED by virtue of authorizing Mayor Ferrer to enter into a contract for the said
Resolution 158 adopted by the Provincial Board of Catanduanes. For purpose. Brig. Gen. Macasiano ordered the destruction and
the construction of the new road, the governor executed Deeds of confiscation of the stalls along G.G. Cruz and J. Gabriel St. and wrote
Sale with 7 property owners who had properties adjacent to the new Palanyag to discontinue the flea market within ten days. The RTC
road. In exchange for these properties, they were given portions in issued a TRO against the implementation of Brig. Gen Macasiano’s
the old road. The petitioner had property along the old road and letter. RTC later on upheld the validity of the ordinance. SC reversed.
claims to be prejudiced by the new private owners of the portions of
the old road. He claims that the old road is a public road owned by Doctrine: Roads and streets which are available to the public in
the province in its governmental capacity and thus cannot be general and ordinarily used for vehicular traffic are still considered
sold/bartered. SC held that the provincial government, like the city public property devoted to public use. In such case, the local
councils, are authorized under RA 5185 in relation to Sec 2246 or government has no power to use it for another purpose or to dispose
RAC. WRT damages, the court held that Cabrera is not entitled to of or lease it to private persons.
them.
Pilapil v. CA (1992)
Doctrine: Provincial board has the duty of maintaining such roads for
Summary: Sps Pilapil and Sps Colomida owned lands near each
the comfort and convenience of its inhabitants. This authority is
other’s. The property of Colomidas has for its use a camino vecinal
inferred from the grant by the national legislature of the funds to the
connected to the National Road, however, this traverses the Sps
Province for the construction of provincial roads.
Pilapil’s property, whom had conveyed threats to the Colomidas for
trying to improve the road. SC ruled in favor of the Pilapils. There is
Dacanay v. Asistio (1992) unrebutted evidence that the Municipality of Liloan, through its
Summary: In 1979, an ordinance was enacted by the Metropolitan Sangguniang Bayan, had approved a zoning plan which indicates
Manila Commission, which designated streets and open spaces as the relative location of the camino vecinal. Under the zoning map
sites for flea markets. Pursuant thereto, Caloocan City Mayor Virgilio approved by the Sangguniang Bayan, the camino vecinal passes the
Robles enacted EO 135, which authorized the use of certain streets side of the land of the Pilapils
and spaces as sites for said markets. Heroes del ’96, where
Petitioner Dacanay lives, was one of those streets. In 1987, Mayor Doctrine: A camino vecinal is a municipal road. The Municipality had
Antonio Martinez caused the demolition of the stalls on the streets the unassailable authority to (a) prepare and adopt a land use map,
and open spaces. The stallholders filed a case for prohibition with (b) promulgate a zoning ordinance which may consider, among
the RTC, but it was dismissed. Later, Martinez was succeeded by other things, the municipal roads to be constructed, maintained,
Respondent Macario Asistio as Mayor of Caloocan City. The latter improved or repaired and (c) close any municipal road. No private
did not pursue the former’s policy of clearing the streets. Dacanay party can interfere with such a right.
wrote to Asistio to implement the RTC decision and continue the
demolition, since Heroes del ’96 still had stalls, but Asistio did not
MMDA v. Bel-Air Village Ass’n Inc (2000)
act. Eventually, Dacanay filed a petition for mandamus with the SC,
Summary: MMDA notified BAVA through a letter that it is requesting
praying that the respondents be ordered to enforce the RTC
BAVA to open Neptune St. (private road owned by BAVA) to the
decision. SC granted the petition. The streets are, no doubt, public
public. On the same day of receipt of the letter BAVA was informed
streets.
that the perimeter wall separating the subdivision from the adjacent
Kalayaan Ave. would be demolished. BAVA prayed for the issuance
Doctrine: The public street is property for public use, hence outside
of a TRO and preliminary injunction enjoining the opening of
the commerce of man. Being outside the commerce of man, it may
Neptune and prohibiting the wall demolition. RTC denied injunction,
not be the subject of lease or other contract. The right of the public
but CA granted it. SC held that MMDA does not have authority to
to use the city streets may not be bargained away through contract.
open Neptune St.
The EO issued by Mayor Robles in 1979 contravenes the general law
Doctrine: The MMDA has limited powers which does not include
that reserves city streets and roads for public use; it infringe upon
police power. Not being a political subdivision, the MMDA is limited
the vested right of the public to use city streets for the purpose they
to the administration and implementation of basic services provided
in its charter and cannot open Neptune St. without the proper

Bautista, Gastanes, Lopez, Sulit Page | 30


C2017 / LAW 154 / Local Government Law / Midterm Reviewer
ordinance. National Legislature may delegate police power to the B. Eminent Domain
President and administrative boards as well as lawmaking bodies of LGC Sec. 19. Eminent Domain. - A local government unit may,
municipal corporations or local government units. Once delegated, through its chief executive and acting pursuant to an ordinance,
the agents can exercise only such legislative powers as are conferred exercise the power of eminent domain for public use, or purpose or
on them by the national lawmaking body. welfare for the benefit of the poor and the landless, upon payment
of just compensation, pursuant to the provisions of the Constitution
Sun Valley Home Owner’s Association v. Barangay Sun and pertinent laws: Provided, however, That the power of eminent
Valley (2011) domain may not be exercised unless a valid and definite offer has
Summary: Barangay Sun Valley issued a resolution ordering the been previously made to the owner, and such offer was not
opening of Rosemallow and Aster street. This was contested by New accepted: Provided, further, That the local government unit may
Sun Valley Homeowner’s Assoc because it was not done through an immediately take possession of the property upon the filing of the
ordinance as mandated by law. The court held that since the lots expropriation proceedings and upon making a deposit with the
were donated to the barangay, they had title to it and it became proper court of at least fifteen percent (15%) of the fair market value
public property. Being public property, it should be open to the of the property based on the current tax declaration of the property
public and not obstructed by the gates placed by the homeowner’s to be expropriated: Provided, finally, That, the amount to be paid for
association. It is not an opening of the road that was done by the the expropriated property shall be determined by the proper court,
barangay which requires an ordinance, but a reminder to the based on the fair market value at the time of the taking of the
homeowner’s association that they should open the road because it property
should actually be open for the general public.
Doctrine: Having been already donated or turned over to the City ART. III, Sec 9 Constitution – Private Property shall not be taken for
Government, the road lots have since then taken the nature of public public use without just compensation
roads which are withdrawn from the commerce of man, and hence
placed beyond the private rights or claims of private party.

RULE 67 Expropriation

Rights of Plaintiff after


Defenses and Order of
Verified Complaint Entry of plaintiff Judgment and
Objections Expropriation
Payment

With
objections/defenses: Ascertainment of
Enter and Appropriate
answer within time Compensation
stated in summons

If none: notice of Uncertain Ownership;


Not Delayed by Appeal
appearance Conflicting Claims

Commissioners Retain

Report and judgment

Bautista, Gastanes, Lopez, Sulit Page | 31


C2017 / LAW 154 / Local Government Law / Midterm Reviewer
1. Contents of Complaint (Sec 1): 7. Rights of Plaintiff (Sec 10)
 Right and purpose of expropriation  Upon payment of compensation (with legal interest)
 Describe the real or personal property o Right to enter
 Join defendants – all persons owning or claiming to own or o Right to retain (if he took immediate possession pursuant to
occupying any part or interest. Sec 2)
 Make averments if a) title is with the Republic but occupied by
private individuals or; b) title is obscure or doubtful 8. Entry Not Delayed by Appeal (Sec 11)
 But if appellate court decides that plaintiff had no right to
2. Entry of Plaintiff (Sec 2): expropriate
 Upon filing of complaint and after due notice to defendant/s o Order RTC to restore possession to defendant
 Deposit of amount equivalent to assessed value o Determine damages

3. Defenses and Objections (Sec 3)


 No objections Moday v. CA (1993)
o File notice of appearance and manifestation Summary: Municipality of Bunawan expropriated Moday’s private
o Contents: 1) Manifestation to the effect that he has no property by virtue of a municipal resolution, which was disapproved
objection or defense; and (2) Specifically by the Sangguniang Panlalawigan. The SC held that the
designating/identifying the property in which he claims to be expropriation may proceed. The Sangguniang Panlalawigan's
interested disapproval of Municipal Resolution No. 43-89 is an infirm action,
 With Objections which does not render said resolution null and void. The
o File an answer to complaint within time stated in summons Sangguniang Panlalawigan was without the authority to disapprove
o Contents: 1) Specifically designating/identifying the property Municipal Resolution No. 43-89 for the Municipality of Bunawan
in which he claims to have an interest in; (2) Nature and clearly has the power to exercise the right of eminent domain and its
extent of the interest; and (3) All objections and defenses to Sangguniang Bayan the capacity to promulgate said resolution,
the complaint or any allegation pursuant to the earlier-quoted Section 9 of B.P. Blg. 337.
o No counterclaim, cross-claim, or third-party complaint
alleged or allowed. Doctrine: The limitations on the power of eminent domain are that
the use must be public, compensation must be made and due
4. Order of Expropriation (Sec 4): process of law must be observed.
 Issued when: 1) Objections or defenses against the right of
plaintiff to expropriate are overruled; or 2) No party appears to Camarines Sur v. CA (1993)
defend the case Summary: Pursuant to Resolution #129, the Province of Camarines
Sur filed expropriation cases against the San Joaquin brothers. The
5. Ascertainment of Compensation (Sec 5) latter moved to dismiss on the ground of inadequacy of price. RTC
 Court appoints not more than 3 Commissioners who will denied the MTD and authorized the Province to take possession of
ascertain the just compensation for property the property upon deposit of a bond. CA reversed and required the
 Objections to appointment: 1) to be filed within 10 days; 2) to be Province to secure authority from DAR to convert the land from
resolved within 30 days agricultural to non-agricultural. SC upheld the RTC, ruling that
there was no requirement for DAR approval under the (old) LGC, nor
6. Commissioners (Sec 6 – 8) in the CARL.
 Commissioners take and subscribe to an oath
 To view and examine the property sought to be expropriated Doctrine: While LGUs’ delegated power of eminent domain is only
 Assess the consequential damages and benefits limited authority, it is complete within its limits. The limitations on
the exercise of the delegated power must be clearly expressed,
 Report – to be filed within 60 days from the time of their
either in the law conferring the power or in other legislations.
appointment
Statutes conferring the power of eminent domain to political
o Parties allowed to object to findings of report within 10 days
subdivisions cannot be broadened or constricted by implication.
 Court may accept report or recommit the same for further facts
o May also set aside report and appoint new commissioners
o May also accept and reject report in part Municipality of Meycauayan v. IAC (1988)
Summary: Municipality of Bulacan wanted to expropriate the lot of
6. Uncertain Ownership; Conflicting Claims (Sec 9) Philippine pipes. Philippine Pipes opposed such resolution. It was
 If ownership of property is uncertain or there are conflicting found upon a petition for review on certiorari to the SC that there
claims was no genuine necessity for the lot to be expropriated since there
o Court may order sums awarded as compensation was other areas (adjacent to the lot) more suitable for the public
o Judgment shall require payment of sums awarded to road project.
defendant before plaintiff can enter property or retain it for
public use

Bautista, Gastanes, Lopez, Sulit Page | 32


C2017 / LAW 154 / Local Government Law / Midterm Reviewer
Doctrine: The government may not capriciously choose what private court is to ascertain the value of the land in question. The courts
property should be taken. With due recognition then of the power of cannot inquire into the necessity of the expropriation. The Supreme
Congress to designate the particular property to be taken and how Court disagreed. It held that based on the records, that there is no
much thereof may be condemned in the exercise of the power of proof of the necessity of opening the road through the cemetery.
expropriation, it is still a judicial question whether in the exercise of (Thus, because no necessity exists, the Court did not categorically
such competence, the party adversely affected is the victim of classify the cemetery as public, although the Court noted that it
partiality and prejudice. seems to have been established under governmental authority. If it
is public cemetery, then the City cannot expropriate a property
already devoted for public use).
Quezon City v. Ericta (1983)
Summary: QC Council enacted an Ordinance. With Sec 9 thereof
Doctrine: If the expropriation is of a certain or particular parcel of
mandating memorial park lots to donate 6% of their land are in
land for some specified public purpose, then the courts would be
order to accommodate pauper burials. SC declared Sec 9 as
without jurisdiction to inquire into the purpose of that legislation. If,
unconstitutional. When the 1983 LGC provided that a sanggunian
upon the other hand, however, the Legislature should grant general
may provide for the burial of the dead in such place and manner as
authority to a municipal corporation to expropriate private land for
prescribed by law or ordinance, it simply authorized the city to
public purposes, the courts have ample authority to make inquiry
provide its own city-owned land or to buy/expropriate private
and to hear proof, upon an issue properly presented, concerning
properties to construct public cemeteries.
whether or not the lands were private and whether the purpose was,
in fact, public.
Doctrine: It is not within the police powers of the City, as it
constitutes confiscation and not merely regulation, (confiscation in
the context of police powers would lead to destruction, and not Municipality of Parañaque v. V.M. Realty Corporation
public use). Neither is it expropriation because there is no just (1998)
compensation when these lots were “donated”. Summary: The Municipality of Paranaque filed a complaint for
expropriation against VM Realty over parcels of land on the strength
City of Manila v. Arellano College (1950) of a SB Resolution. The RTC and CA denied this, stating that there
must first be an ordinance, and that the action was barred by res
Summary: Manila City sought to expropriate the land belonging to
judicata. The SC agreed that there was no cause of action for such
Arellano Law. CFI dismissed plaintiff’s petition. The SC affirmed and
denied the petition for expropriation because it would only benefit a complaint must be pursuant to an ordinance. However, res judicata
cannot bar the State’s exercise of eminent domain, thus Paranaque
few families (who were so poor that they could not afford to buy the
P140,000 lands), and the land was to be Arellano’s permanent may later file an action for expropriation provided all legal requisites
school site (which would benefit the public by preparation of a young are met.
men and young women for useful citizenship and for service to the
government and the community. Doctrines: (1) With the enactment of the LGC, an ordinance is
required to exercise the power of eminent domain. Mere resolution
will not suffice. (2) The principles of res judicata and law of the case
Doctrine: To authorize the condemnation of any particular land by a
grantee of the power of eminent domain, a necessity must exist for cannot bar the State’s exercise of power of eminent domain.
the taking thereof for the proposed uses and purposes. The very
foundation of the right to exercise eminent domain is a genuine Beluso v. Panay (2006)
necessity, and that necessity must be of a public character. The Summary: Petitioners were owners of lots which the Municipality of
ascertainment of necessity must precede or accompany, and not Panay, Capiz sought to expropriate via SB Resolution. The RTC
follow, the taking of the land. Necessary does not mean an absolute granted it and issued an Order. Upon appeal to the CA, Petitioners
but only a reasonable or practical necessity, such as would combine raised the argument that the expropriation was invalid for being
the greatest benefit to the public with the least inconvenience and based on mere resolution, not on an ordinance. The CA did not
expense to the condemning party and property owner consistent discuss nor rule on this point and proceeded to uphold the
with such benefit. expropriation. Upon appeal to the SC, the petitioners averred that
the expropriation was invalid for not being based on an ordinance,
City of Manila v. Chinese Community of Manila (1920) for lacking a previous offer, and for being politically motivated rather
Summary: City of Manila petitioned to expropriate lands for the than for public purpose. The SC held that the first requisite of an
purpose of constructing a public road. Some lands affected are used ordinance was not met therefore all other issues raised were
by the Chinese community as cemetery. The Chinese community unnecessary. The complaint for expropriation was defective.
avers that there is no necessity to expropriate their land. IN fact, Mr.
Tambunting is offering the city another parcel of land for the Doctrine: The Court is not precluded from considering the fact of lack
of proper ordinance for expropriation despite being raised for the
proposed road that would not traverse the Chinese cemetery. CFI
first time on appeal.
held that there was no necessity to expropriate the land of the
Chinese community. The city of Manila appealed this decision,
arguing once its authority to expropriate land has been established,
it may expropriate any land it may desire, the only function of the

Bautista, Gastanes, Lopez, Sulit Page | 33


C2017 / LAW 154 / Local Government Law / Midterm Reviewer
City of Cebu v. Sps. Dedamo (2002)
Summary: Petitioner sought to obtain parcels of land from
respondent through expropriation. After having agreed to conform
to the valuation of the commissioners, petitioner brought an action
contesting the assessments of said commissioners, claiming that
just compensation should be based on the market price of the
property at the commencement of the expropriation proceedings.
SC held that just compensation should be determined as of the time
of taking, the applicable law in this case is Sec. 19, RA 7160. Other
reasons for denying the petition were: the document agreed upon by
the parties should be complied with in good faith, petitioner is
estopped from contesting the values as it failed to object during the
hearing, and substantive law should prevail over procedural law.

Doctrine: Eminent domain is a fundamental State power that is


inseparable from sovereignty. It is the government’s right to
appropriate, in the nature of a compulsory sale to the State, private
property for public use or purpose. However, the Government must
pay the owner thereof just compensation – to be determined at the
time of taking – as consideration.

Republic of the Philippines v. CA (2002)


Summary: Expropriation proceedings, involving land owned by Luis
Santos) were commenced in 1969. The Republic took possession of
the premises and made initial deposit. In 1979, RTC condemned the
properties, ordered payment of just compensation and legal
interest. The Republic failed to pay JC. In 1984, Heirs of Santos
sought payment of JC. When the Republic motioned to deposit the
JC with the court, the Heirs countered, wanting to adjust the
amount, or otherwise have the property returned. In the same year,
RTC ruled that the 1979 Decision had become unenforceable on the
ground of prescription, and ordered the return of the property to the
Heirs. Court held that the 1979 Decision had been partially complied
with upon the Republic’s occupation, utilization, and exercise of
dominion over the property and payment of initial deposit. Court
held that the Heirs are not entitled to recovery of the property,
emphasizing that the case relied upon by the Heirs involved a
municipal government, which did not possess the pervasive and all-
encompassing power vested in the legislative branch of
government. Legal interest was also properly imposed from the time
of taking in 1969.

Doctrine: For local governments to be able to power of eminent


domain, it must, by enabling law, be delegated to it by the national
legislature. But even then, this delegated power of eminent domain
is not a power of eminent, but only of inferior, domain. Expropriated
lots cannot be recovered since they have already taken the character
of public property; remedy is to demand for fair market value of the
property.

Bautista, Gastanes, Lopez, Sulit Page | 34


C2017 / LAW 154 / Local Government Law / Midterm Reviewer
C. Taxation and Fiscal Administration o Taxes on agricultural/aquatic products when sod by
marginal farmers/fishermen
o Excise taxes under the NIRC, and taxes on petroleum
1. Local Taxation products
o Taxes on persons engaged in transportation;
1987 Constitution o Percentage or VAT on sales or similar transactions;
o Charges for motor vehicle registration;
Art X. Sec. 5. Each local government unit shall have the power to o Taxes on the National Government, its agencies and
create its own sources of revenues and to levy taxes, fees, and instrumentalities
charges subject to such guidelines and limitations as the Congress
may provide, consistent with the basic policy of local autonomy. 3. Common Revenue-Raising Powers
Such taxes, fees, and charges shall accrue exclusively to the local  Reasonable fees and charges for services rendered.
 Rates for operation of public utilities owned, operated, and
governments.
maintained by LGU’s.
 Toll fees or charges for the use of any public road, pier, or wharf,
Art X. Sec. 6. Local government units shall have a just share, as waterway, bridge, ferry or telecommunication system funded and
determined by law, in the national taxes which shall be constructed by the LGU concerned.
automatically released to them. o Except those as enumerated by law

Art X. Sec. 7. Local governments shall be entitled to an equitable B. COMMUNITY TAX (Rocky Lecture Note: Poll tax is part of this
share in the proceeds of the utilization and development of the community tax).
national wealth within their respective areas, in the manner
provided by law, including sharing the same with the inhabitants by 1. Who are liable
way of direct benefits.  Natural persons (P5 annual tax + P1 for every P1,000
income, which shall not exceed P5,000)
o Natural persons are liable to pay the community tax
SUMMARY OF LGC PROVISIONS in their place of residence
 Juridical persons (P500 annual community tax) + [which
A. TAXING POWERS OF LOCAL GOVERNMENTS IN GENERAL shall not exceed P10,000]
o Juridical persons are liable to pay the community tax
1. Fundamental Principles (Sec 130) in their place of principal office
 Taxation shall be uniform in each local government unit.
2. Distribution of Proceeds.
 Taxes, fees, charges and other impositions shall:
 BIR shall be the agency tasked with printing the
(a) be equitable and based as far as practicable on the
community tax certificates.
taxpayer's ability to pay;
 Proceeds shall accrue to the general funds of the LGU’s.
(b) be levied and collected only for public purposes; o However, a portion thereof shall accrue to the
(c) not be unjust, excessive, oppressive, or confiscatory; and national government to cover the actual cost of
(d) not be contrary to law, public policy, national economic printing and distribution of the forms and other
policy, or in the restraint of trade. related expenses.
 Collection shall in no case be let to any private person,  The proceeds of the community tax actually and directly
collected by the city or municipal treasurer shall accrue
 Revenue collected shall inure solely to the benefit of, and be
entirely to the general fund of the city or municipality
subject to the disposition by, the local government unit levying concerned.
the imposition unless otherwise specifically provided herein, o If collected through the barangay treasurers,
 Each local government unit shall, as far as practicable, evolve apportioned as follows: 50% to the general fund of
a progressive system of taxation. the city/municipality concerned; 50% shall to the
barangay where the tax is collected.
 The power to impose a tax, fee, or charge or to generate
revenue under this Code shall be exercised by the sanggunian
D. SPECIAL PROVISIONS ON LOCAL GOVERNMENT UNITS
of the concerned LGU through an appropriate ordinance (Sec
1. Provinces
132).  Tax on transfer of ownership of real property
o Tax base is the higher amount between the total
2. Common Limitations consideration involved in the transfer or of the
 Except as provided in the LGC, LGUs may not levy taxes fair market value.
enumerated in Sec. 133. These include:  Tax on Printing and Publication Business
o Income tax (except banks); o Base is gross annual receipts for existing
businesses, and capital investment for new
o Documentary stamp tax;
businesses
o Taxes on acquisitions mortis causa; o Books or reading materials prescribed by DepEd
o Taxes on exported Phil. products; as school texts or references are exempted.
o Customs, fees, and charges (in general)  Franchise Tax

Bautista, Gastanes, Lopez, Sulit Page | 35


C2017 / LAW 154 / Local Government Law / Midterm Reviewer
o Base is gross annual receipts for existing o Rate for banks and other financial institutions
businesses, and capital investment for new  Based on receipts derived from interest,
businesses commissions and discounts from lending
o Exemption: Those provided by law activities, income from financial leasing,
 Tax on Sand Gravel and Other Quarry Resources dividends, rentals on property and profit
o Division of Proceeds: 30% to province; 30% to from exchange or sale of property,
component city/municipality; 40% to barangay insurance premium.
 Rocky Note: eto yung pinag-aagawan ng o Peddlers engaged in the sale of any
mga LGUs. merchandise or article of commerce
 Professional Tax o Businesses subject to the excise, value-added or
o Place of payment: province where he practices percentage tax under the NIRC
his profession or maintains his principal office. o Other businesses not otherwise specified in the
 Payment of the tax in any province entitles LGC, which the sanggunian concerned may
the taxpayer to practice his profession in deem proper to tax
any part of the Philippines without being  Situs of Taxation and Distribution of Proceeds for
subjected to any other tax for his practice. Business Taxes
o Note: Employers must require payment of o For branches of taxpayers of business tax:
professional tax before hiring an employee. 100% to LGU where branch is located
 Amusement Tax  If sale commenced in LGU where no
o Collected from proprietors, lessees, or operators branch exists, LGU where principal
of theaters, cinemas, concert halls, circuses, o Taxpayers with factories, project offices,
boxing stadia, and other places of amusement. plants, and plantations in the pursuit of their
Base: gross receipts from admission fees. business: 30% where principal office is
 For theaters/cinemas, the tax shall first be located; 70% where factory/proj office, etc. is
deducted and withheld by taxpayers and located
paid to the provincial treasurer before the  If plantation is located at a place other
gross receipts are divided between the than where factory is located, the 70%
taxpayers and the distributors of the will be divided 60-40: 60% to location of
cinematographic films. factory; 40% to location of plantation
o Exemption: operas, concerts, dramas, recitals,  If more than 2 factories/plantations/etc.,
painting and art exhibitions, flower shows, 70% will be divided pro rata among the
musical programs, literary and oratorical localities in proportion to their respective
presentations, except pop, rock, or similar volumes of production during the taxable
concerts period.
o Division of proceeds: Shared equally by the  Other fees and charges that may be imposed by the
province and the municipality municipal govenment
 Tax for Every Delivery Truck/Van of Manufacturers o On any business/occupation on the practice of
or Producers, Wholesalers of, Dealers, or Retailers any profession or calling,
in, Certain Products.  Except those reserved to the province under
o Products covered: distilled spirits, fermented Sec 139
liquors, soft drinks, cigars and cigarettes, and o Fees for Sealing and Licensing of Weights and
other products as determined by the Measures
sangguniang panlalawigan o Fishery Rentals, Fees, and Charges – exclusive
o The taxpayers herein shall be exempt from the with municipalities. These may be imposed on:
tax on peddlers.  The grant of fishery privileges to erect
aquatic beds or bangus fry areas;
2. Municipalities (Note: Municipalities in Metro Manila may  The grant of the privilege to gather, take or
levy taxes at rates which shall not exceed by 50% of the catch fisheries using certain means;
maximum rates prescribed by the Local Government Code).  The issuance of licenses for the operation of
fishing vessels.
 Tax on Business
o Manufacturers, assemblers, repackers,
processors, brewers, distillers, rectifiers, and 3. Cities
compounders of liquors, distilled spirits, and  Cities may levy the taxes, fees, and charges which
wines or manufacturers of any article of provinces or municipalities may impose:
commerce of whatever kind or nature; o Exception: otherwise provided by the LGC
o Wholesalers, distributors, or dealers in any  Taxes, fees and charges levied and collected by highly
article of commerce of whatever kind or nature: urbanized and independent component cities shall
o Exporters, and on manufacturers, millers, accrue to them and distributed in accordance with the
producers, wholesalers, distributors, dealers or provisions of the LGC.
retailers of essential commodities enumerated  Rates may exceed the maximum rates allowed for the
hereunder (rice and corn, wheat or cassava four, province or municipality by not more than fifty
cooking oil, laundry soap, agricultural percent (50%) except the rates of professional and
implements, feeds, school supplies, cement, amusement taxes.
etc.):
o Retailers 4. Barangays
o Contractors and other independent contractors

Bautista, Gastanes, Lopez, Sulit Page | 36


C2017 / LAW 154 / Local Government Law / Midterm Reviewer
 Business taxes for business establishments with  The taxpayer requests for a reinvestigation and
gross sales of receipts of the preceding calendar year executes a waiver in writing before expiration of the
of P50,000.00 or less, for cities and P30,000.00 or period within which to assess or collect; and
less, for municipalities.  The taxpayer is out of the country or otherwise cannot
 The following may be levied upon by barangays as be located.
long as the charges and fees are reasonable. 3. Taxpayer can protest deficiency, surcharges, or other
o Services rendered in connection with the defective assessment within 60 days from receipt of notice
regulations or the use of barangay-owned of assessment.
properties or service facilities such as palay, 4. A written claim for refund or credit needs to be filed with
copra, or tobacco dryers. the local treasurer for a case to be maintained in court for
o Commercial breeding of fighting cocks, the recovery of erroneously collected tax.
cockfights and cockpits;  No case or proceeding shall be entertained in any
o Places of recreation which charge admission court after the expiration of two (2) years from the
fees; date of the payment of such tax or from the date the
o Billboards, signboards, neon signs, and outdoor taxpayer is entitled to a refund or credit.
advertisements.
o Barangay clearance for any business or activity
to be located or conducted within the barangay CASES – none of these were discussed in class
E. CIVIL REMEDIES FOR COLLECTION OF REVENUES (ON THE PART
OF LOCAL GOVERNMENT) ESTANISLAO v. COSTALES
1. Lien (administrative or judicial) Facts: Zamboanga City, thru an Ordinance, imposed a P0.01 tax per
2. Civil Remedies liter of softdrinks produced, manufactured, and/or bottled within
 Distraint of personal property the territorial jurisdiction of the city.
 Levy on real property
 Judicial action. Held: SC held that the Ordinance was null and void as it was not
within the authority of the City to impose the said tax. The authority
F. MISCELLANEOUS PROVISIONS of the City is limited to the imposition of a percentage tax on the
1. SEC 186. – LGU’s have the power to levy taxes not imposed gross sales or receipts of non-essential products at the rate of not
in the LGC/NIRC, provided: exceeding 2% of the gross sales or receipts of the softdrinks for the
 Taxes are not unjust, excessive, oppressive, preceding calendar year. The tax being imposed under said
confiscatory, or contrary to national policy.
Ordinance is based on the output or production and not on the gross
 The ordinance levying the taxes shall not be enacted
sales or receipts as authorized under the Local Tax Code.
without public hearing conducted for the purpose.
2. Any question as to the validity of the tax ordinance may be
raised on appeal to the DOJ Secretary within 30 days from PPC v. MUNICIPALITY OF PILILLA
the effectivity thereof. DOJ Sec shall render a decision Facts: PPC owned and maintained an oil refinery and 49 storage
within 60 days from date of receipt of the appeal
3. Tax ordinances shall be published in full for 3 consecutive tanks for its petroleum products in Pililla, Rizal. The Municipality of
days in a newspaper of local circulation Pililla enacted Municipal Tax Ordinance No. 1 (Pililla Tax Code of
4. Copies of tax ordinances shall be furnished to the 1974), and then filed a complaint against PPC for the collection of
respective local treasurers for public dissemination. the business tax from 1979 to 1986; storage permit fees from 1975 to
5. LGUs shall have the authority to adjust the tax rates as 1986; mayor's permit and sanitary inspection fees from 1975 to 1984.
prescribed herein not oftener than once every five years.
But adjustments must never exceed 10% of rates under the
Held: SC upheld the validity of the Municipal Tax Ordinance,
LGC.
6. Unless otherwise provided in the LGC, tax exemptions or especially considering that the provisions on business taxes were
incentives granted to, or presently enjoyed by all persons, literal reproductions of business tax provisions under the Local Tax
whether natural or juridical, including government-owned Code. However, PPC is not liable for storage fees, since the tanks
or controlled corporations, except local water districts, used were owned by PPC, and not by the Municipality. Thus, it is not
cooperatives duly registered under R.A. No. 6938, non- a charge for any service rendered by the municipality. Finally, the SC
stock and non-profit hospitals and educational held that the Municipality can only collect taxes due from 1976,
institutions, are hereby withdrawn upon the effectivity of
following Art. 1143 of the Civil Code (10 year prescription for actions
this Code.
upon an obligation created by law).
G. TAXPAYER’S REMEDIES
1. Local taxes shall be assessed within 5 years from the date Floro Cement v. Judge Gorospe
they become due, and then collected within 5 years from Facts: Floro Cement, a business engaged in manufacturing and
date of assessment. exporting cement, was taxed by the Municipality of Lugait for
2. Prescription periods provided in the preceding paragraphs
manufacturer’s and exporter’s tax pursuant to two Municipal
shall be suspended when:
 The treasurer is legally prevented from making the Ordinances. They claimed tax exemption based on PD 231 and PD
assessment of collection; 463 granting exemptions to mining products.

Bautista, Gastanes, Lopez, Sulit Page | 37


C2017 / LAW 154 / Local Government Law / Midterm Reviewer
Held: Floro Cement is not exempt. Cement is not a mining product MCIAA v. Marcos (2015)
because it is not cement itself that is mined but the ingredients Facts: Following the 1997 ruling above, the City of Lapu-Lapu issued
thereof. For tax ordinances, exemptions are strictly construed Statements of Real Estate Tax to the MCIAA, assessing the lots
against the person seeking to be exempted. comprising the Mactan International Airport. DOJ Secretary issued
Opinion No. 50, holding that MCIAA’s properties used for airport
TUZON v. CA purposes are owned by the State and are merely held in trust by
Facts: Sangguniang Bayan of Camalaniugan (Cagayan) issued MCIAA. Thus, MCIAA sought to declare that the airport properties
Resolution 9 (R9), obligating the municipal treasurer to enter into are exempt from real estate taxes. CA held that MCIAA is a GOCC
agreements with thresher operators for the them to donate 1% of the and is liable for real property tax.
palay threshed by them as a requirement for application for a
municipal permit. Jurado applied for a permit, but was deined by the Held: SC reverses CA using the 2006 MIAA Decision as basis for
mayor and municipal treasurer for failing to comply with R9. Jurado holding that MCIAA is an instrumentality (not a GOCC). Thus, its
challenges validity of R9. properties actually, solely and exclusively used for public purposes
are not subject to real estate taxes.
Held: R9 is valid, since its validity was no longer challenged as an
issue before the SC. However, SC held that the obligation involved COCA-COLA BOTTLERS PHILS. v. CITY OF MANILA
cannot be a donation because donations are liberal, and not Facts: The City of Manila enacted Tax Ordinance 7988 which
obligatory. And if it is to be considered if it is to be considered a tax removed exemptions for certain taxes on businesses including Coca-
ordinance, then it must have shown to have been enacted in Cola. DOJ and BLGF issued Resolutions declaring Ord. 7988 null
accordance with the requirements of the Local Tax Code, such as a and void for failure to comply with the publication requirement.
public hearing the approval by the Secretary of Finance, and the However the City continued assessing business taxes based on the
requisites for publication of ordinances in general. Tax Ordinance. Coca-Cola filed an injunction suit against its
implementation. Pending this, Ord. 8011 was enacted to amend
DRILON v. LIM 7988.
Facts: The City of Manila passed the Manila Revenue Code through
an ordinance. Four oil companies appealed to DOJ Secretary under Held: SC held that Ord. 7988 was null & void as per the DOJ
LGC Sec 187, which provides that questions on constitutionality or Resolution and BLGF Order. The publication requirement for tax
legality of tax ordinances may be appealed to the DOJ Sec. DOJ Sec ordinances is mandatory for this is the only safeguard against any
then declared the ordinance null and void for non-compliance with unjust and unreasonable exercise of the taxing powers by ensuring
the requirements of the procedure in enacting tax ordinances, and that the taxpayers are notified through publication, and are
for containing provisions contrary to law and public policy. RTC then therefore able to voice out their views or objections to the said
reversed DOJ Sec and held that Sec 187 was void for being an measure. Ordinance 8011 cannot cure Ordinance 7988. It was also
exercise of the power of control, thus violative of local autonomy and null & void for the law it sought to amend did not legally exist. If an
the supervisory power of the President. order or law sought to be amended is invalid, then it does not legally
exist. Instead of amending, another tax measure should be enacted
Held: Sec. 187 is valid, as it merely embodies supervision. Under this which strictly complies with the requirements of law, both
power, the supervisor merely sees to it that the rules are follow, but procedural and substantive.
he himself does not lay down such rules, nor does he have the power
to modify or replace them. However, despite the validity of Sec 187, PETRON CORP v. TIANGCO (MAYOR OF NAVOTAS)
the tax ordinance was held valid for complying with the procedural Facts: Petron was assessed business taxes by the City of Navotas for
requirements. While the requirement that the ordinance be posted its depot or bulk plant engaged in selling of diesel fuels to vessels.
was not met, due process was nonetheless satisfied by the Petron filed a complaint against the assessment made, but the RTC
publication in newspapers of general ccirculation. ruled in favor of Navotas. Mayor Tiangco then issued a Closure Order
against Petron for its failure to pay the taxes. Petron filed a petition
MCIAA v. Marcos (1996) for review with the SC. Respondents argue that Sec. 133(h) merely
Facts: MCIAA was assessed real property taxes by the City of Cebu. prohibits LGUs from imposing direct or excises taxes on petroleum
MCIAA contends that under its charter, it is tax exempt. Also, that as products, thus business taxes may be imposed.
a GOCC, it is in the same footing as agency or instrumentality of
government – which is exempt from real property taxes. Held: Assessment is invalid. The two clauses under Sec. 133(h) must
refer to two different prohibitions. The first one prohibits excise taxes
Held: MCIAA is liable to pay real property tax. A close reading of the or taxes on specific articles, and the second prohibits all kinds of
LGC provisions show that the exemptions previously granted to taxes on petroleum products. The second was deemed to include
MCIAA has been withdrawn and that no other exemption was business taxes. Therefore, the assessment made was void. Pursuant
granted to it. to Sec. 133(h) LGC, all sorts of taxes on petroleum products,
including business taxes, are prohibited from being imposed by local
government units.

Bautista, Gastanes, Lopez, Sulit Page | 38


C2017 / LAW 154 / Local Government Law / Midterm Reviewer
MIAA v. CA (2006) supply and distribution of water and/or generation
Facts: The MIAA (corp that manages NAIA) is being assessed real and transmission of electric power.
 Local assessors shall make a classification, appraisal and
property taxes by the Paranaque City. MIAA paid for a portion of the
assessment of real property when:
taxes. Paranaque then threatened to sell at public auction the said o real property is declared and listed for taxation
properties for MIAA’s failure to pay the RPT. purposes for the first time;
o there is an ongoing general revision of property
classification and assessment; or
Held: As a government instrumentality, MIAA is not a taxable o a request is made by the person in whose name the
person. As a general rule, local governments have no power to tax property is declared, the provincial, city or municipal
assessor or his duly authorized deputy shall, in
the national government, its agencies and instrumentalities.
accordance with the provisions of this Chapter, make a
Exception: if otherwise provided in the LGC. However, the portions classification, appraisal and assessment or taxpayer's
of MIAA property that have been leased out to private entities are valuation thereon:
not exempt from RPT. Real property owned by the Republic,  Provided, That the assessment of real property
whether titled in the name of the national government, its agencies shall not be increased oftener than once every 3
or instrumentalities is subject to real estate tax if the beneficial use years except in case of new improvements
substantially increasing the value of said property
of such property is given to a taxable entity.
or of any change in its actual use.
 Real property assessed for the first time shall be liable for
2. REAL PROPERTY TAXATION back taxes.

C. ASSESSMENT APPEALS
A. GENERAL PROVISIONS
1. Fundamental Principles  Appeal must be made by person having legal interest in the
property within 60 days from the date of receipt of the written
 Real property shall be appraised at its current and fair
notice of assessment before the Board of Assessment Appeals
market value.
(BAA).
 Real property shall be classified for assessment o BAA must decide the appeal within 120 days from receipt
purposes on the basis of its actual use. of such appeal, deciding the case based on substantial
 Assessment shall be based on a uniform classification evidence. A person not satisfied with BAA decision can
within each local government unit. appeal to Central Board of Assessment Appeals (CBAA)
 Appraisal, assessment, levy and collection shall not within 30 days after receipt of the BAA decision of the
be let to any private person. BAA. CBAA decision is final and executory.
 The appraisal and assessment of real property shall  Appeals do not suspend collection of realty taxes on the
be equitable. property involved.
 The provinces and cities (and municipalities within
Metro Manila) shall be primarily responsible for the D. IMPOSITION OF REAL PROPERTY TAX
proper, efficient, and effective administration of the
 Province/City may levy ad valorem tax on real property, with a
real property tax.
uniform rate.
 The following are exempted from payment of the real property
B. APPRAISAL AND ASSESSMENT: NOTE: Purpose of these
tax:
provisions is to determine the tax base. o Real property owned by the Republic or any of its
 Duty of persons owning or administering real property, as political subdivisions except when the beneficial use
well as persons acquiring any real property to prepare a thereof has been granted, for consideration or
sworn statement declaring the true value of their property. otherwise, to a taxable person;
 Duty of local assessor to make such declarations if o Charitable institutions, churches, parsonages or
owner/acquirer defaults in his duty. convents appurtenant thereto, mosques, non-profit or
 Claiming tax exemptions is done through filing with the religious cemeteries and all lands, buildings, and
local assessor documentary evidence of such exemptions improvements actually, directly, and exclusively used
within 30 days from date of declaration. for religious, charitable or educational purposes;
 Local assessors must prepare a schedule of fair market o All machineries and equipment that are actually,
values of real properties in their respective local directly and exclusively used by local water districts and
government units. But it is the sangguniang concerned that government owned or controlled corporations engaged
will enact by ordinance the efficacy of such schedule. The in the supply and distribution of water and/or
schedule must be published in a newspaper of general generation and transmission of electric power;
circulation. o All real property owned by duly registered cooperatives
o Amendment thereof may be done by recommendation as provided for under R.A. No. 6938; and
of the local assessors to the sanggunian. o Machinery and equipment used for pollution control
 Real property classified: residential, agricultural, and environmental protection.
commercial, industrial, mineral, timberland or special.  Any exemption from real property tax previously granted to, or
o Special classes of property are those actually, directly presently enjoyed by, all persons, including all GOCC’s were
and exclusively used for hospitals, cultural, or scientific withdrawn upon the effectivity of the Local Government Code.
purposes, and those owned and used by local water
districts, and government-owned or controlled E. SPECIAL LEVIES ON REAL PROPERTY: A province/city may levy
corporations rendering essential public services in the the following taxes for ff. purposes:

Bautista, Gastanes, Lopez, Sulit Page | 39


C2017 / LAW 154 / Local Government Law / Midterm Reviewer
 1% tax on assessed value (AV) of real property to accrue to the o The owner of the property or the person having legal
Special Education Fund (SEF). interest therein requests for reinvestigation and executes
 Tax not exceeding 5% on AV of real property on idle lands (in a waiver in writing before the expiration of the period
addition to basic real property tax). within which to collect; and
 Special levy on lands specially benefitted by public works not o The owner or person having legal interest therein is out
exceeding 60% of actual cost of the works of the country or cannot be located.
o Ordinances imposing a special levy shall describe the
nature, extent, and location of the public woks to be G. DISPOSITION OF PROCEEDS (includes interest, proceeds from
undertaken, the estimated cost, the metes and bounds, use/saleredemption/etc.
and the number of annual installments for the payment  Provinces: 35% to province; 40% to municipality; 25% to
of the special levy (5-10 years only). barangay
o These ordinances must have a prior public hearing, and  Cities: 70% to city; 30% to component brgys [50% to brgy
notice to owners of the affected real property or persons where located, 50% equally to comp brgys]
having legal interest therein as to the date of hearing.
 SEF: automatic release to the local school boards [if province,
o Special levy shall be computed according to the assessed
divide equally bet. prov. and mun. boards]
valuation of the lands affected.
 Idle lands: general fund of LGU where province/city is located
o The special levy shall accrue on the first day of the quarter
after the effectivity of its ordinance.\  Special levy: general fund of LGU which financed such
o Remedies of taxpayer is governed by provisions on works/projects
Assessment Appeals.
H. SPECIAL PROVISIONS
F. COLLECTION OF REAL PROPERTY TAX  Sanggunian shall provide appropriations to defray expenses
 RPT accrues on Jan 1 and constitutes a lien on the property to real property assessments.
from that date. Such lien is superior to any other lien and shall  Sanggunian may pass ordinance (before Jan 1)
only be extinguished upon payment of the delinquent tax. condoning/reducing RPT in case of general failure of crops or
 Treasurer shall post the notice of dates when tax may be paid substantial decrease in price of agricultural products, or
without interest in a conspicuous and publicly accessible place calamity in the LGU concerned.
at the city/municipal hall and publish the same in a  Philippine President may condone/reduce RPT when public
newspaper of general circulation once a week for two weeks. interest requires.
For basic RPT and SEF – must be done on or before Jan 31.  Certificates/documents/papers covering the sale of
 Basic RPT and SEF may be paid without interest in 4 equal delinquent property are exempt from DST/reg fees.
installments (Mar 31, Jun 30, Sep 30, Dec 31)  Assessment notices or tax declaration copies sent through
 Tax discount not exceeding 20% of annual tax due may be mails are exempt from postal fees.
granted to advanced prompt payment of basic RPT and SEF.  Tax delinquencies incurred before the LGC shall be governed
 Protests may only be entertained after payment of the tax. by laws in force then.
Payments under protest shall be annotated with “paid under
protest” on the tax receipts. Protest must be made within 60 Secretary of Finance v. Hon. Ilarde
days from receipt to treasurer. Facts: RD Cabaluna failed to pay land taxes on certain parcels of
o Tax paid shall be held in trust by the treasurer concerned.
land he owned. City Treasurer of Ilo-ilo charged him with penalties
o If protest ruled in favor of taxpayer, the tax paid shall be
applied as tax credit. amounting to more than 24% of the delinquency tax, in
o If against his favor, remedies before Assessment Appeals contravention of Sec. 66 of the Real Property Tax Code, which
may be resorted to. provided that in no case shall the penalty exceed 24% of the
 Notice of delinquency must be posted and published as well, delinquent tax. City Treasurer cited DOF Regulations which provide
and must specify date of delinquency and the fact that that the penalty shall continue to be imposed until it is fully paid in
personal property may be distrained to effect payment. full.
 Interest for unpaid RPT is 2% per month, but the total shall
never exceed 36 months.
Held: The Regulations are null and void for being repugnant to Sec.
 Remedies for collection include administrative and judicial
66. The implementing rules cannot add to or detract from the
actions, including:
o Local governments lien on the property subject to tax; provisions of the law it is designed to implement. From 1 January
o Levy on the real property by means of a warrant and the 1992 onward, the proper basis for the computation of the real
institution of a collection suit. Subsequent public sale of property tax payable must be the LGC, which took effect on 1 January
the property should taxes remain unpaid within 2 weeks 1992, inasmuch as Section 534 thereof had expressly repealed the
after 30 days after service. Real Property Tax Code (1974).
o Collection of RPT through judicial action via a civil action
for collection.
 RPT shall be collected within 5 years from the time they BENGUET CORPORATION v. CBAA
become due, except when inability to pay is a result of fraud Facts: Real property tax was assessed against Benguet Corporation
or intent to evade payment of the tax – in which case action on its bunkhouses occupied for residential purposes by its rank and
may be instituted within 10 years from the discovery of fraud file employees. Provincial Assessor says the tax exemption of
or intent to evade.
bunkhouses under Sec. 3(a) of PD 745 was withdrawn by PD 1955
 Suspension of period of prescription when:
o The local treasurer is legally prevented from collecting (and EO 93).
the tax;

Bautista, Gastanes, Lopez, Sulit Page | 40


C2017 / LAW 154 / Local Government Law / Midterm Reviewer
Held: Properties are taxable. While the realty tax exemption was MANILA INTL AIRPORT AUTHORITY v. CITY OF PASAY
indeed withdrawn, Benguet Corp. should have applied for Facts: Issue in this case is WoN the NAIA properties of MIAA are
restoration of the exemption with the FIRB. While LGUs are charged exempt from the real property taxes.
with fixing the rate of real property taxes, it does not necessarily
follow from that authority the determination of whether or not to Held: Airport properties are exempt. MIAA, not being a stock or non-
impose the tax. Local governments have no alternative but to collect stock corporation, is not a GOCC but a government instrumentality.
taxes as mandated in the Real Property Tax Code. It is the national Under Sec 133(o), LGUs have no power to tax instrumentalities of the
government, expressing itself through the legislative branch, that national government like MIAA. Hence, MIAA is not liable to pay real
levies the real property tax. When local governments are required to property tax for the NAIA Pasay properties. Also, The airport lands
fix the rates, they are merely constituted as agents of the national and buildings are properties of public dominion intended for public
government in the enforcement of the Real Property Tax Code. The use, and as such, are exempt from real property tax under Sec 234(a)
delegation of taxing power is not involved here because the national of the LGC. However, under the same provision, if MIAA leases its
government has already imposed realty tax, leaving only the real property to a taxable person, the specific property leased
enforcement to be done by local governments. The realty tax is becomes subject to real property tax. In this case, only those
enforced throughout the Philippines, but the proceeds of the tax portions of the NAIA Pasay properties which are leased to taxable
accrue to the province, city, municipality and barrio where the realty persons like private parties are subject to real property tax by the
taxed is situated. In contrast, a local tax is imposed by the municipal City of Pasay.
or city council by virtue of the Local Tax Code.
3. SHARES OF LGUs IN THE PROCEEDS OF
NDC v. CEBU NATIONAL TAXES
Facts: The President reserved a parcel of land in Cebu and put it
under the administration of NDC. A warehouse was constructed on
Art X. Sec. 6. supra
the land. The City of Cebu sought to tax these properties (land and
Art X. Sec. 7. supra
warehouse) stating that there is no law exempting NDC from real
estate taxes.
SUMMARY OF LGC PROVISIONS
A. ALLOTMENT OF INTERNAL REVENUE
Held: NDC is liable to the City of Cebu for taxes on the warehouse.
The Government never parted with the ownership of the land thus,  LGUs shall have a 40% share in the national internal
revenue taxes based on the collection of the 3rd fiscal year
it is still exempt from tax. However, this exemption does not extend
(FY) preceding the current FY.
to the improvements (warehouse) on the land. o However, if national government incurs an
unmanageable public sector deficit, the President is
Province of Tarlac v ALCANTARA authorized to make necessary adjustments in the IRAs
of LGUs which shall not be less than 30%.
Facts: Complaint by the Provincial Treasurer praying that TEI be  Only upon recommendation of DOF/DILG/DBM
ordered to pay accrued real estate taxes. TEI refused to pay the same and subject to consultation of Congress and Ligas
for the reason that under Sec. 40, paragraph (g) of PD No. 464 in  LGU shares in the IRAs: Provinces 23%, Cities 23%,
relation to P.D. No. 551, as amended, it was exempt from paying said Municipalities 34%, Barangays 20%
tax. TEI contends that the "other laws" referred to in this Section is o Sharing of each province/city/municipality:
PD 551 (Lowering the Cost to Consumers of Electricity by Reducing the population 50%, land area 25%, equal sharing 25%
Franchise Tax Payable by Electric Franchise Holders and the Tariff on o For barangays with at least 100 inhabitants, their
share shall not be less than P80K per annum,
Fuel Oils for the Generation of Electric Power by Public Utilities).
chargeable against the 20% share of the barangay
from the IRA. The balance shall be allocated on a 60-
Held: The enumerated items upon which taxes shall not be imposed, 40 basis for population and equal sharing.
have no relation at all to, and are entirely different from, real  Sec 286: (a) The share of each LGU shall be released,
properties subject to tax. The provisions deal with franchise tax and without need of any further action, directly to the provincial,
tariff on fuel oils and the "earnings, receipts, income and privilege of city, municipal or barangay treasurer, as the case may be,
generation, distribution and sale of electric current" are the items on a quarterly basis within five (5) days after the end of each
quarter, and which shall not be subject to any lien or
exempted from taxation by the imposition of said tax or tariff duty.
holdback that may be imposed by the national government
On the other hand, the collection complaint filed by petitioner for whatever purpose.
specified only taxes due on real properties. It has always been the  LGU’s must appropriate at least 20% of their annual IRA for
rule that "exemptions from taxation are construed in strictissimi juris development projects
against the taxpayer and liberally in favor of the taxing authority"
primarily because "taxes are the lifeblood of government and their B. SHARE OF LGUS IN THE NATIONAL WEALTH
prompt and certain availability is an imperious need.” Thus, to be  LGUs shall have equitable share n the proceeds derived
exempted from payment of taxes, it is the taxpayer's duty to justify from the utilization and development of the national wealth
the exemption "by words too plain to be mistaken and too within their respective areas.
categorical to be misinterpreted.".  In addition to the 40% IRA, LGUS shall also have a 40%
share of the gross collection derived by the national
government from mining taxes, royalties, forestry and

Bautista, Gastanes, Lopez, Sulit Page | 41


C2017 / LAW 154 / Local Government Law / Midterm Reviewer
fishery charges, and such other taxes, fees, or charges, requirements to the DILG for appraisal before the Committee serves
including related surcharges, interests, or fines, and from notice to the DBM for release of the corresponding funds. Gov.
its share in any co-production, joint venture or production Mandanas of Batangas petitioned to declare the resolutions and
sharing agreement in the utilization and development of
certain provisos in the General Appropriations Acts (GAAs) of 1999,
the national wealth within their territorial jurisdiction.
2000, and 2001 as unconstitutional for uniformly earmarking for
 LGUs shall have a share in he proceeds derived by any
government agency or GOCC engaged in the utilization and each corresponding year P5 billion for the IRA for the LGSEF and
development of the national wealth in the following imposing conditions for the funds’ release.
manner:
o Natural resources in the province: Province 20%, Held: The resolutions and the GAA provisos are unconstitutional.
Component city/municipality 45%, barangay 35% They are, in effect, a “withholding” of a portion of the IRA. The
o Natural resources in highly urbanized or independent LGSEF is part of the IRA or “just share” of the LGUs in the national
component cities: City 65%, barangay 35%
taxes, which shall be automatically released to LGUs. LGUs are not
 If resources located in 2 or more LGU’s, their
above share will be computed based on supposed to be required to perform any act to receive the “just
population 70% and land area 50% share;” it shall be released “without need of further action“. The only
 At least 80% of the proceeds derived from taxing sources of exception to the mandatory automatic release of the LGUs IRA is if
energy shall be applied solely to lower the cost of electricity the national internal revenue collections for the current fiscal year is
in the LGU where such source of energy is located. less than 40% of the collections of the 3rd preceding fiscal year. The
exception does not apply here.
PIMENTEL v. AGUIRRE (supra)
Facts: President issued AO 372, Sec 4 of which provided for
ACCORD v. ZAMORA
withholding of IRAs of LGUs.
Facts: Pres Erap signed into law the GAA of 2000. There is a
provision in this law that withholds P10B of the IRA (named as
Held: Section 4 is invalid. A basic feature of local fiscal autonomy is
“unprogrammed funds”), which will only be released upon
the automatic release (5 days every after quarter of the year) of the
assessment of committee enumerated in the law. Different groups
shares of LGUs in the national internal revenue and is mandated by
questioned the constitutionality of the said law
no less than the Constitution (Art X Sec 6). The withholding, even if
temporary, clearly contravenes the Constitution and the law. Any
Held: Those provisions are unconstitutional. They contravene the
form of retention of the IRA is prohibited.
constitutional mandate to automatically release funds to the LGUs.
The only exception to the automatic release is found in LGC 284 (see
SAMPIANO v. INDAR last 2 sentences of Province of Batangas summary)
Facts: Ogka filed a court action to temporarily suspend the release
by the PNB-Marawi of the October 2004 IRA of the Municipality of
4. CREDIT FINANCING, LOANS, CREDITS AND
Balabagan, while the results of the mayoralty elections were under
contest. The Judge ruled in Ogka’s favor. Sampiano filed an OTHER FORMS OF INDEBTEDNESS OF LGUS
administrative complaint against Indar due to the latter’s order (and
failure to observe procedure in the issuance of the Order).  Any LGU may create indebtedness, and avail of credit
facilities to finance local infrastructure and other socio-
Held: Indar is administratively liable for causing the suspension of economic development projects in accordance with the
approved local development plan and public investment
the release of the IRA’s without proper hearing. The automatic
program. An LGU may avail of credit lines from government
release of the IRA under Section 286 is a mandate to the national or private banks and lending institutions for the purpose of
government through the DBM to effect automatic release of the said stabilizing local finances.
funds from the treasury directly to the LGU, free from any holdbacks  The LGC enumerates certain conditions and procedures for
or liens imposed by the national government. However, this each type of indebtedness/transaction.
automatic release of the IRA from the national treasury does not  LGUs shall appropriate in their respective annual budgets
prevent the proper court from deferring or suspending the release such amounts as are sufficient to pay the loans and other
thereof to particular local officials when there is a legal question indebtedness incurred or redeem or retire bonds,
debentures, securities, notes and other obligations issued
presented in the court pertaining to the rights of the parties to
under this Title.
receive the IRA or to the propriety of the issuance of a TRO or a o Failure to provide the appropriations herein required
preliminary injunction while such rights are still being determined. shall render their annual budgets inoperative.

PROVINCE OF BATANGAS v. ROMULO 5. LOCAL FISCAL ADMINSTRATION


Facts: Pres. Estrada issued EO No. 48 establishing the “Program for
Devolution Adjustment and Equalization” to enhance the capabilities
of LGUs in the discharge of the functions devolved to them. The
Oversight Committee passed 3 Resolutions, with guidelines
requiring LGUs to identify projects eligible for funding under the
portion of LGSEF and submit project proposals and other

Bautista, Gastanes, Lopez, Sulit Page | 42


C2017 / LAW 154 / Local Government Law / Midterm Reviewer

Bautista, Gastanes, Lopez, Sulit Page | 43


C2017 / LAW 154 / Local Government Law / Midterm Reviewer

VIII. Local Government Units

LGU: Barangay Municipality City Province


Role Sec 384 Sec 440 (M); Sec 448 (C) Sec 459
Primary planning and Coordination and delivery of basic, regular and direct Dynamic mechanism for
implementing unit of services and effective governance of the inhabitants within developmental processes
government policies, plans, its territorial jurisdiction. and effective governance
programs, projects, and activities of local government units
in the community, and as a within its territorial
forum, and where disputes may jurisdiction.
be amicably settled.
Manner of Sec. 385 Sec 441 (M); Sec 449 (C); Sec 460 (P)
creation By law or by ordinance Only by an Act of Congress and subject to the approval by a majority of the votes cast in a
plebiscite
Requisites for Sec. 386 Sec. 442 Sec. 450 Sec. 461
Creation Population: at least 2,000 Average annual income: Average annual income: at Average annual income:
inhabitants At least P2,500,000.00 for least P20,000,000.00 for not less than
EXCEPT: Cities and the last two consecutive the last two consecutive P20,000,000.00 and
municipalities within Metro years; years; and either of the either of the following
Manila and other metropolitan Population: at least 25,000 following requisites: requisites:
political subdivisions or in highly inhabitants; (i) Contiguous territory: at (i) Contiguous territory: at
urbanized cities where such Contiguous territory: at least least 100 sq km; or least 2,000 sq km; or
territory shall have a certified 50 sq km (ii) Population: not less than (ii) Population: not less
population of at least 5,000 150,000 inhabitants than 250,000 inhabitants
inhabitants *Municipalities existing as of
the date of the effectivity of
Provided, That the creation this Code shall continue to
thereof shall not reduce the exist and operate as such.
population of the original The territorial jurisdiction of a newly-created unit shall be
barangay or barangays to less properly identified by metes and bounds. The requirement
than the minimum requirement on land area shall not apply where the unit proposed to be
prescribed herein created is composed of one (1) or more islands. The territory
need not be contiguous if it comprises two (2) or more
islands.

The average annual income shall include the income accruing to the general fund of the
unit concerned, exclusive of special funds, transfers and non-recurring income.

Provided, That, the creation shall not reduce the land area, population, and income of the
original unit or units at the time of said creation to less than the minimum requirements.

Chief Officials Sec 387 Sec 443 Sec 454 Sec 463
a) Punong Barangay a) Sangguniang Bayan a) Sangguniang a) Governor
b) 7 Sangguniang Barangay members Panlungsod members b) Vice-governor
members, c) members of the
c) Sangguniang Kabataan Sangguniang
chairman, Panlalawigan
d) Barangay Secretary,
e) Barangay treasurer Administrator; Legal officer; Veterinarian
f) Lupong Tagapamayapa Social welfare and development officer; General services
officer

Bautista, Gastanes, Lopez, Sulit Page | 44


C2017 / LAW 154 / Local Government Law / Midterm Reviewer
Mayor; Vice-mayor; Secretary; Treasurer
Assessor; Accountant; Budget officer
Planning and development coordinator; Engineer/building official
Health officer; Civil registrar.

Chief Punong Barangay Municipal Mayor City Mayor Provincial Governor


Executive Sec 389 Sec 444 Sec 455 Sec 465
Powers, (a) Enforce all laws and a) Exercise general supervision and control over all a) Exercise general
Duties, and ordinances programs, projects, services, and activities supervision and control
Functions (b) Negotiate, enter into, and sign b) Enforce all laws and ordinances and the exercise of its over all programs,
contracts for and in behalf of the corporate powers projects, services, and
Barangay c) Initiate and maximize the generation of resources and activities of the provincial
(c) Maintain public order and revenues, and apply the same to the implementation of government
assist the city or municipal mayor development plans, program objectives and priorities b) Enforce all laws and
and sanggunian members in the d) Ensure the delivery of basic services and the provision of ordinances relative to the
performance of their duties adequate facilities governance of the
(d) Call and preside over the e) Exercise such other powers and perform such other duties province and the exercise
sessions of the Sangguniang and functions as may be prescribed by law or ordinance. of the appropriate
Barangay and the Barangay corporate powers
assembly, and vote to break a tie; c) Initiate and maximize
(e) Appoint or replace the the generation of
appointive Barangay officials; resources and revenues,
(f) Organize and lead an and apply the same to the
emergency group whenever implementation of
necessary development plans,
(g) Prepare the annual executive program objectives and
and supplemental budgets priorities
(h) Approve vouchers for d) Ensure the delivery of
disbursement of funds; basic services and the
(i) Enforce laws relating to provision of adequate
pollution control and protection facilities
of environment; e) Exercise such other
(j) Administer the operation of the powers and perform such
Katarungang Pambarangay other duties
(k) Exercise general supervision and functions as may be
over the activities of SK; prescribed by law or
(l) Ensure the delivery of basic ordinance.
services;
(m) Conduct an annual palarong
Barangay;
(n) Promote the general welfare
(o) Exercise such other powers
and perform such other duties
and functions as may be
prescribed by law or ordinance
Sec 389

*see above for Legislative Bodies and their Powers, Duties, and
Functions

Bautista, Gastanes, Lopez, Sulit Page | 45


C2017 / LAW 154 / Local Government Law / Midterm Reviewer
A. The Barangay  Any member who comes to the aid of persons in authority
 All are persons in authority
SUMMARY OF LGC PROVISIONS
Chapter I – Role and Creation of the Barangay – 384 – 386 Chapter III – The Punong Barangay – 389
 384: Role of the Barangay  389: Chief Executive: Powers, Duties, and Functions
o Basic political unit o Punong Barangay is the chief executive of the barangay
o Primary planning and implementing unit of government o Exercises such powers and perform such duties and functions
policies, plans, etc. as provided by the LGC and other laws
o Forum where people’s views may be expressed o Provision lists 15 powers granted to the Punong Barangay
o Where disputes may be settled pursuant to the general welfare clause (LGC 16)
o Entitled to possess and carry a necessary firearm
 385: Manner of Creation
 Within his territorial jurisdiction
o Created, divided, merged, abolished, or its borders
 In the performance of his peace and order functions
substantially altered
 Subject to appropriate rules and regulations
 By law
 By ordinance of an Sangguniang Panlalawigan or
Panlungsod Chapter IV – The Sangguniang Barangay – 390 – 393
 Subject to plebiscite  390: Composition
 If by S-Panlalawigan, recommendation of S-Bayan o S-Barangay is the legislative body of the barangay
necessary o Composed of:
o Punong Barangay, as presiding officer
 386: Requisites for Creation
o 7 S-Barangay members
o Population: 2000
 EXC: In cities and municipalities of MM: 5000  Elected at large
 Certified by NSO o SK chairman
 Shall not reduce the population of the original barangay  391: Powers, Duties, and Functions
to less than the minimum o Provisions lists 24 itmes
o Territorial jurisdiction properly identified by metes and  392: Other Duties of Sangguniang Barangay Members
bounds or by more or less permanent natural boundaries o Assist the punong barangay
 EXC: If made of two or more islands, territory need not be o Act as peace officers
contiguous o Perform duties and function as the punong barangay may
o Governor or City mayor may prepare a consolidation plan delegate
within its territorial jurisdiction  393: Benefits of Barangay Officials
 To be submitted to the S-Panlalawigan or S-Panlungsod o Honoraria, allowances, and such other emoluments as may
for appropriate action be authorized by law or ordinance
 For municipalities within MM and other metropolitan  Max P1000/month for Punong Barangay
political subdivisions, the plan shall be prepared and  Max P600/month for Members, Treasurer, and Secretary
approved by the S-Bayan  Subject to budget limitations
o Christmas bonus: P1000
Chapter II – Barangay Officials and Offices – 387 – 388 o Insurance coverage
o Free medical care
 387: Chief Officials and Offices
 In any government hospital or institution
o 7 Sangguniang barangay members
 EXC: In cases of extreme urgency, may go to private and
o SK Chairman
expenses shall be charged against the barangay funds
o Barangay Secretary
(max of P5000)
o Barangay Treasurer
o Exempted from tuition and matriculation fees for legitimate
o Lupong Tagapamayapa
dependent children attending state colleges or universities
o S-Barangay may form community brigades and create such
o Entitled to appropriate civil service eligibility
other positions or offices as may be deemed necessary
o (Elective Barangay Officials) Preference in appointments to
 In accordance with the needs of public service
any gov’t position or in any GOCC after their tenure in office
 Subject to budget limitations
o (Barangay tanod brigades [Max 20]) entitled to insurance
 388: Persons in Authority
o For RPC purpose: Punong Barangay, Sangguniang barangay
members, and members of the lupong tagapamayapa are Chapter V – Appointive Barangay Officials – 394 – 396
deemed persons in authority in their jurisdictions  394: Barangay Secretary: Appointment, Qualifications, Powers
o Others officials and members designated by law or ordinance and Duties
and charged with o Appointed by the Punong Barangay
 The maintenance of public order  With the concurrence of the majority of all the S-Barangay
 Protection and security of ife and property members
 Maintenance of desirable and balanced environment  NOT subject to attestation by the Civil Service Commission

Bautista, Gastanes, Lopez, Sulit Page | 46


C2017 / LAW 154 / Local Government Law / Midterm Reviewer
o Qualifications 1. Katarungang Pambarangay
 Legal age
 Qualified voter
 Actual resident of barangay
Chapter VII – Katarungang Pambarangay – 399 – 422
 CANNOT be  399: Lupong Tagapamayapa
o Composition
 S-Barangay member
 Punong barangay (chairman)
 Government employee
 10 to 20 members
 Relative of the punong barangay to the 4th civil degree
o Constituted every 3 years
o Provision lists 8 Powers and Duties under (d)
o Qualifications to be member
 395: Barangay Treasurer: Appointment, Qualifications, Powers  Any person actually residing or working in the barangay
and Duties  Not otherwise disqualified by law
o Appointment and Qualifications same as Barangay Secretary  Possessing integrity, impartiality, independence of mine,
o Treasurer shall be bonded in accordance with existing laws in sense of fairness, and reputation for probity
an amount to be determined by the sangguniang barangay o A notice to constitute the lupon shall be prepared by the
(Max P10,000) punong barangay within 15 days from the start of the term of
 Premiums to be paid by the barangay his office
o Provision lists 8 Powers and Duties under (e)  Posted in 3 conspicuous places for a period of not less than
 396: Other Appointive Officials 3 weeks
o Qualifications, powers, and duties governed by LGC or other o The punong barangay shall, within 10 days, appoint as
laws or by barangay ordinances members those whom he determines to be suitable therefor.
 Made in writing, signed by the punong barangay, and
Chapter VI – Barangay Assembly – 397 – 398 attested to by the barangay secretary
 397: Composition; Meetings o List of appointed members shall be posted in 3 conspicuous
o Composition places for the entire duration of their term in office
 All persons who are actual residents of the barangay o For barangays where the majority of the inhabitants are
 At least 15 years, 6 months of age members of indigenous cultural communities
 Citizen of the Philippines  Local systems of settling disputes shall be recognized
 Duly registered in the list of barangay assembly  Without prejudice to the provisions of the LGC
members  400: Oath and Term of Office
o Meetings o Upon appointment, each lupon member shall take an oath of
 At least twice a year office before the punong barangay
 To hear and discuss the semestral report of the S- o Shall hold office until a new lupon is constituted
Barangay  Unless sooner terminated by
 Held upon call of:  resignation,
 The punong barangay or  transfer of residence or place of work, or
 at least four members of the S-Barangay or  withdrawal of appointment by the punong barangay
 Upon written petition of at least 5% of the assembly with the concurrence of the majority of all the members
members of the lupon
 No meeting shall take place unless a written notice is given  401: Vacancies
1 week prior to meeting o If there are any vacancies for any cause, the punong barangay
 EXC: matters involving public safety or security shall immediately appoint a qualified person
 Presiding Officer (in order of hierarchy)  Holds office for the unexpired term
 Punong Barangay  402: Functions of the Lupon
 The S-Barangay member acting as punong barangay o Exercise administration supervision over the conciliation
 Any assembly member selected during the meeting panels
 If Barangay secretary absent, presiding officer may o Meet regularly once a month to provide a forum for exchange
designated any member to act as secretary of ideas on matters relevant to the amicable settlement of
 398: Powers of the Barangay Assembly disputes; also sharing observations and experiences between
o Initiate legislative processes members
o Decide on the adoption of initiative o Exercise such other powers and perform such other duties
o Hear and pass upon the semestral report of the S-Barangay and functions as may be prescribed by law or ordinance
 403: Secretary of the Lupon
o Barangay secretary concurrently serves as the secretary of the
lupon
o Records the results of mediation proceedings; submits a
report thereon to the city or municipal court
 404: Pangkat ng Tagapagkasundo

Bautista, Gastanes, Lopez, Sulit Page | 47


C2017 / LAW 154 / Local Government Law / Midterm Reviewer
o For each dispute, there shall be a pangkat ng o Objections to venue should be raised in the mediation
tagapagkasundo proceedings; otherwise they are waived
o Composition: 3 members, chosen by the parties to the dispute o Any legal question which may confront the punong barangay
 If the parties fail to agree, then by lots drawn by the lupon in resolving objections to venue may be submitted to the
chairman Secretary of Justice
 405: Vacancies in the Pangkat  410: Procedure for Amicable Settlement
o Any vacancy shall be chosen by the parties to the dispute o Who may initiate proceeding
 Failure to agree = draw lots by chairman  Upon payment of filing fee…
 406: Character of Office and Service of Lupon Members  Any individual who has a cause of action against another
o Lupon members, while in the performance of their official individual
duties, are deemed persons in authority (for RPC)  Involving any matter within the authority of the lupon
o Lupon or pangkat members shall serve without  Complaint may be oral or in writing
compensation EXCEPT as provided in Sec. 393 (Benefits of  To the lupon chairman
Barangay Officials) o Mediation by lupon chairman
 And without prejudice to incentives as provided for in this  Upon receipt of complaint, chairman shall within the next
section and in Book IV, LGC working day
 DILG shall provide for a system of granting economic or  Summon the respondents
other incentives  With notice to the complainants and their witnesses
 407: Legal Advice on Matters Involving Questions of Law  For mediation
o The provincial, city legal officer or prosecutor or the municipal  Failure to mediate in 15 days from first meeting: chairman
legal officer shall render legal advice on matters involving shall set a date for the constitution of the pangkat
questions of law o Suspension of prescriptive period of offenses
 408: Subject Matter for Amicable Settlement; Exception Thereto  While the dispute Is under mediation, conciliation, or
o The lupon shall have the authority to bring together the arbitration, the prescriptive periods for offenses and cause
parties actually residing in the same city or municipality for of action shall be interrupted upon filing the complaint
amicable settlement of all disputes EXCEPT with the punong barangay
(a) Where one party is the government or any subdivision or  Period resumed upon receipt by the complainant of the
instrumentality thereof complaint or the certificate of repudiation or of the
(b) Where one party is a public officer or employee; dispute certification to file action (issued by the lupon or pangkat
relates to the performance of his official functions secretary)
(c) Offenses punishable by imprisonment exceeding one year  Such interruption shall not exceed 60 days from the
or a fine exceeding P5,000 filing of the complaint
(d) Offenses where there is no private offended party o Issuance of summons; hearing; grounds for disqualification
(e) Where the dispute involves real properties located in  The pangkat shall convene not later than 3 days from its
different cities or municipalities; unless the parties agree constitution (for the hearing)
(f) Disputes involving parties who actually reside in barangays  On the day and hour set by the chairman
of different cities or municipalities; except where such  The pangkat may issue summons for the personal
barangays adjoin each other and the parties agree appearance of the parties and witnesses before it.
(g) Such other class of disputes which the President may  Matters of disqualification of the pangkat shall be resolved
determine in the interest of justice or upon by the affirmative vote of the majority of the pangkat
recommendation of the Secretary of Justice o Period to arrive at a settlement
o The court in which non-criminal cases not filling within the  Within 15 days from the day the pangkat convenes;
authority of the lupon under this code are filed may, at any extendible for another period not exceeding 15 days,
time before trial, muto proprio refer the case to the lupon for except in clearly meritorious cases
amicable settlement  411: Form of Settlement
 409: Venue o All amicable settlements shall be in writing
o If persons in dispute are...disputes shall be brought before… o In a language or dialect known to the parties; signed by them
 Residing in the same barangay; lupon of said barangay o Attested by the lupon or pangkat chairman
 Residing in different barangays; barangay where  412: Conciliation
respondent/any of the respondents reside/s, at the o Precondition to filing of complaint in court
election of the complainant  No complaint, petition, or proceeding involving any matter
o If dispute involves real property or any interest therein; within the authority of the lupon shall be filed or instituted
barangay where the real property or a larger portion thereof directly in court or any other government office for
is situated adjudication
o If dispute arose at the workplace where the contending  UNLESS there has been a confrontation between the
parties are employed (or institution where parties are parties before the lupon chairman or the pangkat
studying); barangay where such workplace or institution is
located

Bautista, Gastanes, Lopez, Sulit Page | 48


C2017 / LAW 154 / Local Government Law / Midterm Reviewer
 AND that no conciliation or settlement has been  After the lapse of the period, the settlement may be
reached as certified by the lupon secretary or pangkat enforced by action in the appropriate city or municipal
secretary as attested to by the lupon or pangkat court
chairman or unless the settlement has been repudiated  418: Repudiation
by the parties thereto o Any party to the dispute may, within 10 days from the date of
o Where parties may go directly to court the settlement, repudiate the same
(1) Where the accused is under detention o By filing with the lupon chairman a statement to that effect
(2) Where a person has otherwise been deprived of  Sworn to before him
personal liberty calling for habeas corpus proceedings o Where the consent is vitiated by fraud, violence, or
(3) Where actions are coupled with provisional remedies intimidation
(such as preliminary injunction, attachment, delivery of o Such repudiation shall be sufficient basis for the issuance of
personal property, and support pendente lite) the certification for filing of a complaint
(4) Where the action may otherwise be barred by the  419: Transmittal of Settlement and Arbitration Award to the Court
statute of limitations o The secretary of the lupon shall transmit the settlement or
o Conciliation among members of indigenous cultural the arbitration award to the appropriate city or municipal
communities court within 5 days from the date of the award OR form the
 The customs and traditions of such communities shall be lapse of the 10 day period repudiating the settlement
applied in settling disputes o Shall furnish copies thereof to each of the parties and to the
 413: Arbitration lupon chairman
o The parties may agree that they shall abide by the arbitration  420: Power to Administer Oaths
award of the lupon chairman or the pangkat o The punong bayan, as chairman of the lupong
 At any stage tagapamayapa, and the members of the pangkat are hereby
 Must be in writing authorized to administer oaths in connection with any matter
o May be repudiated within 5 days from the date thereof relating to all proceedings in the implementation of the
o The arbitration award shall be made after the lapse of the katarungang pambarangay
period of repudiation and within 10 days thereof  421: Administration; Rules and Regulations
 The award shall be in writing in a language or dialect o The city or municipal mayor shall see to the efficient and
known to the parties effective implementation and administration of the
 414: Proceedings Open to the Public; Exception katarungang pambarangay
o All proceedings for settlement shall be public and informal o The Secretary of Justice shall promulgate the rules and
 Provided, that the lupon or pangkat chairman may motu regulations necessary to implement this chapter
proprio or upon the request of a party, exclude the public  422: Appropriations
from the proceedings in the interest of privacy, decency, or o Such amount as may be necessary for the effective
public morals implementation of the katarungang pambarangay shall be
 415: Appearance of Parties in Person provided for in the annual budget of the city or municipality
o In all katarungang pambarangay proceedings, the part must concerned
appear in person without the assistance of counsel or
representative Morata v. Go (1983)
 EXC: minors and incompetents; may be assisted by their Summary: Stemming from a recovery of sum of money plus
next of kin who are not lawyers damages the Sps Morata filed an MTD in the CFI on the basis of
 416: Effect of Amicable Settlement and Arbitration Award failure to allege prior availment of the brgy, conciliation and absence
o The amicable settlement and the arbitration award shall of certification of the lupon secretary as required by PD 1508 or the
have the force and effect of a final judgement of a court Katarungang Pambarangay Act. This MTD was denied by the CFI
 Upon the expiration of 10 days from the date thereof Judge Tomol saying that this requirement only applies to cases
 UNLESS repudiated or a petition to nullify the award has cognizable by the inferior courts as per sec. 11,12,and 14 of the PD.
been filed SC dismissed the case for recovery of damages.
o However, this provision shall not apply to court cases settled
by the lupon under the last paragraph of Sec. 408 (Court Doctrine: Requirement applies to actions covered by the CFI as well
referred to lupon) as the inferior courts. The law, as written, makes no distinction
 In which case the compromise settlement agreed upon by whatsoever with respect to the classes of civil disputes that should
the parties shall be submitted to the court and, upon be compromised at the barangay level.
approval, shall have the force and effect of a judgment of
said court
Uy v. Contreras (1994)
 417: Execution
Summary: An argument degenerated into a scuffle between Uy
o The amicable settlement or arbitration award may be
(sub-lessee) and Atayde (sub-lessor). As a result, the Office of the
enforced by execution by the lupon within 6 months from the
Provincial Prosecutor of Rizal filed two informations for slight
date of the settlement.
physical injuries against Uy. The latter filed a motion to dismiss the

Bautista, Gastanes, Lopez, Sulit Page | 49


C2017 / LAW 154 / Local Government Law / Midterm Reviewer
criminal cases for noncompliance with the requirement of P.D. No. compliance with prior barangay conciliation procedure under the
1508 on prior referral to the Lupong Tagapamayapa. Public Revised Katarungang Pambarangay Law and its IRR, as a pre-
respondent Judge Contreras denied the MTD. Hence, this petition by condition to judicial action, particularly whether the certification to
Uy, the defendant. SC granted petition and held that the criminal file action attached to the case records comply with the
cases should be dismissed for noncompliance with Sections 408, requirements.
409, 410, and 412 of the Local Government Code of 1991 in relation
to Sec 7, Rule VI of the Rules Implementing P.D. No. 1508. Mendova v. Judge Afable (2002)
Summary: On Feb. 18, 1998, Mendova filed with the Office of the
Doctrine: To emphasize the vital role which the revised katarungang Barangay Chairman of Poblacion, San Julian, Eastern Samar, a
pambarangay law plays in the delivery of justice at the barangay complaint for Slight Physical Injuries. Because of failure to reach an
level, in promoting peace, stability, and progress therein, and in amicable settlement, he filed with the MCTC a complain for Slight
effectively preventing or reducing expensive and wearisome Physical Injuries on May 4, 1998. Judge Afable dismissed the
litigation, parties to disputes cognizable by the lupon should complaint on the ground of prescription (beyond 2 month period).
exhaust the remedies provided by that law. Government prosecutors The SC held that since Mendova did not avail other remedies such
should exercise due diligence in ascertaining compliance with it, and as a MR, this administrative complaint is premature. Moreover,
trial courts should not hesitate to impose the appropriate sanctions complainant failed to present proof of his receipt of the Barangay
for noncompliance thereof. Certification and so clearly, he cannot now fault respondent judge
for dismissing the case on the ground of prescription.
Wingards v. Mejia (1995)
Summary: Administrative complaints were filed against MTC Judge Doctrine: Pursuant to Section 410(c) of the Local Government Code,
Mejia because, among others, he took cognizance of a criminal case the prescriptive period is interrupted upon filing of the complaint
for grave threats allegedly despite the lack of prior barangay with the barangay, and resumes upon receipt by the complainant of
conciliation. He was held administratively liable. the Certification to File Action issued by the Pangkat Secretary of the
barangay.
Doctrine: Judges are directed to desist from improvidently receiving
and desultorily acting on complaints, petitions, actions or
2. Sangguniang Kabataan
proceedings in cases falling within the authority of the Lupon
Tagapamayapa. The proceedings before the lupon are a
Chapter VIII – Sangguniang Kabataan – 423 – 435
precondition to the filing of any action or proceeding in court or other
government office. Such an initiatory pleading, if filed without  423: Creation and Election
compliance with the precondition, may be dismissed on motion of  424: Katipunan ng Kabataan
any interested party on the ground that it fails to state a cause of  425: Meetings of the Katipunan ng Kabataan
action.  426: Powers and Functions of the Sangguniang Kabataan
 427: Meetings of the Sangguniang Kabataan
Corpuz v. CA (1997)  428: Qualifications
Summary: Corpuz and Alvarado were tenants in Barredo property.  429: Term of Office
Barredo wanted to sell to tenants but tenants had no money so  430: Sangguniang Kabataan Chairman
Barredo sold to Corpuz. Alvarado became Corpuz tenant. Corpuz  431: Powers and Duties of the Sangguniang Kabataan Chairman
wanted Alvarado out because Corpuz’ children needed the room.  432: Sangguniang Kabataan Secretary
Corpuz files UD. MeTC grants. RTC reverse because issue of  433: Sangguniang Kabataan Treasurer
ownership being resolved in NHA and CA affirm RTC. SC held that  434: Privileges of Sangguniang Kabataan Officials
issue of ownership does not abate ejectment suit.  435: Succession and Filing of Vacancies

Doctrine: Referral to Lupong Tagapayapa not a jurisdictional Chapter IX – Pederasyon ng mga Sangguniang Kabataan – 436 – 438
requirement and it cannot affect jurisdiction already acquired over  436: Pederasyon ng mga Sangguniang Kabataan
the subject matter and parties
 437: Constitution and By-Laws
 438: Membership in the Sanggunian
Bonifacio Law Office v. Judge Bellosillo (2002)
Summary: MTC QC Judge Bellosillo was administratively charged for Chapter X – Linggo ng Kabataan – 439
ordering the referral of an ejectment case back to the barangay for  439: Observance of Linggo ng Kabataan
conciliation proceedings despite the complaint allegedly being
attached with a Certification to File Motion. SC said that Judge Sec. 423. Creation and Election. –
should not be sanctioned based on such ground because he was (a) There shall be in every barangay a sangguniang kabataan to be
merely seeking to promote the objectives of barangay conciliation. composed of a chairman, seven (7) members, a secretary, and
a treasurer.
Doctrine: All complaints and/or informations before the RTC/MTC (b) A sangguniang kabataan official who, during his term of office,
shall be carefully scrutinized to determine if there has been shall have passed the age of twenty-one (21) years shall be

Bautista, Gastanes, Lopez, Sulit Page | 50


C2017 / LAW 154 / Local Government Law / Midterm Reviewer
allowed to serve the remaining portion of the term for which he less than eighteen (18) years of age on the day of the election, able
was elected. to read and write Filipino, English, or the local dialect, and must not
have been convicted of any crime involving moral turpitude.
Sec. 424. Katipunan ng Kabataan. - The katipunan ng kabataan
shall be composed of Filipino citizens actually residing in the Sec. 429. Term of Office. - The sangguniang kabataan chairman and
barangay for at least six (6) months, who are fifteen (15) but less than members shall hold office for a period of three (3) years, unless
eighteen (18) years of age on the day of the election, and who are sooner removed for cause as provided by law, permanently
duly registered in the list of the sangguniang kabataan or in the incapacitated, die or resign from office.
official barangay list in the custody of the barangay secretary.
Sec. 430. Sangguniang Kabataan Chairman. - The registered voters
Sec. 425. Meetings of the Katipunan ng Kabataan. - The katipunan of the katipunan ng kabataan shall elect the chairman of the
ng kabataan shall meet at least once every three (3) months, or at sangguniang kabataan who shall automatically serve as an ex
the call of the chairman of the sangguniang kabataan or upon officio member of the sangguniang barangay upon his assumption
written petition of at least one-twentieth (1/20) of its members, to to office. As such, he shall exercise the same powers, discharge the
decide on important issues affecting the youth of the barangay. same duties and functions, and enjoy the same privileges as the
regular sangguniang barangay members, and shall be the chairman
Sec. 426. Powers and Functions of the Sangguniang Kabataan. - The of the committee on youth and sports development in the said
sangguniang kabataan shall: sanggunian.
(a) Promulgate resolutions necessary to carry out the objectives of
the youth in the barangay in accordance with the applicable Sec. 431. Powers and Duties of the Sangguniang Kabataan
provisions of this Code; Chairman. - In addition to the duties which may be assigned to him
(b) Initiate programs designed to enhance the social, political, by the sangguniang barangay, the sangguniang kabataan chairman
economic, cultural, intellectual, moral, spiritual, and physical shall:
development of the members; (a) Call and preside over all meetings of the katipunan ng
(c) Hold fund-raising activities, the proceeds of which shall be tax- kabataan and the sangguniang kabataan;
exempt and shall accrue to the general fund of the (b) Implement policies, programs, and projects within his
sangguniang kabataan: Provided, however, That in the jurisdiction in coordination with the sangguniang barangay;
appropriation thereof, the specific purpose for which such (c) Exercise general supervision over the affairs and activities of the
activity has been held shall be first satisfied; sangguniang kabataan and the official conduct of its members,
(d) Create such bodies or committees as it may deem necessary to and such other officers of the sangguniang kabataan within his
effectively carry out its programs and activities; jurisdiction;
(e) Submit annual and end-of-term reports to the sangguniang (d) With the concurrence of the sangguniang kabataan, appoint
barangay on their projects and activities for the survival and from among the members of the sangguniang kabataan, the
development of the youth in the barangay; secretary and treasurer and such other officers as may be
(f) Consult and coordinate with all youth organizations in the deemed necessary; and
barangay for policy formulation and program implementation; (e) Exercise such other powers and perform such other duties and
(g) Coordinate with the appropriate national agency for the functions as may be prescribed by law or ordinance.
implementation of youth development projects and programs
at the national level; Sec. 432. Sangguniang Kabataan Secretary. - The sangguniang
(h) Exercise such other powers and perform such other duties and kabataan secretary shall:
functions as the sangguniang barangay may determine or (a) Keep all records of the katipunan ng kabataan and
delegate or as may be prescribed by law or ordinance. sangguniang kabataan;
(b) Prepare and keep the minutes of all meetings of the katipunan
Sec. 427. Meetings of the Sangguniang Kabataan. - The sangguniang ng kabataan and sangguniang kabataan;
kabataan shall meet regularly once a month on the date, time, and (c) Prepare all forms necessary for the conduct of registrations,
place to be fixed by the said sanggunian. Special meetings may be elections, initiatives, referenda, or plebiscites, in coordination
called by the sangguniang kabataan chairman or any three (3) of its with the barangay secretary and the COMELEC; and
members by giving written notice to all members of the date, time, (d) Perform such other duties and discharge such other functions
place and agenda of the meeting at least one (1) day in advance. as the chairman of the sangguniang kabataan may prescribe or
Notices of regular or special meetings shall be furnished the punong direct.
barangay and the sangguniang barangay.
A majority of the members of the sangguniang kabataan shall Sec. 433. Sangguniang Kabataan Treasurer. - The sangguniang
constitute a quorum. kabataan treasurer shall:
(a) Take custody of all sangguniang kabataan property and funds
Sec. 428. Qualifications. - An elective official of the sangguniang not otherwise deposited with the city or municipal treasurer;
kabataan must be a Filipino citizen, a qualified voter of the (b) Collect and receive contributions, monies, materials, and all
katipunan ng kabataan, a resident of the barangay for at least one other resources intended for the sangguniang kabataan and
(1) year immediately prior to election, at least fifteen (15) years but the katipunan ng kabataan;

Bautista, Gastanes, Lopez, Sulit Page | 51


C2017 / LAW 154 / Local Government Law / Midterm Reviewer
(c) Disburse funds in accordance with an approved budget of the (1) In municipalities, pambayang pederasyon ng mga
sangguniang kabataan; sangguniang kabataan;
(d) Certify to the availability of funds whenever necessary; (2) In cities, panlungsod na pederasyon ng mga sangguniang
(e) Submit to the sangguniang kabataan and to the sangguniang kabataan;
barangay certified and detailed statements of actual income (3) In provinces, panlalawigang pederasyon ng mga
and expenditures at the end of every month; and sangguniang kabataan;
(f) Perform such other duties and discharge such other functions (4) In special metropolitan political subdivisions,
as the chairman of the sangguniang kabataan may direct. pangmetropolitang pederasyon ng mga sangguniang
kabataan; and
Sec. 434. Privileges of Sangguniang Kabataan Officials. - The (5) On the national level, pambansang pederasyon ng mga
sangguniang kabataan chairman shall have the same privileges sangguniang kabataan.
enjoyed by other sangguniang barangay officials under this Code (b) The pederasyon ng mga sangguniang kabataan shall, at all
subject to such requirements and limitations provided herein. levels, elect from among themselves the president, vice-
During their incumbency, sangguniang kabataan officials shall be president and such other officers as may be necessary and shall
exempt from payment of tuition and matriculation fees while be organized in the following manner:
enrolled in public tertiary schools, including state colleges and (1) The panlungsod and pambayang pederasyon shall be
universities. The National Government shall reimburse said college composed of the sangguniang kabataan chairmen of
or university the amount of the tuition and matriculation fees: barangays in the city or municipality, respectively;
Provided, That, to qualify for the privilege, the said officials shall (2) The panlalawigang pederasyon shall be composed of
enroll in a state college or university within or nearest their area of presidents of the panlungsod and pambayang
jurisdiction. pederasyon;
(3) The pangmetropolitang pederasyon shall be composed of
Sec. 435. Succession and Filling of Vacancies. – presidents of the panlungsod and pambayang
(a) In case a sangguniang kabataan chairman refuses to assume pederasyon;
office, fails to qualify, is convicted of a felony, voluntarily (c) The elected presidents of the pederasyon at the provincial,
resigns, dies, is permanently incapacitated, is removed from highly urbanized city, and metropolitan political subdivision
office, or has been absent without leave for more than three (3) levels shall constitute the pambansang katipunan ng mga
consecutive months, the sangguniang kabataan member who sangguniang kabataan.
obtained the next highest number of votes in the election
immediately preceding shall assume the office of the chairman Sec. 437. Constitution and By-Laws. - The term of office, manner of
for the unexpired portion of the term, and shall discharge the election, removal and suspension of the officers of the pederasyon
powers and duties, and enjoy the rights and privileges ng mga sangguniang kabataan at all levels shall be governed by the
appurtenant to the office. In case the said member refuses to constitution and by-laws of the pederasyon in conformity with the
assume the position or fails to qualify, the sangguniang provisions of this Code and national policies on youth.
kabataan member obtaining the next highest number of votes
shall assume the position of the chairman for the unexpired Sec. 438. Membership in the Sanggunian. –
portion of the term. (a) A sangguniang kabataan chairman shall, upon certification of
(b) Where two (2) or more sangguniang kabataan members his election by the COMELEC and during his tenure of office is
obtained the same next highest number of votes, the other elected as pederasyon president, serve as an ex officio member
sangguniang kabataan members shall conduct an election to of the sangguniang panlalawigan, sangguniang panlungsod,
choose the successor to the chairman from among the said and sangguniang bayan, as the case may be, without need of
members. further appointment.
(c) After the vacancy shall have been filled, the sangguniang (b) The vice-president of the pederasyon whose president has been
kabataan chairman shall call a special election to complete the elected as president of a higher pederasyon shall serve as ex
membership of said sanggunian. Such sangguniang kabataan officio member of the sanggunian concerned without need of
member shall hold office for the unexpired portion of the term further appointment.
of the vacant seat. (c) The pederasyon president or vice-president, as the case may
(d) In case of suspension of the sangguniang kabataan chairman, be, shall be the chairman of the committee on youth and sports
the successor, as determined in subsections (a) and (b) of this development of the sanggunian concerned.
section shall assume the position during the period of such
suspension. CHAPTER 10. - Linggo ng Kabataan

CHAPTER 9. - Pederasyon ng mga Sangguniang Kabataan Sec. 439. Observance of Linggo ng Kabataan. –
(a) Every barangay, municipality, city and province shall, in
Sec. 436. Pederasyon ng mga Sangguniang Kabataan. – coordination with the pederasyon ng mga sangguniang
(a) There shall be an organization of all the pederasyon ng mga kabataan at all levels, conduct an annual activity to be known
sangguniang kabataan to be known as follows: as the Linggo ng Kabataan on such date as shall be determined
by the Office of the President.

Bautista, Gastanes, Lopez, Sulit Page | 52


C2017 / LAW 154 / Local Government Law / Midterm Reviewer
(b) The observance of the Linggo ng Kabataan shall include the (respondent Sales), COMELEC en banc issued an order to suspend
election of the counterparts of all local elective and appointive petitioner’s proclamation if she won (which she did). SC held that
officials, as well as heads of national offices or agencies petitioner was ineligible to run for the position. Sec428, LGC
stationed or assigned in the territorial jurisdiction of the local specifically provides that an elective official must not be more than
government unit, among in- school and community youth 21 years old on the day of his election. On the day of the elections,
residing in the local government unit concerned from ages petitioner was 21 years, 11 months and 20 days old, thus she is
thirteen (13) to seventeen (17). During said week, they shall hold ineligible to run as a candidate.
office as boy and girl officials and shall perform such duties and
conduct such activities as may be provided in the ordinance Doctrine: The phrase "not more than 21 years of age" means not over
enacted pursuant to this Chapter. 21 years, it is not equivalent to "less than 22 years old.”

ALU v. Letrondo-Montejo (1994) Montesclaros v. COMELEC (2002)


Summary: Petitioners are members of ALU-TUCP employed by AMS Summary: 20-year old petitioners (kids) seek to prevent the
Farming Corporation. Based on the CBA of the parties, AMS will pay postponement of the SK elections and the reduction of the age
its employees a holiday pay. December 4, 1992 was declared a requirement from “at least 15 but not more than 21 years of age” (as
special day for the holding of SK Elections. The petitioners are provided in the LGC of 1991) to “at least 15 but less than 18 years of
claiming their holiday pay for the said date; AMS refused on the age” (as recommended by COMELEC, provided in the consolidated
ground that it is not a holiday within the contemplation of the CBA. bill of the 2 houses, and subsequently provided by law under R.A.
The Supreme Court held that AMS Farming must pay the holiday No. 9164, which was passed after this petition was filed). The kids
pay to its employees. Election for members of the SK may properly argue, among others, that the acts discriminate, disenfranchise,
be considered a "local election" within the meaning of the CBA and single out, and dismember the SK members who are 18 but not more
the day on which it is held to be a holiday. than 21 years old, composed of about 7 million youth. Court denied
the petition. There was no actual controversy, the kids have no
Doctrine: Sangguniang Kabataan (SK) is part of the local standing, and there was no constitutional issue when the petition
government structure. The Local Government Code creates in every was filed.
barangay a Sangguniang Kabataan
DOCTRINE: SK membership is not a property right. It is a mere
Mercado v. Board of Election Supervisors (1995) statutory right conferred by law. One who is no longer qualified
Summary: After election, Mercado was declared the SK chairman of because of an amendment in the law cannot complain of being
Brgy. Mabalor. However, his rival Pangilinan filed a protest with the deprived of a proprietary right to SK membership
BES. After a recount, Pangilinan was declared the winner. Mercado
sought recourse with the RTC but the latter dismissed it averring B. The Municipality
that it had no jurisdiction pursuant to COMELEC Res. No. 2499
vesting BES as the final arbiter of SK election controversies. Before
the SC, Mercado assailed the RTC order and also prayed for the
Muñez v. Arino (1995)
Summary: Mayor Irisari issued a warrant of arrest against Muñez
nullification of Res. 2499 as the provision for BES as final arbiter was
after the latter failed to attend conference re: a land dispute. Muñez
contrary to Omnibus Election Code and Constitution. The SC held
that Res. No. 2499 was not contrary to the Election Code nor the filed (1) a complaint against the mayor for grave misconduct and
usurpation of judicial function with the Office of the Ombudsman,
Constitution for the provisions invoked referred to barangay officials.
SK officials are NOT the barangay officials contemplated in and (2) an administrative complaint for violation of the Constitution,
Omnibus Election Code. However, Mercado’s appeal to the RTC misconduct in office, and abuse of authority with the Sangguniang
Panlalawigan. The criminal case against the mayor for usurpation of
must still be granted in its exercise of judicial review.
judicial function came before MCTC Judge Ariño, who initially denied
the mayor’s motion to quash on the ground that the power of mayors
Doctrine: BES has direct general supervision in the conduct of SK
election and as the final arbiter of all election protests. No law in to issue warrants of arrest had ceased to exist as of Feb. 2, 1987
effect prior to the ratification of the Constitution had made the SK when the Constitution took effect. Judge Ariño, however, reversed
chairman an elective barangay official. The SK’s chairman being an his previous order and dismissed the case, relying on a DILG
ex-officio member of the sanggu brgy does not also make him a Resolution (which found that the mayor only issued an invitation or
summons, not a warrant of arrest). While Judge Ariño may have
barangay official for the law specifically provides who are its elective
acted in good faith, SC still held him administratively held liable. A
members.
warrant of arrest was clearly issued. Mayor Irisari justified his order
on the basis of Sec. 143(3) of the former LGC (BP Blg. 337). This
provision has, however, been repealed by Art. III, Sec. 2 of the 1987
Garvida v. Sales (1997) Constitution.
Summary: Petitioner sought to run for the position of Sangguniang
Kabataan chairman in the 1996 elections. However, her certificate of Doctrine: The mayor no longer has the power to conduct preliminary
candidacy was disapproved due to her age. Although she was investigations, much less issue orders of arrest. The function of
eventually allowed to run, due to a petition filed by her rival determining probable cause and issuing, on the basis thereof,

Bautista, Gastanes, Lopez, Sulit Page | 53


C2017 / LAW 154 / Local Government Law / Midterm Reviewer
warrants of arrest or search warrants, may be validly exercised only of this Constitution and the national sovereignty as well as territorial
by judges. integrity of the Republic of the Philippines.

Greater Balanga Devt v. Municipality of Balanga, Bataan Sec. 16. The President shall exercise general supervision over
autonomous regions to ensure that the laws are faithfully executed.
(1994)
Supra
Sec. 17. All powers, functions, and responsibilities not granted by this
Constitution or by law to the autonomous regions shall be vested in
C. The City the National Government.

Lim and Garayblas v. CA (2002) Sec. 18. The Congress shall enact an organic act for each
Summery: Mayor Lim instructed policemen to inspect Bistro’s autonomous region with the assistance and participation of the
license and work permits. This cause stoppage of operations in regional consultative commission composed of representatives
Bistro’s nightclub and restaurant. Lim also refused to accept Bistro’s appointed by the President from a list of nominees from
application for a business license and he later closed down the multisectoral bodies. The organic act shall define the basic structure
establishment. Lim contends that as mayor, he has the power to of government for the region consisting of the executive department
grant and refuse applications for business licenses. This power and legislative assembly, both of which shall be elective and
implicitly includes power to inspect, investigate and close down representative of the constituent political units. The organic acts
Bistro’s operations for violation of the conditions of its licenses and shall likewise provide for special courts with personal, family, and
permits. SC held that Lim cannot close down these establishments property law jurisdiction consistent with the provisions of this
because powers are expressly premised on the violations of the Constitution and national laws.
conditions.
The creation of the autonomous region shall be effective when
Doctrine: Power of the mayor to issue business licenses and permits approved by majority of the votes cast by the constituent units in a
necessarily includes the corollary power to suspend, revoke or even plebiscite called for the purpose, provided that only provinces, cities,
refuse to issue the same. However, the power to suspend or revoke and geographic areas voting favorably in such plebiscite shall be
these licenses and permits is expressly premised on the violation of included in the autonomous region.
the conditions of these permits and licenses. Also, the Mayor cannot
close down an establishment without due process of law; there must Sec. 20. Within its territorial jurisdiction and subject to the provisions
be a prior notice and hearing. of this Constitution and national laws, the organic act of
autonomous regions shall provide for legislative powers over:
(1) Administrative organization;
C. The Province (2) Creation of sources of revenues;
(3) Ancestral domain and natural resources;
E. Application of the Code to LGU in Autonomous (4) Personal, family, and property relations;
(5) Regional urban and rural planning development;
Region
(6) Economic, social, and tourism development;
SEC. 526. Application of this Code to Local Government Units in the (7) Educational policies;
Autonomous Regions. - This Code shall apply to all provinces, cities, (8) Preservation and development of the cultural heritage; and
municipalities and barangays in the autonomous regions until such (9) Such other matters as may be authorized by law for the
time as the regional government concerned shall have enacted its promotion of the general welfare of the people of the region.
own local government code.
Sec. 21. The preservation of peace and order within the regions shall
F. The Autonomous Region in Muslim Mindanao be the responsibility of the local police agencies which shall be
organized, maintained, supervised, and utilized in accordance with
1987 Constitution, Art. X applicable laws. The defense and security of the regions shall be the
responsibility of the National Government.
Sec. 1. The territorial and political subdivisions of the Republic of the
Philippines are the provinces, cities, municipalities, and barangays. RA 6734: An Act Providing for the Organic Act for Autonomous Region
There shall be autonomous regions in Muslim Mindanao and the of Muslim Mindanao
Cordilleras as hereinafter provided.
RA 9054: An Act to Strengthen and Expand the Organic Act for the
Sec. 15. There shall be created autonomous regions in Muslim Autonomous Region of Muslim Mindanao, Amending RA 6734
Mindanao and in the Cordilleras consisting of provinces, cities,
municipalities, and geographical areas sharing common and Art II, Sec. 3. Seat of Autonomous Government. - The regional
distinctive historical and cultural heritage, economic and social legislative assembly, hereinafter referred to as the Regional
structures, and other relevant characteristics within the framework Assembly, shall by law, fix the permanent seat of government of the
regional government in any province or city that is a member of the

Bautista, Gastanes, Lopez, Sulit Page | 54


C2017 / LAW 154 / Local Government Law / Midterm Reviewer
autonomous region, taking into consideration accessibility and cultural communities, for lower positions in the above organs of the
efficiency in which its mandate may be carried out under this government cannot be appointed anymore thereto.
Organic Act. xxx
Election of legislators to represent the autonomous region in the
Art III, Sec. 3. Devolution of Powers. - The regional government shall Congress of the Republic shall be done pursuant to the rules of the
adopt a policy on local autonomy whereby regional powers shall be Commission on Elections.
devolved to local government units particularly in areas of
education, health, human resource, science and technology and Art V, Sec 5. Representatives in Executive Departments and
people empowerment. Until a law implementing this provision is Constitutional Bodies. -At least, one (1) qualified inhabitant of the
enacted by the Regional Assembly, Republic Act No. 7160, the Local autonomous region recommended by the Regional Governor
Government Code of 1991, shall continue to apply to all the consultation with the Regional Assembly and concerned sectors of
provinces, cities, municipalities, and barangay within the the autonomous region shall be appointed, as far as practicable, in
autonomous region. each of the departments, offices or bureaus and constitutional
bodies of the central government or national government that deal
The Regional Assembly may not pass any law to diminish, lessen, or with the autonomous region, in primarily confidential, highly
reduce the powers, functions, and shares in the internal revenue technical, or policy-determining positions.
taxes of the said local government units as provided by Republic Act
No. 7160, the Local Government Code of 1991. Art VI, Sec 1 Regional Assembly. - The legislative power of the
autonomous government shall be vested in the Regional Assembly
Art IV, Sec. 1. Powers and Functions. - Subject to the provisions of the except to the extent that it is reserved to the people by provisions on
Constitution, the Regional Government shall exercise those powers initiative and referendum as provided by law.
and functions expressly granted to it in this Organic Act, or
necessary for or incidental to the proper governance and Art VI, Sec 2. Election of Regional Assembly. - The Regional Assembly
development of all the constituent units within the autonomous shall be composed of Members elected by popular vote, with three
region consistent with the policy on regional and local autonomy (3) members elected from each of the legislative districts.
and decentralization.
Art VII, Sec 1. Executive Power. - The executive power shall be vested
The Regional Government may enact its own regional in a Regional Governor. He shall be elected by the qualified voters of
administrative code and regional local government code consistent the autonomous region.
with the Constitution. The powers and functions already vested upon
and the shares of the national taxes provided by Republic Act No. Art VIII, Sec 1. Exercise of Judicial Power. - The judicial powers shall
7160, the Local Government Code of 1991, to provinces, cities, be vested in the Supreme Court and in such lower courts as may be
municipalities, and barangay in the autonomous region shall not be established by law including the Shari'ah Courts in accordance with
reduced. Section 5 hereof.

Art V, Sec 2. Cabinet Membership. - As far as practicable, it shall be Art VIII, Sec 2. Justices from Autonomous Region. - It shall be the
the policy of the national government that there shall be at least one policy of the central government or national government that,
(1) member of the cabinet with a rank of a department secretary who whenever feasible, at least one (1) justice in the Supreme Court and
is an inhabitant of the autonomous region to be recommended by two (2) justices in the Court of Appeals shall come from qualified
the Regional Governor in consultation with elected officials and jurists of the autonomous region. For this purpose, the Regional
concerned sectors of the autonomous region. Governor may, after consultations with the Regional Assembly and
concerned sectors in the autonomous region, submit the names of
Art V, Sec 4. Representation of Autonomous Region in General in the qualified persons to the Judicial and Bar Council for its
Central Government or National Government. -Representation of the consideration. The appointments of those recommended by the
inhabitants of the autonomous region in the central government or Regional Governor to the judicial positions mentioned above are
national government may be done by appointment or election. without prejudice to appointments that may be extended to other
qualified inhabitants of the autonomous region to other positions in
Appointment of inhabitants of the autonomous region to positions the Judiciary.
in the central government or national government shall be subject
to central government or national government standards and Art VIII, Sec 9. Jurisdiction of the Shari'ah Appellate Court. -The
guidelines. Such appointment shall be made only upon Shari'ah Appellate Court shall:
recommendation by the Regional Governor after consultation with
the Regional Assembly and the concerned sectors of the (a)Exercise original jurisdiction over petitions for certiorari,
autonomous region. prohibition, mandamus, habeas corpus, and other auxiliary writs
and processes only in aid of its appellate jurisdiction; and,
Right of representation shall not be construed in such a way that (b)Exercise exclusive appellate jurisdiction over all cases tried in the
applicants from the autonomous region, especially Muslims and Shari'ah district courts as established by law.

Bautista, Gastanes, Lopez, Sulit Page | 55


C2017 / LAW 154 / Local Government Law / Midterm Reviewer
Disoman Cop v. Datumanong (2004) Governor Mutilan issued an office order designating Dr. Saber also
Summary: DO 119 created a DPWH Marawi Sub-District Engineering as OIC for the same hospital. Saver filed with the CA a petition for
Office and RA 8999 constituted into an engineering district to be quo warranto, claiming that he was the lawfully designated OIC. CA
known as the First Engineering District of the Province of Lanao del held in Saber’s favor. Petitioners appealed to the SC. SC held in their
Sur. Petitioners challenged their constitutionality, alleging that favor and granted their petition. Under the current law, Macacua has
DO119 and RA8999 violate the constitutional autonomy of the the authority to assign and designate Sani to their respective
ARMM. DO 119 has tasked the Marawi Sub-District Engineering position.
Office with functions that have already been devolved to the DPWH-
ARMM First Engineering District in Lanao del Sur. SC held that By Doctrine: The ARMM Local Code vests in the Provincial Governor the
creating an office with previously devolved functions, R.A. 8999, in power to “exercise general supervision and control over all
essence, sought to amend R.A. 6074. The amendatory law should programs, projects, services, and activities of the provincial
therefore first obtain the approval of the people of the ARMM before government.” Upon the effectivity of the ARMM Local Code, the
it could validly take effect. Absent compliance with this requirement, power of supervision and control over the provincial health officer
R.A. 8999 has not even become operative. passed from the Regional Secretary to the Provincial Governor.
From then on the Provincial Governor began to exercise the
Doctrine: In treading their chosen path of development, the Muslims administrative authority to designate an Officer inCharge in the
in Mindanao are to be given freedom and independence with provincial health office pending the appointment of a permanent
minimum interference from the National Government. This provincial health officer.
necessarily includes the freedom to decide on, build, supervise and
maintain the public works and infrastructure projects within the Time Period Laws Applicable
autonomous region. The devolution of the powers and functions of 1st Before August 1, 1989 EO 119 & 1984 LGC
the DPWH in the ARMM and transfer of the administrative and fiscal Period
management of public works and funds to the ARG are meant to be 2nd August 1, 1989 - Organic Act of 1989,
true, meaningful and unfettered. With R.A. 8999, however, this Period January 1, 1992 1984 LGC
freedom is taken away, and the National Government takes control
3rd January 1, 1992 - Organic Act of 1989,
again. Evidently, the intention is to cede some, if not most, of the
Period March 3, 1994 1991 LGC, EO 133
powers of the national government to the autonomous government
in order to effectuate a veritable autonomy. The continued 4th March 3, 1994 – Organic Act of 1989,
enforcement of R.A. 8999, therefore, runs afoul of the ARMM Period August 14, 2001 ARMM LGC
Organic Acts and results in the recall of powers, which have 5th August 14, 2001 Organic Act of 2001,
previously been handed over. Period onwards ARMM LGC

Abbas v. COMELEC (1989)


Summary: Petitioners Abbas, Mam-o, et al. assail the
G. Cordillera Administrative Region
constitutionality of several portions of RA 6734 entitled “An Act
Providing for an Organic Act of ARMM” on several grounds. They EO 220: Creating a Cordillera Adminstrative Region (CAR),
pray that the COMELEC be enjoined from conducting the plebiscite Appropriating Funds Therefor And For Other Purposes
and the DBM from releasing funds for that purpose. They claim that
RA 6738 violates the Constitution and the Tripoli Agreement. Court Sec. 3. Purposes. The CAR shall have the following purposes:
ultimately upheld the constitutionality of the RA 6734. (a) Administer the affairs of government in the region as defined in
Section 4 and 5 below;
Doctrine: Creation of the autonomous region shall take effect only (b) Accelerate the economic and social growth and development
when approved by a majority of the votes cast the constituent units of the units of the region; and
in a plebiscite, and only those provinces and cities where a majority (c) Prepare for the establishment of the autonomous region in the
vote in favor of the Organic Act shall be included in the autonomous Cordilleras.
region. The single plebiscite contemplated by the Constitution and
R.A. No. 6734 will be determinative of (1) whether there shall be an Sec. 4. Scope of Authority and Responsibility.
autonomous region in Muslim Mindanao and (2) which provinces The CAR shall have authority and responsibility in the region over
and cities, among those enumerated in R.A. No. 6734 shall comprise the following:
it. (a) Regional administrative system;
(b) Economic, social and cultural development;
(c) Agricultural, commercial and industrial development and
Pandi v. CA (2002) promotion of tourism;
Summary: ARMM Secretary of Health Macacua issued a (d) Infrastructure development;
memorandum assigning Dr. Pandi as Officer in Charge of the (e) Urban and rural development, protection of ancestral domain
Integrated Provincial Health Office – Amai Pakpak General Hospital, and land reform;
Lanao del Sur (also detailed Dr. Sani as provincial health officer of (f) Regional educational system, including the establishment and
the same hospital). A little later, the Lanao del Sur Provincial maintenance of educational institutions and the formulation of

Bautista, Gastanes, Lopez, Sulit Page | 56


C2017 / LAW 154 / Local Government Law / Midterm Reviewer
educational policies to cultivate the indigenous Cordillera act was approved by the region, and the DOJ Secretary issued a
cultures and inculcate traditional values; memo to the President reiterating the COMELEC resolution (also to
(g) Health, sports, welfare and social services; the effect of certifying as to the validity of the creation of the
(h) Development of indigenous laws and political institutions, autonomous region despite Ifugao being the lone province).
particularly those of direct democracy and collective Congress also enacted RA 6861 setting the election for the said
leadership, as well as the promotion of indigenous institutions region. Lastly, the President issued AO 160 abolishing the CEB and
and processes for conflict resolution and dispute settlement; CRA. Petitioners questioned the validity of the creation of the
(i) Preservation and enhancement of indigenous customs, autonomous region despite Ifugao being the sole province which
traditions, languages and cultures; voted for its inclusion. The SC ruled that Ifugao, in itself, cannot
(j) Strengthening of the bodong system of tribal unity and comprise the Cordillera Administrative Region.
cooperation;
(k) Protection and preservation of the cultural identity, values, Doctrine: A single province cannot comprise an autonomous region.
mores and norms of the various ethno-linguistic groups in the It is contrary to the Constitution and RA 6766. It is also impractical
Cordilleras; and illogical.
(l) Promotion of social justice and protection of human rights,
particularly the rights of women, children, the elderly and Cordillera Broad Coalition v. COA (1990)
disadvantaged groups, as well as the rights of people's supra
organizations; and
(m) Such other matters as may be authorized by law or delegated
by the President for the promotion of the general welfare. H. The Metropolitan Manila Development Authority

Sec. 5. Powers and Functions. LGC, Sec. 11. The Congress may, by law, create special metropolitan
The CAR shall coordinate the planning and implementation of political subdivisions, subject to a plebiscite as set forth in Section
programs and services in the areas enumerated in Section 4. 10 hereof. The component cities and municipalities shall retain their
Accordingly, it shall be vested with, among others, the following basic autonomy and shall be entitled to their own local executives
powers and functions: and legislative assemblies. The jurisdiction of the metropolitan
(a) Coordinate with the local government units as well as with the authority that will hereby be created shall be limited to basic
executive departments of the National Government in the services requiring coordination.
supervision of field offices and in identifying, planning,
monitoring, and accepting projects and activities in the region; MMDA v. Bel-Air Villages Ass’n (2000)
(b) Appoint, supervise, control and discipline personnel of the CAR supra
and of such other offices as may be funded by it;
(c) Manage and control funds, facilities and equipment
appropriated for the CAR;
MMDA v. Garin (2005)
Summary: Garin filed a case questioning the authority of the MMDA
(d) Advise the National Government on matters affecting the
to confiscate driver’s license of traffic rule violators claiming that
Cordilleras;
such authority was not granted to it and being a violation of the due
(e) Undertake studies towards codifying the customary laws of the
process clause. The SC ruled that MMDA had the power to
tribes, including the pagtas of the bodong system; and
confiscate Garin’s license.
(f) Promulgate and implement resolutions, rules and regulations
necessary to achieve effectively the purposes of this Executive
Doctrine: Confiscation of driver’s license is an exercise of Police
Order and to carry out the powers and functions of the CAR.
Power which is exercised only by the legislature and to whom the
legislature has delegated such to. As ruled in BelAir, the MMDA may
Ordillo v. COMELEC (1990) not legislate policies, but may merely enforce and administer rules.
Summary: The people of the Cordilleras voted on the plebiscite for If a traffic law or regulation validly enacted by the legislature or
the creation of the Cordillera AUTONOMOUS Region but ONLY those agencies to whom legislative powers have been delegated, the
Ifugao voted in favor of it. The Executive Secretary issued a petitioner is not precluded to confiscate and suspend or revoke
memorandum for the winding up of affairs of the Cordillera drivers’ licenses.
Executive Board (“CEB”) & the Cordillera Regional Assembly
(“CRA”), the COMELEC issued a resolution stating that the organic

Bautista, Gastanes, Lopez, Sulit Page | 57


C2017 / LAW 154 / Local Government Law / Midterm Reviewer

IX. Municipal Officers and Employees

A. Elective Officials  74: Limitations on Recall


 75: Expenses Incident to Recall Elections
Chapter I – Qualifications and Election – 39 – 43
 39: Qualifications 1. Qualifications
 40: Disqualifications
 41: Manner of Election LGC Sec. 39. Qualifications. –
 42: Date of Election (a) An elective local official must be a citizen of the Philippines; a
 43: Term of Office registered voter in the barangay, municipality, city, or province
or, in the case of a member of the sangguniang panlalawigan,
Chapter II – Vacancies and Succession – 44 – 47 sangguniang panlungsod, or sanggunian bayan, the district
 44: Permanent Vacancies in the Offices of the Governor, Vice- where he intends to be elected; a resident therein for at least
Governor, Mayor, and Vice-Mayor one (1) year immediately preceding the day of the election; and
able to read and write Filipino or any other local language or
 45: Permanent Vacancies in the Sanggunian
dialect.
 46: Temporary Vacancy in the Office of the Local Chief Executive
(b) Candidates for the position of governor, vice- governor or
 47: Approval of Leaves of Absence
member of the sangguniang panlalawigan, or Mayor, vice-
mayor or member of the sangguniang panlungsod of highly
Chapter III – Local Legislation – 48 – 59
urbanized cities must be at least twenty-three (23) years of age
 48: Local Legislative Power on election day.
 49: Presiding Officer (c) Candidates for the position of Mayor or vice-mayor of
 50: Internal Rules of Procedure independent component cities, component cities, or
 51: Full Disclosure of Financial and Business Interests of municipalities must be at least twenty-one (21) years of age on
Sanggunian Members election day.
 52: Sessions (d) Candidates for the position of member of the sangguniang
 53: Quorum panlungsod or sangguniang bayan must be at least eighteen
 54: Approval of Ordinances (18) years of age on election day.
 55: Veto Power of the Local Chief Executive (e) Candidates for the position of punong barangay or member of
 56: Review of Component City and Municipal Ordinances or the sangguniang barangay must be at least eighteen (18) years
Resolutions by the Sangguniang Panlalawigan of age on election day.
(f) Candidates for the sangguniang kabataan must be at least
 57: Review of Barangay Ordinances by the Sangguniang
fifteen (15) years of age but not more than twenty-one (21) years
Panlungsod or Sangguniang Bayan
of age on election day.
 58: Enforcement of Disapproved Ordinances or Resolutions
 59: Effectivity of Ordinances or Resolutions
Notes:
Chapter IV – Disciplinary Actions – 60 – 68
 60: Grounds for Disciplinary Actions Modes of losing Philippine citizenship:
 61: Form and Filing of Administrative Complaints (1) Naturalization in a foreign country;
(2) Express renunciation of citizenship; and
 62: Notice of Hearing
(3) Subscribing to an oath of allegiance to support the Constitution
 63: Preventive Suspension
or laws of a foreign country
 64: Salary of Respondent Pending Suspension
 65: Rights of Respondent Modes to reacquire citizenship:
 66: Form and Notice of Decision (1) Direct act of congress
 67: Administrative Appeals (2) Naturalization
 68: Execution Pending Appeal (3) Rrepatriation

Chapter V – Recall – 69 – 75 Abella v. COMELEC (1991)


 69: By Whom Exercised Facts: Adelina Larrazabal is the wife of a candidate for provincial
 70: Initiation of the Recall Process governor of Leyte who was disqualified for lack of residence. She
 71: Election on Recall filed her own CoC in substitution. Silvestre De la Cruz filed a petition
 72: Effectivity of Recall to disqualify her for false statements in her CoC regarding her
 73: Prohibition from Resignation residence. Abella, another candidate, also filed a complaint.

Bautista, Gastanes, Lopez, Sulit Page | 58


C2017 / LAW 154 / Local Government Law / Midterm Reviewer
COMELEC disqualified Larrazabal and disallowed Abella (who got Labo v. COMELEC (1989)
the 2nd highest number of votes) from being proclaimed. Larrazabal Petitioner Ramon Labo, Jr. was proclaimed mayor-elect of Baguio
insists she is a resident and a registered voter of Kananga. De la Cruz City. A petition for quo warranto was filed by the private respondent
and Abella contend she is a resident and registered voter of Ormoc Luis Lardizabal. Petitioner now asks the SC to restrain the COMELEC
City (a component city of the province of Leyte, independent of the from looking into the question of his citizenship as a qualification for
province). his office as Mayor of Baguio City. He claims that the issue here is
the timeliness of petition for quo warranto against him. The Court
Held: Larrazabal’s disqualification proper for failure to show that held that the petition was filed on time and that is not the problem.
she and her husband maintain separate residences. A citizen may The question is whether or not the petitioner is a foreigner and the
leave for other places for study, practice of vocation, or business, but
there is no showing that Larrazabal left Kananga for these reasons. Held: Ramon J. Labo, Jr. is NOT a citizen of the Philippines and
Larrazabal failed to prove animus revertendi. As a voter in Ormoc therefore DISQUALIFIED from continuing to serve as Mayor of
City, Larrazabal is prohibited from voting and being voted in elective Baguio City. The citizenship and voting requirements were not
offices in the province of Leyte, as mandated by the Constitution and possessed at all in the first place on the day of the election. The
RA 179. Abella should not be installed as the governor. People voted petitioner was disqualified from running as mayor and, although
for Larrazabal; the net effect is that Abella lost. elected, is not now qualified to serve as such. Philippine citizenship
is not a cheap commodity that can be easily recovered after its
Frivaldo v. COMELEC (1989) renunciation. This may not be accomplished by election to public
Facts: Frivaldo was elected governor of Sorsogon but was previously office. The people of that locality could not have, even unanimously,
naturalized as an American citizen. In light of this fact, the League changed the requirements of the Local Government Code and the
of Cities sought to annul his proclamation and to remove him from Constitution.
his office. The Court held that he was not a Filipino Citizen because
he did not reacquire his citizenship through the means accorded by Labo v. COMELEC (1991)
the law either through PD 725 or CA 63. His mere participation in Facts: Ramon Labo Jr. and Roberto Ortega vied for the position of
the congressional elections in 87 (which resulted in an automatic Mayor of Baguio city in the May 1992 Elections. Following the 1989
renunciation of his American Citizenship according to US laws) did SC ruling declaring Labo was not a Filipino citizen, his opponent
not amount to reacquisition. Ortega filed a disqualification case.

Held: The reason for this inquiry is the provision in Article XI, Section Held: SC upholds the 1989 ruling, but declared that Ortega (as
9, of the Constitution that all public officials and employees owe the second-placer) cannot be declared the Mayor. Following Sec. 44 of
State and the Constitution "allegiance at all times" and the specific the LGC, the Vice-Mayor must become Mayor. Although Labo had
requirement in Section 42 of the Local Government Code that a already applied for repatriation, the Special Committee had yet to
candidate for local elective office must be inter alia a citizen of the act upon his application. Mere application for repatriation does not
Philippines and a qualified voter of the constituency where he is amount to an automatic reacquisition of Philippine Citizenship.
running. Section 117 of the Omnibus Election Code provides that a
qualified voter must be, among other qualifications, a citizen of the
Philippines, this being an indispensable requirement for suffrage
Mercado v. Manzano (1999)
Facts: Mercado and Manzano are vice mayor candidates in Makati.
under Article V, Section 1, of the Constitution.
A certain Mamaril filed for disqualification of Manzano for being a
US citizen. Manzano was disqualified by second division of comelec.
Frivaldo v. COMELEC (1996) He filed an MR which remained pending after election. Mercado
Facts: Frivaldo and Lee ran for governor of Sorsogon. Frivaldo was sought to intervene but his motion was also not acted upon.
disqualified by Comelec but filed MR. Election was conducted Comelec en banc reversed the decision of division and said that
without a ruling on the MR. Frivaldo won. Comelec suspended Manzano is qualified. Mercado files petition with SC.
proclamation and eventually disqualified Frivaldo and proclaimed
Lee as the winner. Frivaldo files a petition in Comelec and was Held: SC states that Manzano is not disqualified. What the
rendered as qualified to hold elective position. The issue is whether prohibition in the LGC on dual citizens refer to is not really dual
or not Frivaldo is disqualified because he was a US citizen prior to citizenship but dual allegiance. He also effectively renounced his US
the election and only reacquired citizenship on the day when he was citizenship by taking his oath of allegiance and stating in his CoC
supposed to assume office. that he is a PH citizen.

Held: SC states that the citizenship requirement is not reckoned


prior to the election or during the filing of CoC but on the day the Coquilla v. COMELEC (2002)
person is elected and the term of office is supposed to begin. Facts: Coquilla, a natural born Filipino, became a naturalized US
citizen when he joined the US Navy. Upon retirement, he was
repatriated to the Philippines. In Nov. 2000, he applied to be a
registered voter of Oras, Eastern Samar. In Feb. 2001, he filed for
candidacy for Mayor of Oras. The incumbent mayor, Alvarez,
opposed this saying that Coquilla has not satisfied the 1 year

Bautista, Gastanes, Lopez, Sulit Page | 59


C2017 / LAW 154 / Local Government Law / Midterm Reviewer
residency requirement. SC declared that he did not satisfy the
residency requirement as his residence should be counted from the
time he was repatriated, Nov. 2000.

Held: The residence in Sec. 39(a) LGC is to be understood as domicile


or legal residence that is, “the place where a party actually or
constructively has his permanent home, where he, no matter where
he may be found at any given time, eventually intends to return or
remain (animus manendi). A domicile of origin is acquired by every
person at birth. It is usually the place where the child’s parents
reside and continues until the same is abandoned by acquisition of
new domicile.

Limbona v. COMELEC (2008)


Facts: Norlainie and Mohammad Limbona filed their certificates of
candidacy to run for mayor of Pantar, Lanao del Norte.
Subsequently, Malik also filed his COC. Malik also filed a petition for
disqualification of the Limbonas on the ground of lack of the one-
year residency requirement.

Held: The Supreme Court upheld Norlainie’s disqualification.


 The term "residence" as used in the election law is synonymous
with "domicile," which imports not only intention to reside in a
fixed place but also personal presence in that place, coupled with
conduct indicative of such intention.
 For purposes of election law, the question of residence is mainly
one of intention. In order to acquire a domicile by choice, there
must concur (1) residence or bodily presence in the new locality,
(2) an intention to remain there, and (3) an intention to abandon
the old domicile.
 A person's "domicile" once established is considered to continue
and will not be deemed lost until a new one is established.
 To change domicile, one must demonstrate an actual removal or
an actual change of domicile; a bona fide intention of abandoning
the former place of residence and establishing a new one, and
definite acts which correspond with the purpose. There must be
animus manendi coupled with animus non revertendi. The
purpose to remain in or at the domicile of choice must be for an
indefinite period of time; the change of residence must be
voluntary; and the residence at the place chosen for the new
domicile must be actual.
 Norlainie's claim that she has been physically present and
actually residing in Pantar for almost 20 months prior to the
elections, is self-serving and unsubstantiated. Norlainie’s
evidence cannot persuade the Court that she has abandoned her
domicile of origin or her domicile in Marawi City.

Bautista, Gastanes, Lopez, Sulit Page | 60

You might also like